The Leviathan

Ace your homework & exams now with Quizwiz!

What are the taste cells innervated by?

Taste cells in individual taste buds synapse with primary afferent axons from branches of three cranial nerves: the facial (VII), glossopharyngeal (IX), and vagus (X) nerves. The taste cells in fungiform papillae on the anterior tongue are innervated exclusively by the chorda tympani branch of the facial nerve; in circumvallate papillae, the taste cells are innervated exclusively by the lingual branch of the glossopharyngeal nerve; and in the palate they are innervated by the greater superior petrosal branch of the facial nerve. Taste buds of the epiglottis and esophagus are innervated by the superior laryngeal branch of the vagus nerve.

Where are upper motor neurons located?

TWO PRIMARY LOCATIONS Motor cortex (in frontal lobe) • voluntary movement • skilled movements Brainstem • posture and body position • locomotion • orientation to sensory stimuli • emotional expression

In somatosensory cortex, the area representing the foot is supplied by what cerebral artery?

anterior cerebral artery

What is dystonia?

Persistently distorted body or limb posture.

Describe the ventral spinocerebellar tract

Carries information from lower trunk and leg. Proprioceptive axons enter the cord and synapse on cells around the border of the ventral horn; axons of the second order cells cross the midline, travel up the cord and brainstem contralaterally, enter the cerebellum through the superior cerebellar peduncle, and then cross back.

What are cataracts?

Cataracts (increased opacity of lens).UV exposure contributes. Surgically correctable.

What receptor mechanism confers specificity for salty tastes?

Cation selectivity of the amiloride-sensitive Na+ versus H+ sensitive proton channel provides the basis for specificity of salty tastes.

What makes up the dorsal basal ganglia?

Caudate nucleus, Globus Pallidus, Putamen

Where are the cell bodies of the vestibular nerve located?

Cell bodies of vestibular nerve located in Vestibular (Scarpa's) Ganglion

Describe the 3rd order neuron of the dorsal column system.

Cell body in VPL. Axon projects through the posterior limb of the internal capsule, and then through the corona radiata to terminate in somatosensory cortex in the postcentral gyrus. Within the posterior limb of the internal capsule, body topography is maintained with axons for lower limbs traveling most caudally and for upper limbs traveling most rostrally (illustration of this later, in discussion of trigeminal system).

Describe the 1st order neuron of the dorsal column system.

Cell body in dorsal root ganglion. Central process (axon) enters the cord and ascends in the dorsal columns to synapse in the dorsal column nuclei. Information from the lower body travels in fasciculus gracilis; information from the upper body travels in fasciculus cuneatus.

Describe the 2nd order neuron of the dorsal column system.

Cell body in the dorsal column nuclei (nu. cuneatus and gracilis). Axons cross midline as the internal arcuate fibers to form the medial lemniscus. These axons synapse in the ventral posterior lateral nucleus (VPL) of the thalamus.

Generalize the cell layers of the retina

Cells and layers of the retina. Photoreceptors (rods and cones) receive light input. Horizontal cells and bipolar cells receive input from photoreceptors. The bipolar cells synapse onto amacrine cells and ganglion cells. Ganglion cell axons form the optic nerve which ascends to the thalamus. All cells in the retina prior to ganglion cells have graded potentials, while ganglion cells are the first in the circuit to have action potentials.

What is the role of the long-term regulation of feeding?

Long-term regulation of feeding is part of a homeostatic mechanism to maintain energy balance

What muscles does CN IV innervate?

Superior Oblique

What is the superior cerebellar peduncle connection called?

Superior cerebellar peduncle = brachium conjunctivum -OUTPUT

Where is visual acuity highest?

Visual acuity is highest in fovea.

Which of the following can result from cerebellar lesions: a. resting tremor b. fasciculations c. ataxia d. nystagmus e. atrophy f. paralysis

-Ataxia -Nystagmus

If a patient has a stroke blocking the artery described in question 1, what will be the main deficit in eye movements? Which way would the patient's eyes probably deviate at rest - left or right?

-Can't make voluntary eye movements to the left. -Right

What are the motor components of the basal ganglia?

-Caudate nucleus -Putamen -Globus pallidus (external Globus pallidus, internal) -Subthalamic nucleus -Substantia nigra (pars compacta, pars reticulata)

What are the subdivisions of the trigeminal nuclei?

-Chief Sensory Nucleus -Spinal Trigeminal Nucleus -Mesencephalic Nucleus

What are the upper motor neuron pathways divided into?

-Classically, the UMN pathways were divided into pyramidal and extrapyramidal systems. -The pyramidal system was so named because its axons traverse the pyramids in the medulla; this system consists of the corticospinal axons (and the functionally related corticobulbar pathway). -The extrapyramidal system comprises the descending tracts that originate in the other UMN areas, plus the basal ganglia and their connections with motor regions. You will hear this nomenclature frequently (it has been used for a long time), but the concept of medial vs. lateral systems, and the concept of feedforward/feedback control, have provided more useful functional perspectives.

Describe isolated and coactivation of alpha and gamma motor neurons

-Co-activation of alpha and gamma motor neurons usually occurs in voluntary muscle contraction. When only alpha neurons are activated (A), the intrafusal fibers stay relaxed while the surrounding muscle contracts, and Ia discharge ceases. -With co-activation of alpha and gamma neurons, the shortening of the spindle caused by extrafusal fiber contraction is coupled with contraction of intrafusal fibers, and the Ia discharge persists.

Briefly describe the cochlea

-Coiled structure contains receptor organ. Like the vestibular receptor organ, the sensory cell is the hair cell.

What are the vestibulospinal tracts?

2 tracts: medial vestibulospinal and lateral vestibulospinal • both tracts are part of the "medial system" of descending motor control

Describe oxytocin's social role

2. Oxytocin neurons of the hypothalamus project to many regions of the brain, including to limbic structures (see next lecture). The release of oxytocin is thought to promote many social behaviors, including: a. Mother-infant bonding b. Bonding among mating pairs c. Social bonding

What are the main characteristics of nociceptors?

Nociceptor axons conduct relatively slowly, are either only lightly myelinated, or are unmyelinated. Nociceptors are characterized according to the properties of axons associated with them

What is the function of the Spinothalamic pathway?

"Fast pain", i.e., sharp, well-localized stabbing pain. (This pathway is sometimes referred to as the "lateral spinothalamic pathway".)

Describe the anatomy and function of the primary motor cortex

Precentral gyrus and paracentral lobule Execution of fine, skilled movements.

What are the two main types of cerebellar inputs?

1. Climbing Fibers 2. Mossy Fibers -Each input fiber BRANCHES, with one branch going to the deep nuclei and one branch to cerebellar cortex.

Describe the anatomical pathways of the medial vestibulospinal pathway

- Semicircular canals provide input to the medial vestibular nucleus. - Output fibers of MVN descend in medial longitudinal fasciculus (mlf) to cervical levels, mostly ipsilateral, providing excitatory and inhibitory input to motorneurons of the neck muscles.

What is an anticipatory action?

"Pull the green handle when you hear the tone" -Biceps contraction to pull the handle, gastrocnemius contraction to prevent falling forward. ***NOTE: gastroc. contraction precedes biceps contraction.***

What is the function of the Spinoreticular pathway?

"Slow pain", dull, aching or burning pain. The pain is not well-localized. (This pathway is sometimes referred to as the "anterior spinothalamic pathway".)

What is dysmetria?

"mismeasurement".

What are the sub modalities of somatic sensation?

(1) Discriminative touch: recognition of size, shape and texture of objects and their movement across the skin. (2) Proprioception: the sense of static position and movement of the limbs and body. (3) Nociception: signaling tissue damage or chemical irritation, perceived as pain or itch. (4) Temperature sense: warmth and cold.

Describe Vanilloid receptors (VR)

(1) Most VR receptors are sensitive to both heat and capsaicin, the ingredient in chili peppers that causes the burning or tingling sensation produced by spicy foods. Since the receptor is sensitive to both capsaicin and heat, peppers seem "hot". (2) Some VR receptors are sensitive only to heat and not to capsaicin. Others are activated by acids (H+). (3) VR receptors are found in C and A-Delta fibers (on their free nerve endings). (4) Different VR receptors are activated by different heat levels: the VR-1 receptor is activated by moderate heat (45oC) that is considered uncomfortable, while the VRL-1 receptor has a higher heat threshold (52oC). (5) VR receptors are members of the TRP (transient receptor potential) family: many other somatosensory receptors also belong to the TRP family.

Describe the representation of the visual field within the visual area of the calcimine sulcus

(A) An overview of the visual fields and of the visual pathway from above. (B) The retinogeniculate and geniculostriate pathways in the sagittal plane. Input from the inferior visual field reaches the upper bank of the calcarine sulcus. Note the disproportionately large representation of the macula; the central 10 degrees of visual field space occupies about one half of the visual cortex.

Describe the location of nociceptors

(A) The cell bodies of pain afferent fibers reside in a series of dorsal root ganglia (DRG) along the spinal cord or in the trigeminal ganglia. (B) Pseudo-unipolar neurons in the DRG give rise to a peripheral branch of the pain nerve fiber that ramifies within the skin or muscle. Afferents carrying pain and temperature information terminate as free endings. The second branch of the pseudo-unipolar neuron gives rise to a central process that synapses with neurons in the spinal cord or brainstem.

Describe the motor unit

(A) The motor unit. Diagram showing a lower motor neuron in the spinal cord and the course of its axon to its target muscle. (B) Each motor neuron synapses with multiple fibers within the muscle.

What is Dysgeusia?

(Parageusia): distortion of taste (result of medication).

An uncal herniation can press the III nerve against the tentorial notch, reducing or eliminating conduction in the nerve. Describe the effects on eye movements if the right III nerve is compromised. How will the right pupil compare to the left pupil?

- Eye likely to be deviated laterally at rest (unopposed lateral rectus); few movements possible. -Loss of parasympathetics in III means right pupil will be dilated compared to left pupil.

Describe the anatomical pathways of the lateral vestibulospinal pathway

- Lateral and inferior vestibular nuclei receive primary afferent input from maculae of utricle and saccule, also input from cerebellum and spinal cord. - Axons of the lateral and inferior vestibular nuclei descend in the lateral vestibulo-spinal tract, and terminate ipsilaterally within the anterior horn of the spinal cord, at cervical through lumbar segments. - Axons contact (directly and via spinal interneurons) alpha and gamma motorneurons to antigravity muscles (mainly axial and extensor musculature).

What are some of the targets of deep cerebellar nuclei outputs?

- VL of thalamus - reticular formation - red nucleus - vestibular nuclei - superior colliculus

Describe the restiform body connections

- restiform body (bulk of ICP) • spinal cord • inferior olivary nucleus • reticular formation

What should one keep in mind when discussing the CEREBELLAR PEDUNCLES?

--DO NOT CONFUSE THESE WITH THE CEREBRAL PEDUNCLES -The cerebral peduncles are at the base of the midbrain. The base of each cerebral peduncle, the "basis pedunculi" or "crus cerebri", contains the corticospinal, corticobulbar, and corticopontine fibers. There are 2 cerebral peduncles (one on each side), and 6 cerebellar peduncles (3 on each side). Damage to the cerebral peduncles can cause UMN syndrome. Damage to the cerebellar peduncles would cause "cerebellar" signs (described below).

Describe the 1st, 2nd, and 3rd order neurons of the Spinothalamic pathway

-1st order neuron: Aδ fibers with cell body in dorsal root ganglion. -2nd order neuron: Cell bodies in laminae I and V of dorsal horn. Axons cross midline in the anterior white commissure to the anterolateral aspect of the spinal cord and brainstem. -3rd order neuron: cell body in VPL. Axons of VPL cells project through the posterior limb of the internal capsule to terminate in somatosensory cortex in the postcentral gyrus, which is important for the localization of pain.

Describe the projections of the Spinoreticular pathway

-1st order neuron: C-fibers with cell body in dorsal root ganglion. Higher order neurons: -2nd-order neuronal cell bodies are in lamina II (also known as substantia gelatinosa) of the dorsal horn in the cord. This pathway is multisynaptic with no single locale for 3rd-order neurons, so we abandon that terminology here. -The 2nd order neurons project to the supraspinal levels and to other spinal laminae, which then project up to the reticular formation (bilaterally). The contralateral portion of the projection crosses the midline in the anterior white commissure. -From there information goes to the intralaminar nuclei of the thalamus. These extra synapses to reach the thalamus take more transmission time, contributing to the slowness of the pain. These pathways are not as well organized somatotopically; slow pain is not well-localized. -Projections from the intralaminar nuclei of the thalamus travel through the internal capsule and corona radiata to reach multiple cortical areas, including the postcentral gyrus, insula and anterior cingulate gyrus. The latter 2 areas probably contribute to the affective (suffering) aspects of pain, whereas the postcentral gyrus is more important for accurate localization of pain.

What is dysarthria?

-A general term for disorder in speech (referring to difficulty in articulation). -Dysarthria can occur from cerebellar or non-cerebellar motor disorders. It can take the form of scanning speech, in which speech is slower and presented one-word-at- a-time. This is a sign of midline cerebellar lesions.

Describe the purpose of the Crossed-extensor reflex (also called flexion-crossed extension reflex)

-Activated by flexor reflex -Main effect is to activate limb extensor musculature on side of body contralateral to nociceptive input. -Nociceptive fibers terminate on interneurons that cross the midline to the ventral horn of the contralateral side -Spinal cord circuitry responsible for the flexion- crossed extension reflex. -Stimulation of cutaneous receptors in the foot (by stepping on a tack) leads to activation of spinal cord local circuits that serve to withdraw (flex) the stimulated extremity and extend the other extremity to provide compensatory support.

What does activity in the motor cortex lead to?

-Activity in primary motor cortex leads to coordinated, goal-directed movements -THEREFORE: Goal-directed movements, not individual muscles, or even direction of movement, are represented in motor cortex.

What is spinal shock? How does it factor into the equation of determining between LMN and UMN syndrome?

-After a sudden injury of UMNs, particularly with a lesion in cortex or internal capsule, the initial clinical picture is different: • flaccid muscles • hypotonia • hyporeflexia -All deficits contralateral to the lesion. -This is known as spinal shock. -Over time - days to weeks - the picture changes to classic UMN syndrome.

Describe the interaction of the red nucleus and the cerebellum

-And, one more discussion of the red nucleus. The red nucleus has two parts (we don't identify them by name); one connected to cerebellum and inferior olive and one a source of UMNs. As pointed out in the UMN lectures, this latter part is very small in humans. -A major rationale for focusing on the cerebellar connections of the red nucleus, and virtually ignoring the UMN aspect, is that a lesion of the red nucleus in humans actually yields primarily cerebellar signs. This can be interpreted as resulting from interruption of both the cerebellum-red nucleus-inferior olive circuit and the almost certain interruption of cerebellar output fibers in the superior cerebellar peduncle that pass through or around the margins of the red nucleus as they travel onward to the VL of the thalamus. -Now, if a lesion occurs in the left red nucleus, and includes the adjacent cerebellothalamic fibers, which side of the body would you expect to exhibit cerebellar signs? (Answer: right). Don't panic if this doesn't make sense immediately. However, it is worth continued effort until you do understand it; we will reinforce this issue with exercises in lab.

Describe the electrical effect of odorant binding to olfactory cells

-Applying odorants to the cilia evokes a slow receptor potential. The receptor potential propagates down the dendrite to the soma of the olfactory receptor cell. Action potentials are first evoked at the axon initial segment (and recorded in the soma). These action potentials then propagate down the axon of the receptor cell. -Generation of receptor potentials in response to odors takes place in the cilia of receptor neurons. -Odorants evoke large depolarizations (inward currents) when applied to the cilia (left panel) but only small currents when applied to the cell body (right panel).

What forms the lateral olfactory tract? What does it target?

-Axons from mitral and tufted cells form the lateral olfactory tract. Lateral olfactory tract fibers terminate in the olfactory cortex which is a broadly defined area on the ventral surface of the telencephalon. -Areas that are part of the olfactory cortex are: anterior olfactory nucleus, olfactory tubercle, piriform cortex, anterior cortical amygdaloid nucleus, peri-amygdaloid nucleus and lateral entorhinal cortex. Lateral olfactory tract axons caudally form lateral olfactory stria which terminate in the olf tubercle and piriform cortex. Posterior portion of the lateral olfactory tract terminates in amygdala and entorhinal cortex.

Describe the main taste pathway

-Axons from rostral (gustatory) part of the solitary nucleus project to ventral posterior complex of the thalamus, where they terminate in the medial half of the ventral posterior medial nucleus. From here projections go to regions of neocortex including anterior insula in the temporal lobe (insular taste cortex) and the operculum of the frontal lobe. The pathway from solitary nucleus to ventral posteromedial nucleus to cortex - underlies discriminative aspects of taste, and is exclusively ipsilateral (unlike other sensory systems).

Describe the histology of the primary motor cortex.

-Brodmann's area 4, in precentral gyrus -Projections arise from layer 5 of cerebral cortex

Describe feedforward motor control for balance

-Cortical projections to the reticulospinal tracts 1. Premotor Cortex (corticobulbar projections) 2. Reticular Formation (Reticulospinal tracts) 3. Spinal cord ventral horn

Where do the upper motor neurons in the brainstem project to?

-Descend through the anterior-medial white matter of the spinal cord -Project to the lower motor neurons in the medial ventral horn -Control the axial and proximal limb muscles (balance and posture)

Where do the upper motor neurons of the cerebral cortex project to?

-Descend through the lateral white matter of the spinal cord -Project to the lower motor neurons in the lateral ventral horn -Control the distal limb muscles (skilled movements)

Describe the effect of lowered dopamine on the basal ganglia

-Direct pathway drives movement. DA excites direct pathway. Loss of DA means direct path is less active in PD. -Indirect path inhibits movement. DA inhibits the indirect pathway. Loss of DA means indirect path is more active in PD.

What does the Dorsal Column Medial Lemniscus System do?

-Discriminative Touch -This system carries most of what we refer to casually as "touch", excluding pain and temperature. Specific information (and the tools for assessment) include discriminative TOUCH (two-point discrimination tested with calipers), "FINE TOUCH" (cotton ball), ability to sense PRESSURE on the skin, VIBRATION (tuning fork), and CONSCIOUS PROPRIOCEPTION (with eyes closed, patient reports position of limbs as they are moved by examiner).

What is Meniere's Disease?

-Due to increased endolymphatic fluid pressure (endolymphatic hydrops) -Severe, recurrent episodes (lasting for less than 4 hours) include severe vertigo, nystagmus, and auditory symptoms -Eventual loss of receptor cells (with accompanying loss of vestibular function) unless controlled -Meniere's probably has several origins

Describe the flexor reflex (nociceptive reflex)

-Flexor reflex (also called nociceptive reflex) Stimulus is nociceptive input to A-delta or C fibers -Nociceptive fibers terminate on interneurons -Excitatory interneurons terminate on flexor A-alpha motorneurons. These activate flexor muscles to move the limb away from the nociceptive input. -Inhibitory interneurons terminate onto alpha motor neurons that supply the extensor muscles to the limb. The result is to suppress contraction in the extensors that would oppose the action of the flexor muscles.

What is the goal of the lateral premotor cortex?

-For movements based on external events. -Lesions: may lose ability to follow verbal commands or sensory cues

What is the goal of the medial premotor cortex?

-For movements based on internal events. -Spontaneous movements -Movements from memory Lesions: reduction in self-generated movements

Describe the anatomical pathways of the vestibulo-ocular reflex

-Hair cells of ampullary crests of semicircular canals provide input to the medial (MVN) and superior vestibular (SVN) nuclei. -Output fibers of MVN and SVN project to oculomotor nuclei (III, IV, VI), to extraocular muscles controlling eye movement. Otolith organs and lateral vestibular nucleus are involved, but to a lesser extent.

Briefly describe the anatomy of CN VIII in the inner ear.

-Hair cells synapse upon vestibular nerve fibers (with cell bodies in the Vestibular or Scarpa's ganglion), or on auditory nerve fibers with cell bodies in Spiral ganglion. -These two nerves combine to form the VIIIth cranial nerve and pass through the skull at the internal auditory meatus.

What are signs and symptoms of auditory system dysfunction?

-Hearing loss -Tinnitus: perception of sound when there is no sound. -Impairments of speech comprehension

Describe the hypothalamospinal pathways

-Hypothalamospinal pathway is major descending pathway controlling sympathetic outflow. It is direct, ipsilateral to intermediolateral cell column of thoracic and lumbar cord 1. Functions include increased cardiovascular function (heart rate, blood pressure, vasoconstriction), increased ventilation, sweating, piloerection, adrenal medullary stimulation, pupillary dilatation, decreased G-I tract secretions and motility, orgasm 2. lesions of hypothalamospinal tract are one cause of Horner's syndrome, a loss of sympathetic outflow to ipsilateral face, head, and body. Some signs include: -pupillary constriction (miosis) -loss of sweating (anhydrosis) -slight drooping of eyelid (ptosis) -enopthalmos (sunken appearance of eye)

What is necessary for LH and FSH secretion in males and females?

-In males and females, the pulsatile and coordinated release of GnRH (once every few hours, to a few times a day) is necessary for proper secretion of gonadotropins from the anterior pituitary. -Continual infusion of GnRH into anterior pituitary stops secretion of gonadotropins LH and FSH -Some forms of precocious puberty can be treated by long- acting drugs that evoke sustained GnRH release. Net result is elimination of gonadotropin output that reverses pubertal development

Explain the size principle and the regulation of muscle force.

-Increasing or decreasing the number of motor units active at any given time changes the amount of force produced by the muscle. Progressively increasing the number of motor units recruited results in gradually increasing muscle tension. -Motor units are recruited according to their size. The smallest motor units are recruited by the weakest synaptic stimulation. As the strength of the stimulus increases, recruitment of motor units takes place in the order of small to large. This systematic recruitment of motor units according to their size is called the size principle.

What is ataxia?

-Lack of coordination ("clumsiness") of voluntary movements. May include delay in initiating movements, errors in range of movement (dysmetria), and errors in the rate or rhythm of movements. Patients with cerebellar damage often have a "drunken sailor's gait"; they also frequently have a very wide stance. -Ataxia is often associated with cerebellar damage, but may also be caused by other lesions. One can imagine, for example, that arm movements would be poorly coordinated - i.e., ataxic - following lesion of primary somatosensory cortex because the affected limb would feel "numb".

Compare and contrast the crossing of the lateral and medial motor systems

-Lateral system is crossed -Medial system is bilateral

What is Weber's Test?

-Lateralization test -Simple hearing test

What results from varying lesions of the premotor and primary motor cortices?

-Lesion all: paralysis, weakness -Lesion "just" primary motor cortex: primarily loss of fine movements. -Lesion "just" premotor cortex: loss of coordination and ability to perform complex series of movements. This deficit is known as "apraxia".

Describe the role of the solitary nucleus in taste

-Main visceral nucleus in brainstem. -Two regions: rostral gustatory nucleus, caudal cardiorespiratory nucleus. -Taste fibers (nerves X, IX, VII) terminate in rostral solitary nucleus. Termination is topographic along the rostral-caudal axis of the rostral portion of the nucleus of the solitary tract (also called gustatory nucleus). Facial nerve terminates rostrally, glossopharyngeal nerve in the mid region and vagus nerve caudally. This topographic arrangement allows for integration of information from visceral and taste systems. -The caudal part of n. solitarius receives innervation from sub-diaphragmatic branches of the vagus, which control gastric motility.

Describe the mechanism of sound transduction of cochlear hair cells

-Mechanism of hair cell activation is very similar to vestibular hair cells. 1. Motion of basilar membrane in response to sound causes hair cells to move relative to tectorial membrane. (Fig. 17) 2. Shearing of hair cell stereocilia in one direction increases potassium influx, depolarizing hair cell. Shearing in other direction reduces potassium influx below resting levels, hyperpolarizing cells. 3. Depolarized hair cells release excitatory neurotransmitter onto distal processes of auditory nerve fibers, causing them to discharge action potentials.

What are the main functional classes of sensory receptors?

-Mechanoreceptors -Thermoreceptors -Photoreceptors -Chemoreceptors -Nociceptors

What are the most common color vision deficits?

-Most common deficits involve L (red) and M (green) cones 1. Genes for L and M cones are located on X chromosome, so males are much more likely to display these deficits. ~6% of men have anomalous red or green pigments, while ~ 2% of men are missing red or green pigments entirely. 2. Result is reduced sensitivity to light at wavelengths corresponding to the missing photopigment, and reduced ability to distinguish colors in the green to red part of the spectrum.

Describe the outputs of the basal ganglia

-Most outputs originate from the globus pallidus, internal segment (GPi) and the substantia nigra, pars reticulata (SNr). -The major targets of basal ganglia output are the VA and VL nuclei of the thalamus, which project to motor cortex (broadly defined; i.e., primary motor cortex and premotor areas; Brodmann's areas 4, 6, 8). -A smaller output goes to the superior colliculus and reticular formation. -The outputs are GABAergic and inhibitory. VA/VL projections excite motor cortex, leading to movements. Thus, anything that increases basal ganglia output will inhibit the VA/VL and reduce overall movements, and vice versa.

Compare the function of muscle spindles to the function of the golgi tendon organ

-Muscle spindles and GTOs have different responses to passive muscle stretch (top figure) and active muscle contraction (bottom figure). -Both afferents discharge in response to passively stretching the muscle, but the GTO response is much less than that of the spindle. When the extrafusal muscle fibers are made to contract by stimulation of their a motor neurons, however, the spindle is unloaded (relieved of tension) and therefore falls silent, whereas the rate of GTO firing increases.

Describe Muscle spindles

-Muscle spindles are found in most striated (skeletal) muscles. -A muscle spindle consists of four to eight specialized intrafusal muscle fibers surrounded by a capsule of connective tissue. The intrafusal fibers are distributed within and in a parallel arrangement with extrafusal fibers of skeletal muscle, which are force producing fibers. Sensory afferents are coiled around the central part of the intrafusal spindle. -When the muscle is stretched, tension placed on the intrafusal fibers activates mechanically gated ion channels in the nerve endings, triggering action potentials.

Where do the OVLT and SFO organs project to?

-Neurons in both NST and SFO project to the magnocellular neurons of the SON and PVN. Result in increased secretion of ADH. -Both NST and SFO neurons may project to the dorsomedial hypothalamus to contribute to thirst. -Neurons of lateral hypothalamic area activate sympathetic responses to lowered blood volume, e.g., vasoconstriction, increased force of cardiac contraction

Describe the neurons of the sympathetic nervous system

-Neurons in the sympathetic NS innervate targets throughout the body. These neurons are located in the intermediolateral cell column in the thoracolumbar region of the spinal cord. -The sympathetic NS governs functions that allow the body to mobilize resources for maximal utility; hence, the "fight or flight" designation. Some fight or flight responses include pupil dilation, increased heart rate and stroke volume, bronchial dilation, inhibition of peristalsis and secretion of catecholamines by the adrenal gland. -These responses use energy (catabolic). The sympathetic NS is tonically active. The adrenal medulla, sweat glands, piloerector muscles and arterial blood vessels only receive innervation from the sympathetic division of the ANS.

What is Nystagmus?

-Normal physiological nystagmus is a fast eye movement following the slow movements of the vestibular ocular reflex. The fast movement is in the direction of head rotation, and its function is to return the eyes to the center of the orbit. (see Nystagmus Worksheet) -Clinically observed nystagmus may involve alternating fast and slow movements in opposite directions. It is common to give the direction of the fast phase. -Spontaneous nystagmus or inappropriate nystagmus are signs of damage to vestibular system or cerebellum

Describe the premotor cortex in macaque

-Not well documented in humans -Note locations of CORRECTED: frontal eye field and, (the corrected) cingulate motor areas

How is nociceptive pain caused?

-Peripheral nerve endings on pain fibers (e.g. in the skin) are activated by different chemicals that are released following tissue damage. Tissue damage releases bradykinin, prostaglandins, serotonin (5-HT), etc, which activate nociceptors. Nociceptor activation leads to the release of substance P and CGRP (calcitonin gene related peptide) from the nociceptor nerve endings. Substance P acts on mast cells which release histamines which directly re-activates nociceptors. CGRP and substance P dilate blood vessels which causes edema. -Nociceptive pain can also occur without inflammation, through direct damage to the nerve terminals without the release of secondary chemicals.

Describe the primary motor cortex

-Primary motor cortex is essential for voluntary movement. It provides for fine control of individual muscles, especially for fine movements of a single digit or part of digit. -Primary motor cortex is somatotopically organized. The homunculus is distorted, with larger areas of cortex devoted to regions with finer muscle control..

How is proprioception tested?

-Proprioception is tested by asking the patient to close her eyes and report limb position as the examiner moves individual limbs or digits. -The Romberg test also assesses proprioception. The patient stands with eyes open and feet together, and the examiner assesses how much the patient sways. The patient then closes his eyes. If swaying increases, inadequate proprioceptive input is reaching the brainstem (with eyes open, the visual system was compensating for loss of proprioceptive input). The Romberg test is positive if swaying increases with eyes closed, indicating damage to dorsal column system.

What is the function of the lateral hypothalamic area?

-Receives projections from arcuate nucleus -Its activation stimulates feeding behaviors -Cells containing MCH (a peptide) project throughout brain, including to cerebral cortex: appears to activate feeding behaviors

What are the primary characteristics of the Analysis of visual features in visual cortex

-Receptive field properties -Orientation selectivity -Visual form and motion

Describe vestibular neuritis

-Selective inflammation of vestibular nerve, often due to viral infection. Includes vertigo and nystagmus. Often lasts >24 hours

What are some auxiliary memory structures?

-Septal nuclei -Medial dorsal nucleus and pre-frontal cortex also participate in learning/memory

How many different types of olfactory receptors are there?

-Signal transduction in the olfactory system occurs via approximately 1000 distinct olfactory receptor molecules. -Each receptor cell expresses only ONE receptor molecule type. The structure of each receptor gene is unique, so that each receptor type expresses a unique protein. -This allows each receptor type (protein) to bind a different odorant. (i.e. each olfactory receptor cell seems to express just one - or only a very few - of the 1000 types of receptor genes).

Describe Merkel Cell Afferents

-Slowly adapting fibers; account for ~25% of the mechanosensory afferents in the hand. -Enriched in fingertips, and are the only afferents to sample information from receptor cells located in the epidermis. -Merkel cell afferents have the highest spatial resolution of all the sensory afferents - individual Merkels can resolve spatial details of 0.5 mm. -They are highly sensitive to points, ridges, and curvature; they process information about form and texture.

Describe Ruffini Afferents

-Slowly adapting fibers; elongated, spindle-shaped, capsulated, and located deep in the skin, and in ligaments and tendons. -The long axis of the corpuscle is oriented parallel to the stretch lines of the skin: Ruffini corpuscles are particularly sensitive to the cutaneous stretching produced by digit or limb movements, or movements of fingers. -They account for ~20% of the mechanosensory hand receptors. -Together with muscle receptors, Ruffinis provide information to the CNS about finger position and conformation of the hand.

Where are sour and bitter receptors located?

-Sour and bitter: lowest at tongue tip. Greatest on back and sides. -Behavior elicited: grimaces, puckering, high salivary secretion to dilute tastant.

Describe stimulation of prostatic or vaginal secretions during the sexual response

-Stimulation of prostatic or vaginal secretions -Mechanoreceptors that record genital stimulation synapse in brainstem nuclei; information is conveyed to somatosensory cortex; parasympathetic activity increases

Compare and contrast lesions of the corticobulbar tract above and below the facial nucleus

-Strokes involving the motor cortex or internal capsule commonly affect corticobulbar projections to the facial nuclei. Facial nerve problems are also common, due to a tumor or to Bell's palsy (temporary loss of facial nerve function). Corticobulbar lesions paralyze the lower face; facial nerve/nucleus lesions paralyze whole side of face. -If we consider the facial nucleus as a reference point, you will understand a term commonly applied to lesion of the corticobulbar system: "supranuclear". A supranuclear lesion is above the (facial) nucleus, and the deficits are of the corticobulbar pattern. The term supranuclear can be applied to any corticobulbar projection, not just that to the facial nucleus.

What are the general features of the retina? What cells does it contain?

1. Pigment Epithelium 2. The two main photoreceptor cell types 3. Bipolar Cells 4. Horizontal Cells, Amacrine Cells 5. Ganglion Cells

Describe taste cells. What pathways are involved in their transduction?

-Taste cells are polarized epithelial cells with an apical and basal domain separated by tight junctions. Tastant-transducing channels (salt and sour) and G-protein-coupled receptors (sweet, amino acid, bitter) are limited to the apical domain. -Intracellular signaling components that are coupled to taste receptor molecules are also enriched in the apical domain. Voltage-regulated Na, K, and Ca channels mediate neurotransmitter release from presynaptic specializations at the base of the cell onto terminals of peripheral sensory afferents. These channels are limited to the basolateral domain. -The neurotransmitter, serotonin, among others, is found in taste cells, and serotonin receptors are found on the sensory afferents. TRP (transient receptor potential) channels, which facilitate G-protein coupled receptor-mediated depolarization, are expressed in taste cells.

Describe the neurotransmitters of the basal ganglia

-The basal ganglia use multiple neurotransmitters. -Acetylcholine is another neurotransmitter that is important in basal ganglia function. Acetylcholine is used by interneurons in the striatum [because they are interneurons, and therefore do not project out of the striatum, they are not drawn in the circuit diagram]. The output of both direct and indirect pathways is affected by acetylcholine, and drug therapy for various basal ganglia diseases takes into account the cholinergic circuitry.

Describe the indirect pathway of the basal ganglia

-The indirect pathway (dotted line in figure) goes from striatum to GPe to the subthalamic nucleus, to GPi. As in the direct pathway, the projection from the striatum to GP (in this case, GPe) is inhibitory. There is an additional loop through the subthalamic nucleus that includes an inhibitory link (GABA) and an excitatory link (glutamate). -Trace the pathway: increased activity in cortical inputs leads to increased activity in the subthalamic nucleus, which increases GPi activity and thus inhibits the VA/VL. Reduced activity in VA/VL causes reduced activity in motor cortex. The net effect is that activity in the indirect pathway leads to a decrease in motor cortex activity and decreased movement.

What is the source of voluntary movements?

-The motor cortex is the source of voluntary movements. A lesion of motor cortex leads to loss of voluntary control, i.e., paralysis, of the affected muscles. -The other sources of UMNs are involved in various levels of reflexive motor control, and are to varying extents also under the influence of motor cortex.

What do upper motor neuros project to?

-They are neurons that project to lower motor neurons and associated local circuits

Describe the ventral corticospinal tract

-The remaining fibers (those that do not cross in the pyramidal decussation) descend as the anterior corticospinal tract. This tract is functionally similar to other medial pathways, except that its cortical origin is associated with direct voluntary motor control. When these axons reach the appropriate level of the spinal cord, they terminate in the medial ventral horn. -Some texts say they cross before terminating, while others suggest they end bilaterally. The difference is unimportant from a functional standpoint, because circuitry in the cord connects the two sides to coordinate the trunk muscles that the ventral corticospinal tract would control. If the ventral corticospinal tract were lesioned on one side, the one on the contralateral side could compensate to some degree.

Describe the intertwined structure and function of the semicircular canals

-The sensory organ, or ampullary crest, consists of hair cells (and support cells) that project into the ampulla, or swelling of each canal (Fig. 7). Hair cells are embedded in a gelatinous mass called the cupula. When the head undergoes rotational acceleration in the plane of the canal, the relative motion of the endolymphatic fluid against the cupula causes a shearing action that either depolarizes or hyperpolarizes the hair cell. -Within the ampullary crest of a given canal, all hair cells are aligned in the same direction, so all are similarly excited by motion of the endolymph in a particular direction. -Each semicircular canal is ONLY affected by head movement ONLY within the PLANE of that canal.

What does the somatic sensory system mediate?

-The somatic sensory system mediates a range of sensations: touch, pressure, vibration, limb position, heat, cold, pain. -These sensations are transduced by receptors within the skin or muscles and conveyed to different nervous system targets.

Briefly describe thermal receptors and nociceptors

-Thermoreceptors sense temperature - both heat and cold, while nociceptors respond to noxious stimuli, that result in a sensation of pain. Both receptor types are free nerve endings and are found in skin, muscles and joints and on internal organs. (Nociceptors and thermoreceptors will be studied in detail in the lecture on Pain Mechanisms).

Describe the odd pattern of facial muscle deficits that are observed with lesions of the corticobulbar tract on one side as compared to the facial nucleus on one side

-This one is complicated, because there are bilateral projections to the portion of the facial nuclei that innervate the upper face, whereas there are only contralateral projections for the parts of the nucleus that innervate the lower face. -Damage to the right corticobulbar tract will cause paralysis of the muscles of the left lower face. The left forehead and eyelid is innervated by the part of the left facial nucleus that has lost input from right cortex but still has input from the intact, left cortex. This pattern of loss differs from that of damage to the facial nucleus or nerve on one side, which would cause paralysis of all (upper and lower) facial muscles on that side.

What is Rinne's Test?

-Threshold by air/bone conduction -Simple hearing screening test

Summarize the cortical motor areas

-To review the motor cortical areas, the 3 largest motor cortical areas are primary motor cortex (Brodmann's area 4), premotor cortex (lateral part of area 6), and supplementary motor cortex (medial part of area 6). Other cortical areas important for motor function include frontal eye fields for eye movements, Broca's area for speech production and cingulate motor areas for emotional expression. -The somatosensory cortex and more posterior parts of the parietal lobe ("higher order" somatosensory areas) provide sensory information necessary for planning and carrying out movements. Parietal somatosensory areas and each of the "other" areas listed above provide input to (i.e., project to) primary and premotor cortical areas.

Describe the vestibular apparatus

-Two chambers and three semicircular canals contain sensory receptor organs. -These include the hair cells (receptor cells) and associated support structures.

Describe spatial discrimination of somatosensory input

-Two-point discrimination -The size of the receptive field and the innervation density determine the spatial accuracy with which tactile stimuli can be sensed. -The minimum inter-stimulus distance required to perceive two simultaneously applied stimuli as distinct is called two-point discrimination.

Describe internuclear ophthalmoplegia

-Vergence is a special form of horizontal eye movements. Recall that vergence is part of the "near reflex", or accommodation for near vision. It is a non-conjugate movement, and it is not controlled by the PPRF. Rather, it is controlled by cells in the midbrain, near the oculomotor nucleus. The significance here is that these cells are spatially separate from the PPRF. -Suppose a patient has a lesion of the left MLF. You ask her to look to the right. Her right eye abducts as it should (although it shows some nystagmus). Her left eye fails to adduct. Now, put your finger directly in front of her and have her visually track your finger as you move it toward her. Her eyes converge; even her left eye can adduct, meaning that her left III nerve and medial rectus are functional. This pattern of deficits - on lateral gaze, one eye abducts with nystagmus and the other eye fails to adduct - is characteristic of a lesion of MLF. The condition is known as "internuclear ophthalmoplegia", and is often a result of multiple sclerosis, which causes scattered regions of demyelination in the nervous system. -The lesion interrupts both the PPRF projections to the oculomotor nucleus and the inhibitory internuclear neurons. The deficits are apparent when the patient is asked to look to the side opposite the lesion. The lesion does not interrupt the regions necessary for vergence. Other patterns of deficits may be observed following MLF damage; the important points to remember are that the MLF carries a variety of types of fibers that serve different roles in eye movements, and that it is possible to damage the MLF without directly damaging the gaze centers or visual motor nuclei.

Describe Mechanoreceptors specialized for proprioception

-While cutaneous receptors provide information about external stimuli, proprioceptors (receptors for "self") provide information about mechanical forces arising within the body itself, particularly from the musculoskeletal system. Proprioceptors provide detailed and continuous information about the position of limbs and other body parts in space. -The two best studied proprioceptors are the muscle spindle and the golgi tendon organ.

Describe the purpose of eye movements

-Why are eye movements needed? Recall that the organization of our retina restricts high acuity vision to the region of the fovea. An important function of eye movements is to direct the fovea to objects of interest. -A second issue is that without eye movements, we literally cannot see. Even during fixation, our eyes are constantly moving, although the movements are so small that we do not notice them. However, if experimental means are used to prevent the movements (by paralysis of the eye muscles), or to move a visual image simultaneously so that the image is stabilized (i.e., does not move) on the retina, the image quickly fades. The fading is presumably due to adaptation of the photoreceptors in the retina, which cease to respond to an unchanging stimulus. The normal "micro- movements" of the eye keep the image moving enough to avoid this adaptation, and thus to allow us to see.

5. Are the following UMNs part of the corticospinal or corticobulbar system: • neurons in motor cortex that project to the lumbar spinal cord. • neurons in motor cortex that project to the facial nuclei.

1. corticospinal 2. corticobulbar.

Describe endolymph

-similar in composition to intracellular fluid (high potassium) -created by specialized epithelial cells -drains into venous sinuses through endolymphatic sac

Describe the pathway of the ascending auditory pathway

1. Auditory Nerve: Fibers of neurons in spiral ganglion project to each division of the ipsilateral cochlear nucleus. 2. Cochlear nucleus: Dorsal (DCN) and ventral (VCN) 3. Axons leave the cochlear nuclei by way of several fiber tracts (Acoustic striae). Most of these axons cross the midline. The largest group of these axons crosses the midline in the Trapezoid Body. 4. Superior olivary complex (SO) 5. Lateral lemniscus is composed of fibers of neurons from superior olive and cochlear nuclei projecting to the inferior colliculus. Cell bodies of the nucleus of the lateral lemniscus lie within this tract. 6. Inferior colliculus (IC) 7. Brachium of the inferior colliculus is composed of fibers of IC neurons projecting to medial geniculate body. 8. Medial geniculate body (MGB) 9. Auditory radiation (part of the internal capsule) is the fiber pathway from MGB to auditory cortex. 10. Primary Auditory cortex (in Transverse temporal gyrus)

Explain how tests of sensory systems are diagnostic of CNS function

1. Basic reflexes test CNS viability 2. Sensory deficits help to identify the location of CNS lesions

What are the main categories of hearing loss?

1. Conductive loss 2. Sensorineural loss 3. Central loss

What is the functional role of the Papez Circuitry?

1. Consolidate short-term memory into long-term memory 2. Forms relations among multiple elements of a memory • memory in context, memory of facts and events (declarative memory) • spatial memory

What do sensory systems contribute to?

1. Contribute to reflex regulation of our internal state (e.g. arterial pressure receptors and chemoreceptors, muscle stretch receptors) 2. Contribute to our state of arousal 3. Contribute to the perception of the external world and to the perception of our bodies with respect to the external world (e.g. smell, taste, vision, touch, hearing, vestibular sense)

Generalize the motor system players involved in eye movements

1. Cranial nerve nuclei III, IV, and VI 2. Reticular Formation (gaze centers) 3. Vestibular System (VOR, Smooth Pursuit) 4. Motor Cortex (FEF, other cortical areas) 5. Cerebellum (all eye movements) 6. Basal Ganglia (Recall paucity of eye movements in Parkinson's)

How are the frequency distinguishing mechanisms of the cochlea regulated?

1. Energy-utilizing mechanisms are responsible for full sensitivity and frequency selectivity of hair cell responses. 2. Outer hair cells play a big role 3. Efferent innervations are regulatory

Describe Alzheimer's

1. Entorhinal cortex and hippocampal formation are involved at an early stage. 2. Cholinergic cells of septal nuclei (and basal nucleus of Meynert) are lost

Compare cones to rods

1. Larger, more concentrated in the center of the retina 2. Different shape: no membrane around outer section, have invaginations of cell membrane where photopigments are located 3. Operate at moderate to high levels of light. Less saturation 4. Quick return to baseline after light allows fast temporal response 5. Transduction mechanism the same 6. Three different types of cones have different visual pigments (opsins). These are sensitive to different parts of the visual spectrum 7. Cones mediate color vision: the relative activation of the cone photoreceptors is responsible for our perception of color 8. No convergence: one-to-one projection from cones to bipolar

Describe how inhibition can be used to compare information from receptors located on different parts of a sensory surface

1. Lateral inhibition enhances selectivity and sharpens contrast between regions that receive strong vs. weak stimulation (Figure 7B). 2. Compares information on the left and right sides of the body (e.g., stimuli presented to the two eyes)

Describe how M and P channels diverge after V2

1. Motion and location information eventually projects to parietal cortex (dorsal pathway) -Neurons are highly sensitive to location and direction of motion, but not to form. -Lesions of this dorsal pathway diminish motion perception and smooth pursuit eye movement 2. Form and color information eventually projects to inferotemporal cortex (ventral pathway) -Neurons are highly selective for color and/or form, but not location -E.g., some neurons selective to a particular face, located anywhere in a large part of visual field -Lesions of the ventral pathway diminish form and color vision.

What are the descending pathways of the auditory system?

1. Olivo-cochlear pathways: modulation of cochlear function through inputs on outer hair cells. 2. Descending projections from auditory cortex to auditory thalamus, midbrain, and brainstem. Basis for modulation of ascending input.

Describe the Second-order visceral afferents in the spinal cord

1. Originate in the dorsal horn and intermediate gray 2. Many are part of the anterolateral system 3. Receive information from dorsal root ganglion axons that carry pressure or stretch information (from heart, bladder, gut walls), chemosensation (oxygen sensitive cells in the carotid body), or nociception (pain) from damaging stretch, ischemia or the presence of irritating chemicals 4. Synapse in the thalamus, nucleus of the solitary tract (NST) and the reticular formation 5. "Referred Pain" from convergence of visceral sensory information with general nociceptive and crude mechanosensory input from superficial sources.

Describe the main types of receptor organs in the vestibular system.

1. Otolith organs (utricle and saccule) serve static balance, linear acceleration. 2. Semicircular canals (three) sense head rotation in different planes.

Describe Visceral Motor Neurotransmission

1. Preganglionic neurons (both sympathetic and parasympathetic) use acetylcholine (Ach) as neurotransmitter. Cholinergic receptors are ligand-gated ion channels (nicotinic) or G-protein coupled receptors (muscarinic). 2. Postganglionic sympathetic neurons primarily release norepinephrine (NE) on their targets. Exception: Ach released on sweat glands; and Adrenal medulla releases both epinephrine and norepinephrine. Adrenergic receptors are all G-protein coupled receptors. 3. Postganglionic parasympathetic neurons release Ach on their targets. 4. Neuropeptides are co-transmitters, especially at muscarinic synapses. 5. The enteric nervous system uses many neuropeptides.

Describe the innervation of the sexual response

1. Preganglionic parasympathetic innervation from sacral cord (S2-S4, through the pelvic nerves). Arousal=Parasympathetic 2. Sympathetic innervation from inferior mesenteric ganglion outflow via T10-L2. Orgasm=Sympathetic 3. Motor input from α-motor neurons in the ventral horn

How is the motor cortex divided?

1. Primary motor cortex 2. Premotor cortex (more rostral) - divisible into lateral and medial parts

Describe cochlear implants

1. Stimulation of auditory nerve fibers by electrodes threaded through cochlea 2. Multiple stimulation sites activate different auditory nerve fibers depending on the frequency of sounds 3. Patients can regain some hearing and speech perception after loss of hair cells. Requires that auditory nerve fibers are intact.

Compare and Contrast Rods and Cones

1. Rods have high sensitivity to light whereas cones have a lower sensitivity to light. 2. Rods operate beter at lower light levels than cones do 3. Rods have a slow temporal response to events, whereas cones have a fast temporal response to events

Describe the fluid filled compartments of the cochlea

1. Scala vestibuli -opening to oval window -contains perilymph 2. Scala media -separated by Reissner 's membrane from scala vestibuli -contains endolymph generated by stria vascularis 3. Scala tympani -separated by basilar membrane from scala media -connected to scala vestibuli at the helicotrema (at apex of the cochlea) -borders the round window -contains perilymph

Describe the role of the muscle spindle in the myotactic reflex

1. Sensory receptor embedded within most muscles. 2. Sensory signal for stretch reflex originates with the muscle spindle. 3. Each spindle has 8-10 intrafusal fibers arranged in parallel with extrafusal muscle fibers. (Extrafusal fibers make up most of the muscle). 4. Two classes of intrafusal fibers: nuclear bag fibers and nuclear chain fibers. 5. Each muscle spindle contains 2-3 nuclear bag fibers and 4-6 nuclear chain fibers. 6. Group 1a afferents - the large diameter sensory axons - coil around the central part of each class of intrafusal fiber. 7. Group II afferents: form endings mainly on nuclear chain fibers. 8. Because Group 1a and Group II afferents are the largest diameter axons in peripheral nerves, they mediate very rapid reflex adjustments when the muscle is stretched.

Describe why shearing in direction of kinocilium causes depolarization in the hair cell

1. Shearing of hair bundle toward kinocilium opens channels located near the tips of the stereocilia. 2. This allows K+ to enter and depolarize hair cell. Causes inward Ca++ flux, resulting in release of excitatory transmitter from hair cell, onto dendrites of primary vestibular neuron. 3. Shearing of hair bundle away from kinocilium hyperpolarizes hair cell because it reduces number of open channels below resting level. As a result transmitter release is reduced below resting level. 4. No effect on channel opening when shearing is at right angles to preferred direction.

Describe the movement of sound pressure waves through the middle and inner ear.

1. Sound waves impinge on tympanic membrane, causing it to vibrate. 2. Ossicles vibrate as a unit. 3. Stapes moves in and out of oval window, in response to sound vibrations. 4. Pressure waves move up scala vestibuli through the perilymph fluid. 5. Pressure waves move across scala media and basilar membrane, and into scala tympani. -High frequency sounds cause maximum vibration nearer to base of cochlea. -Low frequency sounds cause maximum vibration nearer to apex of the cochlea. 6. Increased pressure in perilymph of scala tympani is relieved at round window.

What are the main structures associated with emotional expression?

1. The amygdala and regions connected to it are concerned with several aspects of emotions and motivational states.

Describe the mechanism of the inverse myotactic reflex

1. This sensory fiber (the gamma motor neuron) synapses on an inhibitory interneuron in the spinal cord, which in turn synapses onto motorneurons that supply the muscle innervated by the Golgi tendon organ. This inhibition of a muscle initiated by its own Golgi tendon organs is called autogenic inhibition. 2. The result is that increased tension in a muscle can have an inhibitory effect on further contraction of the muscle. 3. However, this inhibition is balanced against other inputs to the muscle from stretch receptors and descending influences. The inverse myotatic reflex is thought to be an important contributor to the moment-by- moment regulation of the force of muscle contraction.

Describe the pathway for pupillary dilation

1. Under sympathetic control 2. Includes hypothalamic projections to intermediolateral horn of spinal cord-->superior cervical ganglion-->pupillary dilator muscles in iris

Describe the role of the septal nuclei in memory

1. located just ventral to septum pellucidum and just rostral to anterior commissure 2. input from hippocampal formation (via fornix). Other inputs from amygdala, and ventral tegmental area of midbrain (dopaminergic) 3. output to hippocampal formation (via fornix) and to several other limbic system structures: • limbic association cortex • medial dorsal thalamic nucleus • habenula (via stria medullaris) 4. part of forebrain cholinergic system that includes cells in the nearby basal nucleus of Meynert. Cholinergic inputs to hippocampus and neocortex are associated with arousal and synaptic plasticity. Alzheimer's disease involves degeneration of cholinergic neurons in both of these nuclei.

What are the layers of the cerebellar cortex?

1. molecular layer 2. Purkinje cell layer 3. granule cell layer

What is Urbach-Wiethe Disease?

1. rare disorder that results in bilateral calcification of anterior-medial temporal lobes without damage to hippocampus or temporal neocortex 2. cannot recognize emotional content in fearful facial expressions, while able to recognize positive emotional content of facial expressions 3. does not experience fearful emotional reaction to many stimuli that normally evoke fear

Describe Korsakoff's

1. thiamine deficiency due to chronic alcoholism 2. damages diencephalic structures (mammillary bodies and medial dorsal nucleus

Give an overview of the motor pathways

A schematic view of the motor system, including LMNs, UMNs and basal ganglia. We will add the cerebellum later on, for a complete overview of the motor paths. Note the hierarchical arrangement: • LMNs innervate muscles. • UMNs modulate the LMNs • The basal ganglia modulate the UMNs. -Use this schematic to help you review the motor pathways. Note that there is no midline indicated on the schematic! You know that some of these pathways cross the midline, while others do not. A useful exercise is to trace the major pathways, identifying those that cross, and where they cross (e.g., the pyramidal decussation).

1. UMNs generally innervate a. alpha motor neurons b. gamma motor neurons c. interneurons in ventral horn d. extrafusal muscle fibers e. a, b and c

A, B, and C

Describe the descending modulation of nociceptive input.

A. Multiple sources of descending connections to the spinal cord. B. Dorsal horn projection neuron receives multiple sources of sensory input. C. Enkephalin-containing local circuit neuron regulates transmission of nociceptive information based in descending inputs.

What are the main features of receptive fields?

A. Receptive fields differ in size and location. The size of receptive fields on the skin determines "two-point discrimination", the ability to distinguish stimuli that are closely spaced on the skin. B. Two-point discrimination ability varies across the body surface and is directly related to the size of sensory receptive fields across the body surface.

Describe the arrangement of photoreceptors and neural elements in the eye.

A. Rods and cones are distributed differently in the eye. More densely-packed cones in the fovea (coupled with one-to-one relationship with bipolar and ganglion cells) provides highest level of visual acuity. B. Arrangement of photoreceptors and neural elements in and around the fovea. In the center of the fovea (the foveola), the neural elements are shifted to the side so that light has a less obstructed path to the photoreceptors.

Give examples of sensory receptor selectivity

A. Selectivity of photoreceptors to the wavelength of light depends on intrinsic properties of each receptor. B. Selectivity of auditory receptors to the frequency of sound depends the location of the receptor on the basilar member. Each graph shows the lowest sound intensity required to activate the receptors across different sound frequencies.

What are the types of sensory receptor adaptation?

A. Slow adaptation: continued response to stimulus. B. Rapid adaptation: response to ongoing stimulus occurs only at onset of stimulus.

Describe the topographic representation of body surface in primary somatosensory cortex.

A. Somatosensory cortex is located in the postcentral gyrus. B. Representation of different parts of the body is mapped in an orderly way in the postcentral gyrus. C. "Homunculus", or "little man" illustrates the disproportionate representation of the face, especially the lips, and the hands.

11.Which cerebral arteries supply motor cortex? How does the arterial distribution relate to the motor homunculus?

ACA - lower limb; MCA rest of body plus face.

What neurotransmitters are associated with mood disorders and memory

ALL major neurotransmitter systems related to mood disorders and memory (e.g. dopamine, serotonin, norepinephrine, acetyl choline) have extensive connections with structures of the Limbic System.

What is an Acoustic neuroma?

Acoustic neuroma (=vestibular schwannoma) • non-malignant tumor usually begins in vestibular component of VIIIth nerve • progressive hearing loss in affected ear, may include tinnitus • vertigo and nystagmus are variable (WHY?), but vestibular function is progressively reduced. • as tumor grows, facial nerve dysfunction will occur

Give the action of the inferior rectus muscle when the eye is abducted and adducted.

Action with eye abducted: Depression Action with eye adducted: Extorsion

Give the action of the superior rectus muscle when the eye is abducted and adducted.

Action with eye abducted: Elevation Action with eye adducted: Intorsion

Give the action of the inferior oblique muscle when the eye is abducted and adducted.

Action with eye abducted: Extorsion Action with eye adducted: Elevation

Give the action of the Superior oblique muscle when the eye is abducted and adducted.

Action with eye abducted: Intorsion Action with eye adducted: Depression

What are the actions of FSH and LH in the female?

Actions of the anterior pituitary hormones in females regulate function of gonads in more complex ways (review) i. FSH stimulates follicular development and secretion of estrogen and progesterone through its action on granulosa cells. ii. LH stimulates follicular thecal cells to produce, indirectly, estrogen and progesterone iii. LH surge triggers ovulation iv. LH supports corpus luteum after ovulation

Describe the differing responses of cells within visual cortex columns and the cells between columns

Adjacent regions of visual cortex are primarily responsive to the left or right eye (ocular dominance). (A) Eye- specific layers in LGN project to eye-specific columns in the input layer of visual cortex, layer 4. Layer 4 cells project to other layers such that they mix with layer 4 inputs from adjacent other-eye columns. Thus, cells in column centers respond to one eye, and cells between columns respond to both eyes, with a gradation between. (B) Left and right ocular dominance regions are illustrated in a flattened view of visual cortex. (C) Continuous changes in orientation-selective responses are mapped across the surface of visual cortex. (D) Looking at a cross sectional view of visual cortex, eye-specific stripes consist of a series of smaller columns with different orientation selectivity.

Describe the directionality and crossing of the basal ganglia

All the pathways within the basal ganglia and their output to UMNs are uncrossed; i.e., the basal ganglia on the right side modulate motor cortex on the right side. Damage on the right side would then cause deficits in movements primarily on the left side of the body, because of the crossing of the corticospinal tracts.

Generalize the functioning of chemical sensory systems

All three chemosensory systems use receptors that interact with relevant molecules in the environment. For taste and smell, sensory transduction relies primarily on G-protein coupled receptors and second messenger- mediated signaling.

Describe Alternating hemianalgesia

Alternating hemianalgesia - "alternating" means occurring on one side of the face and on the opposite side of the body. This condition results from a lesion in the lateral medulla, which interrupts the anterolateral pathways and the spinal trigeminal nucleus and tract.

What is alternating hemiplegia?

Alternating hemiplegia. Hemiplegia refers to paralysis on one side of the body. Alternating refers to occurrence on different sides at different levels. In the motor system, we can get alternating hemiplegia with lesions in the brainstem. Such a lesion can interrupt the corticospinal pathway above the pyramidal decussation (causing a contralateral paralysis) and interrupt cranial nerve nuclei/fibers that innervate ipsilateral muscles. For example, a lesion in the right medial medulla can damage the right pyramid and right hypoglossal fibers. The paralysis will affect the left side of the body and the right side of the tongue. Lesions at higher levels can affect other motor cranial nerves (e.g., III, VI, VII). We will see examples later in the course.

What gives sound its intensity?

Amplitude of the pressure change determines the intensity of sound. We perceive sound intensity as (mostly) loudness of sounds. We hear sounds over a very broad range of sound pressures, about a factor of a million. We use a logarithmic measure, the decibel, to express this broad range of sound pressures that we perceive.

Describe the receptive field properties of the visual cortex?

Analysis of visual form depends on increasingly complex receptive field properties in visual cortex. (Fig. 17A). Additional classes of cells (not shown) respond to oriented bars that move in specific directions, or that appear in many parts of the receptive field.

What are taste buds? How many do we have, and what makes them up?

Approximately 4000 taste buds in humans are distributed throughout the oral cavity and upper alimentary canal. Each taste bud contains 30 to 100 taste cells (the sensory receptor cells), plus a few basal cells. Taste buds are located on the tongue, palate, pharynx and, in smaller numbers, on the epiglottis and upper third of the esophagus.

What is the olfactory bulb?

As the axons leave the olfactory epithelium, they coalesce to form the olfactory nerve (Cranial nerve I). Each olfactory nerve projects ipsilaterally to the olfactory bulb, which lies on the ventral aspect of the cerebral hemisphere. The most distinctive feature of the olfactory bulb is an array of more or less spherical accumulations of neuropil 100-200 micro m in diameter, called glomeruli, which lie just beneath the surface of the bulb and are the synaptic target of the primary olfactory axons.

What are gaze centers?

As you can gather from our discussion of the various actions of the different eye muscles, a "simple" elevation of the eyes could require different patterns of muscle activity depending on the initial position of the eyes. Perhaps it is not surprising then that the brain has special regions of the reticular formation that coordinate vertical and horizontal eye movements. These so-called gaze centers receive "movement commands" from other parts of the brain and send the appropriate signals to the visual motor nuclei, i.e., cranial nerve nuclei III, IV and VI.

What's an example of a corneal imperfection?

Astigmatism

What is the auditory system?

Auditory system: Designed to sense and analyze sound.

Describe the exit of visual information from the retina

Axons from throughout the retina collect and exit at optic disk a. Optic nerve (cranial nerve II) is a central neural structure b. Optic disk -No photoreceptors (blind spot) -Entry point for central retinal artery c. Each optic nerve contains all axons from one eye

Describe the deficit pattern observed with various located lesions of the corticobulbar tract.

BEFORE CROSS: Contralateral AT CROSS: Bilateral AFTER CROSS: Ipsilateral This crossed pattern applies to projections to hypoglossal nuclei, HOWEVER, most corticobulbar projections terminate bilaterally so a lesion of the origin (e.g. cortex) on one side yields minimal deficits

If you lesioned a crossed portion of the corticospinal tract at different areas, what motor deficit would you expect?

BEFORE CROSS: Contralateral AT CROSS: Bilateral AFTER CROSS: Ipsilateral

Describe varying lesions of the optic radiation

Because the optic radiation separates into two parts on its path to visual cortex, lesions can affect only one of these pathways. a. The more medial part passes underneath the parietal lobe and terminates in the cuneus. Posterior Cerebral Artery supplies this part of optic radiation. A total loss causes Contralateral Homonymous Lower Quadrantanopia b. Meyer's loop passes through the temporal lobe, terminating in the lingual gyrus. Temporal lobe lesions principally affect fibers in Meyers loop, causing Contralateral Homonymous Upper Quadrantanopia. Middle cerebral artery supplies Meyers loop.

Describe the reciprocal connections of the central taste pathway

Between solitary tract nucleus, via the pons, to the hypothalamus and amygdala. These projections influence affective aspects of food.

What are the receptors of the vestibular organs?

Both systems use a mechanoreceptor called the hair cell.

Describe Lesions of the spinocerebellar pathways

Briefly, a lesion affecting the spinocerebellar tracts leads to "clumsy", uncoordinated movements known as ataxia. The deficits are ipsilateral to the lesion. If the lower limbs are involved, the ataxia manifests as an "unsteady gait" with a tendency to fall toward the side of the lesion. We'll discuss these deficits in more detail when we consider the cerebellum.

What areas constitute the premotor cortex?

Brodmann's areas: area 6, as well as specialized contributions from 8, 44, 45, 23, 24

How do central sensory neurons convey varying sensory information despite the fact that they can only fire all or nothing potentials

By the use of neural codes: 1. The rate and temporal pattern of actions potentials discharged by individual neurons 2. The relative activity among a population of neurons, each with different stimulus tuning 3. Spatial pattern of activity across an array of neurons

What is the trigeminal pathway?

CN V -All three divisions - ophthalmic, maxillary and mandibular - carry all three types of somatosensory information. All trigeminal axons enter the brainstem at mid-pons.

How is two point discrimination, vibration, and fine touch tested?

Calipers are used to test two-point discrimination and a tuning fork can be used for vibration. Fine touch can be tested with a cotton swab.

What are caloric tests?

Caloric tests examine function of semicircular canals on one side. - Warm or cool water placed in one external auditory meatus evokes characteristic nystagmus - Direction: Cool Opposite Warm Same (COWS)

Describe the clinical significance of eye movements

Central or peripheral (cranial nerve) problems can cause deficits in eye movements. Careful testing of the eye movements is simple and can be useful for assessing the integrity of several cranial nerves and important sections of the brainstem and descending pathways.

Describe the internal layers of the cerebellum

Cerebellar cortex White matter Deep nuclei

Describe the general organization of the cerebellar cortex

Cerebellar cortex includes 5 cell types arranged in 3 layers (see figure).

Contrast signs of cerebellar damage with those of UMN, LMN or basal ganglia damage.

Cerebellar signs: intention tremor, ataxia, hypotonia (also dysmetria, dysarthria, dysdiadochokinesia, decomposition of movement, pendular reflexes, nystagmus). UMN signs: spastic paralysis, hypertonia, hyperreflexia, and maybe Babinski sign, clonus, decreased superficial reflexes. LMN signs: flaccid paralysis, areflexia or hyporeflexia, hypotonia, fasciculations, atrophy Basal ganglia: hypokinesia or hyperkinesia; may have resting tremor, rigidity

What are the roles of the cerebellum and basal ganglia?

Cerebellum and basal ganglia (not shown) make contributions that we'll discuss in coming lectures).

63 year old man, alcoholic "in remission"; staggers when walking; other movements normal.

Cerebellum, anterior lobe atrophy

What is the role of the cerebral cortex in movement?

Cerebral cortex initiates, plans and coordinates voluntary movement

Describe symptoms of cerebrocerebellar lesions

Cerebrocerebellum - lateral hemisphere/dentate: delayed initiation; decomposition of movements

Describe the cerebrocerebellum

Cerebrocerebellum-lateral hemispheres and dentate nucleus. Also called "pontocerebellum", since the inputs from cerebral cortex actually arrive via the pontine nuclei. This area is especially important in motor planning.

Describe climbing fibers

Climbing fibers originate in the inferior olivary complex in the medulla. These olivocerebellar fibers leave the inferior olive, decussate and enter the cerebellum via the inferior cerebellar peduncle.

Describe the effect of chronic alcoholism on the cerebellum

Chronic alcoholism can lead to degeneration in the vermis and paravermal regions of the anterior lobe. These patients suffer from tremor and ataxia of the trunk and legs; the head and arms are spared. The distribution of deficits can be understood by looking at the representations (there are several) of the body in the spinocerebellum...

What is the role of the OVLT and SFO circumventricular organs?

Circumventricular organs associated with osmotic and fluid balance. These organs lack a blood-brain barrier. The vascular organ of the lamina terminalis (OVLT) contains osmoreceptors. The Subfornical organ (SFO) is where angiotensin II levels are detected by the hypothalamus.

Proprioceptive information from the right leg is relayed by which: Clarke's nucleus or external cuneate nucleus?

Clarke's nu. [external cuneate carries info from upper body]

What fact must be kept in mind when testing the cranial nerves?

Clearly, the action of a particular muscle depends on the position of the eye. This fact must be kept in mind when testing the integrity of cranial nerves III, IV and VI and the pathways that innervate them. The figure below provides a guide for testing each muscle (relatively) independently.

What is the clinical significance of the somatosensory system?

Clinical evaluation of patients routinely requires an assessment of the sensory systems to infer the nature and location of potential neurological problems. Knowledge of how the various sensory modalities are transduced, relayed, represented and processed to generate appropriate behavioral responses is essential to diagnosing and treating a wide variety of diseases.

Give a clinical example of the stretch reflex

Clinical testing: this is the reflex activated by tapping patellar (or other) tendon

What is clonus?

Clonus is a repetitive flexion-extension of a joint in response to a single flexion or extension. The presence of clonus indicates UMN damage.

What is Choreoathetosis?

Combination of chorea and athetosis.

How would you distinguish a peripheral nerve lesion from a dorsal root lesion?

Compare distribution of the deficit: a dorsal root lesion will have a dermatomal pattern, whereas peripheral nerve lesion will match the distribution of that nerve.

What is Ageusia?

Complete loss of taste (rare).

Give a general, system-wide overview of the auditory system

Complex arrangement of fiber bundles and nuclei. 1. Crossed and uncrossed components 2. Why the complexity? Brainstem nuclei create different auditory response properties. Higher centers integrate these different responses to fully represent auditory stimuli.

How is sexual function controlled?

Complex coordination of sympathetic, parasympathetic, and motor responses, governed by complex cognitive, emotional and contextual clues processed in the limbic forebrain.

What is decomposition of movement?

Complex movements often involve simultaneous movements around multiple joints. Cerebellar damage eliminates this possibility; the patient will move one joint at a time. Common with lateral hemisphere lesions. lack of check - inability to stop a limb as resistance is suddenly removed.

Describe the dynamics of continence

Continence is maintained by sympathetic tone / inhibition of parasympathetic activity 1. Sympathetic activation [T10 to L2 via inferior mesenteric ganglion] imposes: i. Inhibition of the detrusor muscle ii. Excitation (constriction) of the internal sphincter 2. Combined actions permit bladder filling and holding

Total destruction of the medial lemniscus on the right side of the medulla would lead to a deficit where?

Contralateral body, all dermatome levels

Describe how the hypothalamus controls osmotic balance in the body

Control of plasma concentration of water 1. Osmoreceptor neurons are located in the vascular organ of the lamina terminalis (OVLT) or adjacent areas of the anterior hypothalamus. The OVLT is one of several brain regions lacking a blood-brain barrier (Fig. 6). 2. OVLT neurons send a direct, excitatory projection to the hypothalamic cells responsible for secretion of antidiuretic hormone (ADH) into the posterior pituitary. These cells are magnocellular neurons of the supraoptic nucleus (SON) and paraventricular nucleus (PVN) of the hypothalamus (Fig. 7B). 3. Secreted through the posterior pituitary, ADH acts on cells of the kidney to reabsorb water from the urine. 4. OVLT neurons also project elsewhere to activate thirst. The dorsomedial hypothalamus (DMH) appears to be a relay in pathways to the thalamus and cerebral cortex (anterior cingulate and medial prefrontal) that are related to conscious perception of thirst.

Compare the convergence in rods and cones

Convergence in rods, no convergence in cones. Thus: 1. Rods have poorer acuity but greater sensitivity to light 2. Cones have greater acuity but need more light to respond

What is the task of the conductive system of the ear?

Conveys mechanical vibration in sound waves to inner ear.

What are the primary recipients of ascending vestibular information?

Cortical areas 3A and 5 are primary recipients of ascending vestibular information.

What controls the facial muscles?

Corticobulbar tracts, facial nuclei and facial nerves

What is the current view of satiety control?

Current view based on understanding that leptin signaling is a major element of long-term control of feeding 1. Leptin: a hormone produced and released by adipose cells at levels directly related to amount of stored lipid -increased leptin decreases feeding and increases energy consumption -decreased leptin increases feeding behaviors and decreases energy consumption -Leptin-deficient mice become obese, and leptin replacement restores more normal weight 2. Insulin, secreted by pancreas in response to elevated fat levels plays a similar role in the long-term regulation of feeding

What does damage to upper motor neurons cause?

Damage to UMNs produces a characteristic set of deficits, including paralysis (or weakness) and spasticity (hypertonia plus hyperreflexia). The deficits are distinct from those associated with damage to other parts of the motor system.

12.How would you distinguish damage to the face region of motor cortex from damage to the facial nerve?

Damage to motor cortex = corticobulbar lesion. Deficit would affect contralateral lower face. Damage to facial nerve affects whole side of face, ipsilateral to lesion.

Describe the Intervening Synapses and Decussation of the Dorsal Column System

Decussation in Caudal Medulla (Internal Arcuate Fibers)

Describe the distribution of analgesia following damage to ventral white commissure of the spinal cord at the level of C7

Deficit forms a ring around the body at C7. The important issue is that the deficit is bilateral, and that most of the ascending information from lower dermatomes is unaffected

Describe deficitis in the S cone

Deficits in S cones with blue photo pigments (tritanopia) are rare. Since they occur on an autosome (chromosome 7), they affect men and women equally.

Generally describe deficits of color vision

Deficits in color vision:multiple disorders resulting from the genetic lack or malformation of cone pigments. These cause perceptual disorders collectively called "color-blindness". Characterized by inability to distinguish certain colors.

What is deuteranopia?

Deuteranopia involves photopigments in the M-cone (green).

What is the clinical significance of the pupillary light reflex?

Diagnostic tool for testing brainstem and cranial nerve function.

Describe the direct projections of the motor cortex to the lower motor neurons

Direct: corticospinal tracts to LMNs are especially important for speed and agility; critical for fine, fractionated movements of the fingers.

Describe the direct vs. indirect pathways through the basal ganglia. What effect does each pathway have on movement (increase or decrease)?

Direct: striatum - GPi/SNr. Activity in the direct pathway increases movement. Indirect: striatum - GPe - subthalamic nucleus - GPi/SNr. Activity in indirect pathway decreases movement.

What are come clinical issues of fluid balance?

Disorders can separately or jointly affect responses to changing fluid volume or plasma water concentration a. Disorders centered in OVLT or SFO affect both water retention and thirst mechanisms. b. Disorders in magnocellular neurons affect water retention but not thirst. c. Diabetes insipidus: failure to concentrate urine

Describe the effects of a lesion of the anterior white commissure in the spinal cord on the ALS.

Disrupts pain and temperature sensation bilaterally- that is, an entire ring of the body will be missing these sensations. Hemisection of the cord will cause contralateral loss of pain and temperature, and ipsilateral loss of discriminative touch.

What is disuse atrophy?

Disuse atrophy. Atrophy is a sign we have associated with lower motor neuron damage. Paralysis resulting from UMN damage can also lead to atrophy because the muscle is not used. In both cases, the atrophy takes time to develop; often, atrophy following LMN damage is more severe. Obviously, you would not use atrophy alone as a diagnostic tool.

Describe the First CNS Synapse In the Ascending Pathway of the Dorsal Column System

Dorsal Column Nuclei In the Medulla

Generalize the control of eye movement

Each eye is controlled by 6 external eye muscles, three pairs operating in roughly antagonistic fashion

What information is carried by the optic tract? What visual information bypasses it?

Each optic tract carries a complete representation of the contralateral one-half of the visual field. Thus, fibers from the nasal hemiretina of each eye cross to the opposite side at the optic chiasm, whereas fibers from the temporal hemiretina do not cross. In the illustration, light from the right half of the binocular zone falls on the left temporal hemiretina and right nasal hemiretina.

Describe the general effect of light hitting a photoreceptor

Effects of light hitting photoreceptor: • Decreases cGMP concentration • Closes Na+/Ca2+ channels (gated by cGMP) • Hyperpolarizes cell • Less neurotransmitter released onto next cell in pathway (bipolar)

Describe the effects of temporal lobe lesions

Effects of temporal lobe lesions in humans support roles in 1. establishing emotional significance of sensory stimuli (e.g., fear, anger, rage, sexual behavior). 2. generate appropriate responses to emotion-laden sensory stimuli. 3. affective state

Compare and contrast the autonomic and somatic nervous system in terms of efferent innervation, effector, function, spinal cord region, and tonotopic organization.

Efferent Innervation: Somatic-one neuron; Autonomic-two neurons (post/pre ganglionic) Effector: Somatic-muscles; Autonomic-visceral organs/structures (heart, lungs, glands) Function: Somatic-voluntary; Autonomic-involuntary Spinal Cord Region: Somatic-entire; Autonomic-localized Tonotopic Organization: Somatic-Rigid/logical (dermatome/myotome); Autonomic-diffuse

What is the function of ghrelin?

Feeding behavior is promoted by ghrelin -Hormone released by stomach near beginning of expected meal -Strongly stimulates appetite and food consumption by activating NPY- and AgRP-containing neurons in arcuate nucleus

7. Briefly discuss feed forward and feed back mechanisms for maintaining postural stability. Describe major motor pathways involved.

Feedforward is for anticipated instability; relies especially on cortico-reticulospinal tracts. Feedback is for unanticipated instability, detected by the vestibular system, and corrected via vestibulospinal tracts.

Describe the functional arrangement of the semicircular canals

Functional arrangement: Designed to sense angular acceleration that accompanies rotation of the head -Angular acceleration causes endolymphatic flow, which in turn causes movement of cupula.

What is the general role of the ganglion cells of the retina?

First cells to have action potentials in the retina

What is fixation?

Fixation (on a stationary object). Okay, so this isn't exactly a movement, but it is accomplished by an active neural system; i.e., it is not simply the lack of eye movements. It is coordinated largely by the superior colliculus and associated pathways.

What happens with lesions to the front eye field?

Following damage to the FEF, the patient is unable to make voluntary saccades to the contralateral side, although the eyes may still track a moving object as it enters the contralateral visual field (i.e., smooth pursuit is less dependent on the FEF). In addition, at rest the eyes are deviated toward the side of a FEF lesion. The deficits are often minor and temporary if the lesion is small (i.e., involves only one of the cortical fields but not others). A very large lesion can result in permanent deficits. 1. Can't move eyes away from side of lesion 2. At rest, eyes may be deviated toward side of lesion

What receptors are used to amino acid (and umami) tastants?

For amino acid (umami) tastants, transduction occurs via a PLC- mediated IP3-dependent mechanism that leads to Ca++ entry.

Describe the vestibular nuclei

Four nuclei on each side receive input from vestibular sense organs and other brain regions. They differ somewhat in inputs, outputs, and functional roles.

What does the fovea contain?

Fovea contains cones, almost exclusively (Fig. 10A) • cones packed more densely in fovea than in periphery • low convergence from cones to bipolar cells to ganglion cells • results in higher spatial resolution for maximal acuity

What is the fovea?

Fovea is in central part of macula: depression/pit caused by lateral displacement of bipolar and ganglion cells for minimum optical distortion (Fig 10B) • avascular zone allows low optical distortion (Fig. 10B)

Describe the chemical senses from an evolutionary perspective.

From an evolutionary perspective, the chemical senses are thought of as the oldest or most primitive sensory systems.

Describe the anatomy and function of the premotor cortex

Frontal lobe, anterior to primary motor cortex Planning and selecting movements.

How do gamma motor neurons regulate the tension created by muscle activation?

Gamma motor neurons regulate the tension or force generated by muscular contraction through the inverse myotatic reflex.

Describe the general descending pathways of the hypothalamus to the brainstem and spinal cord

General descending projections to brainstem and spinal cord 1. to brainstem autonomic centers associated with autonomic output (via dlf, hypothalamomedullary tract, and others) -cardiovascular regulation -digestion, including salivation 2. to brainstem centers associated with shivering 3. to sympathetic and parasympathetic neurons in spinal cord via hypothalamospinal tract

What does GnRH act on?

GnRH acts on gonadotroph cells of the anterior pituitary, including cells that secrete follicle stimulating hormone (FSH) and luteinizing hormone (LH) (Fig. 8).

What proportion of retinal ganglion cells are off-center? What about on-surround?

Half of all ganglion cells are OFF-center/ON-surround (the opposite of the ganglion cell described above). This means they are inhibited when light shines on their center cone, but are excited when light shines on cones surrounding the center cone. This occurs because the center cone is connected to a different bipolar cell, and that uses a different type of glutamate receptor that has effects opposite to the one in bipolar cells of the ON-center pathway. This OFF-center bipolar cell in turn connects to an OFF-center/ON-surround ganglion cell.

Describe the Edinger-Westphal nucleus

Location: Midbrain Cranial Nerve: CN III Postganglionic Synapse: Ciliary ganglion

Describe the Nucleus Ambiguus

Location: Nucleus ambiguus Cranial Nerve: Medulla Postganglionic Synapse: Heart

Explain how horizontal movements are accomplished in the eye.

Horizontal movements are accomplished by the lateral and medial rectus muscles -- the lateral rectus for abduction and the medial rectus for adduction.

What can contribute to neuropathic pain?

Immune responses can contribute to neuropathic pain.

Where is the vertical gaze center?

In rostral midbrain, near CN III nucleus. What is it called? rostral interstitial nucleus of the medial longitudinal fasciculus (riMLF).

Describe the indirect projections of the motor cortex to the lower motor neurons

Indirect: cortico-reticulospinal pathway for feedforward controls. Focused on axial muscles and proximal limb muscles. Movements limited to this system show reduced speed and agility, but many tasks can be accomplished with "just" the indirect pathway (as would occur, e.g., if the pyramids were lesioned in the medulla).

What are the components of the inferior cerebellar peduncle?

Inferior cerebellar peduncle (ICP) has two components: a. restiform body-the bulk of the ICP; contains all the ICP fibers except the connections between the cerebellum and the vestibular nerve and nuclei b. juxtarestiform body ("next to the restiform body")-contains the fibers traveling between the cerebellum and vestibular system. -INPUT AND OUTPUT

Describe lateral inhibition

Inhibition can shape the receptive field of sensory neurons. Some second order neurons show lateral inhibition, in which inhibitory inputs from an adjacent set of receptor neurons inhibit the second order neuron when the stimulus includes certain locations. The effect of the inhibition is to place a sharp border on the second order neuron's excitatory receptive field, thus enhancing contrast between the regions of the sensory surface that evoke spikes and nearby regions

Describe the somatic nervous system

Innervates skeletal muscle directly (one neuron) and is under voluntary control. The length of the spinal cord has neurons that innervate skeletal muscle. There is a rigid/logical organization of the somatic nervous system that is reflected in the dermatome sensory regions and myotomes.

What is Dyskinesia?

Involuntary movements.

What is protanopia?

Involves photopigments in the L-cone (red).

The VA/VL nuclei project to the [ipsilateral or contralateral] cerebral cortex.

Ipsilateral

Describe the concept of antagonistic muscles

Joint/body movements are mediated by muscles having antagonistic or opposing mechanical actions -flexors/extensors: e.g., hamstring vs. quadriceps -lateral vs. medial rectus muscles controlling eye movement

What is chorea?

Jerky, random movements (sometimes described as dance-like) of the limbs or face.

Describe the Superior salivatory nucleus

Location: Pons Cranial Nerve: CN VII Postganglionic Synapse: Pterygopalatine and submandibular ganglion

What is the role of lower motor neurons?

LMNs (and associated local circuits) coordinate many fundamental functions, including reflexes as well as more complex movements such as locomotion.

What is Akinesia?

Lack of movement due to difficulty initiating movements.

How does one test for analgesia?

Lack of pain is called analgesia. A sharp probe is used for testing. An important clinical tool for evaluating the nervous system is careful testing of both anterolateral and dorsal column/medial lemniscus systems

What is Ballism?

Large-amplitude, violent movements of an entire limb.

What muscles does CN VI innervate?

Lateral Rectus

Where is the lateral motor system located?

Lateral system in lateral funiculus

What structures play a role in learning and memory?

Learning and Memory: The hippocampus plays a critical role in functions related to memory. The hippocampus connects indirectly to widespread areas of cerebral cortex. Lesions of the hippocampus or functionally related areas affect consolidation of short-term memory into long-term memory.

2. Contrast the medial and lateral systems of descending motor control. Which is "contralateral" and which is "bilateral"?

Medial: "bilateral", posture and balance, paths descend and terminate medially, affect axial and proximal limb muscles Lateral: "contralateral", skilled movements, paths descend and terminate laterally, affect distal limb muscles.

Describe the orientation selectivity of the visual cortex

Many Area 17 neurons are responsive to spatially oriented contours of light or dark: e.g., bars or lines, rather than to spots of light. • Area 17 neurons differ in their preferred orientation of light or dark shapes (Fig. 17B,C). • This arises because neurons with concentric receptive fields located in slightly different locations converge onto a visual cortical neuron. (Fig. 17D). • Orientation selectivity is organized in visual cortex (Fig. 18B).

Describe the effects of medial and lateral lesions in the medulla on the ALS.

Medial lesions disrupt discriminative touch for the contralateral body whereas lateral lesions disrupt pain and temperature sensation for the contralateral body (review the locations of the pathways!).

Describe the differing adaptation rates of mechanorecptors

Meissner and Pacinian are rapidly adapting; Ruffini and Merkel are slowly adapting somatosensory afferents.

What regions of the olfactory cortex do mitral cells project to?

Mitral cells project to five different regions of the olfactory cortex: (1) the anterior olfactory nucleus which innervates the contralateral olfactory bulb (2) the olfactory tubercle (3) the piriform cortex (4) parts of the amygdala (5) parts of the entorhinal cortex

Describe the speed of the stretch reflex

Monosynaptic stretch reflex is rapid a. Sensory afferents and motorneurons have high conduction velocities b. Single CNS synapse (<1 ms synaptic delay) c. Response in muscle can be as short as 20 ms after sensory stimulus

Compare the saturation between rods and cones

More saturation in rods than cones. As brightness of light flash increases, cone response returns to baseline quickly, while rod response is extended in time. Thus: 1. Cones can respond in bright light, while rods saturate (all channels closed) and can no longer respond in bright light. 2. Cones have fast responses and can signal closely-spaced events, while rods can't distinguish such events.

Describe mossy fibers

Mossy fibers constitute all the remaining inputs, including those from spinal cord (the spinocerebellar pathways), the vestibular nerve and nuclei, and the pontine nuclei (more about these a little later).

3. Contrast motor cortex versus brainstem for functional contributions to movement.

Motor cortex: for planning, initiating and directing voluntary, skilled movements Brainstem: Basic movements, postural control, locomotion, orientation to sensory stimuli, emotional expression

What are motor functions handled by?

Motor functions are handled by projections from motor cortex

What is represented in the primary motor cortex?

Movements, not individual muscles or body parts, are represented in primary motor cortex. Nonetheless, the location of a lesion directly correlates with the distribution of the deficit.

What mechanisms contribute to the sensation of pain?

Multiple mechanisms contribute to the sensation of pain. Pain is subject to various types of modulation, which can take place at the level of pain receptors in the periphery, in signal transduction mechanisms, and in anatomical pathways in the peripheral and central nervous systems.

What is muscle spindle activity moderated by?

Muscle spindle activity is regulated by G (gamma) motor neurons

How is gonadal function controlled by the hypothalamus?

Neuroendocrine control of gonadal function is based on hypothalamic neurons that produce gonadotropin releasing hormone (GnRH) 1. GnRH neurons are located in many parts of the human hypothalamus. Many terminate on the capillary system in the median eminence of the hypothalamus that carries GnRH to the anterior pituitary (Fig. 7A). -GnRH neurons arise in the olfactory placode and migrate into the hypothalamus during embryonic development -Kallmann Syndrome: includes failure of GnRH neurons to migrate, leading to failure to develop central control of gonads and failure to develop gonadal function.

What are big examples of chronic pain?

Neuroimmune and neuropathic pain are examples of chronic pain syndromes.

Describe the Afferent Neuron of the Dorsal Column System

Neuron of the Dorsal Root Ganglion

Describe the receptor cell of the Dorsal Column System

Neuron of the Dorsal Root Ganglion

What kind of neurons produce oxytocin?

Neurons of the paraventricular and supraoptic nuclei (PVN and SON) synthesize and release oxytocin into the posterior pituitary

What is the ventral striatum?

Nucleus Accumbens

What does NST mean?

Nucleus of Solitary Tract

What are papillae? What are their types?

On the tongue, most taste occur in small elevations called papillae. There are three types of papillae: fungiform (which contain about 25% of the total number of taste buds), circumvallate (which contain 50% of the taste buds), and foliate (which contain 25%). Fungiform papillae are found only on the anterior two-thirds of the tongue; the highest density is at the tip. They have a mushroom-like structure (hence their name) and typically have about 3 taste buds at their apical surface. There are 9 circumvallate papillae arranged in a chevron at the rear of the tongue. Each consists of a circular trench containing about 250 taste buds along the trench walls. Two foliate papillae are present on the posterolateral tongue, each having about 20 parallel ridges with about 600 taste buds in their walls.

What do on-center cells respond best to?

On-center cells respond best when the entire central part of the receptive field is stimulated ("Illumination of Entire Center"). These cells also respond well, but less vigorously, when only a portion of the central field is stimulated by a spot of light ("Spot of Light in Center"). Illumination of the surround with a spot of light ("Spot of Light in Surround") or ring of light ("Illumination of Entire Surround") reduces or suppresses the cell firing, which resumes more vigorously for a short period after the light is turned off. Diffuse illumination of the entire receptive field elicits a relatively weak discharge because the center and surround oppose each other's effects.

Describe the central dogma of odorant receptor and chemical interaction

One receptor type/one gene/one protein/one odorant type!

What is an important anatomical note about the paravertebral ganglia?

Only the T1 to L2 segments of the spinal cord have white (preganglionic) and gray (postganglionic) communicating rami. For the segments above or below this range, the preganglionic axons run up to cervical chain ganglia or down to sacral chain ganglia before exiting via a gray communicating ramus and joining the spinal nerves.

What taste region is innervated by Trigeminal?

Oral cavity

Describe the organization of projections from the retina to the later geniculate nucleus

Organization of projections from the retina to the lateral geniculate nucleus. Axons from the right hemiretina of each eye project to different layers of the right lateral geniculate nucleus to create complete representation of left visual hemifield. Layers 1 and 2 form magnocellular pathway (M pathway), while layers 3-6 form parvocellular pathway (P pathway). Layers 1, 4, and 6 receive input from the contralateral eye that crosses in the optic chiasm.

Describe the origin and termination of the corticobulbar tract

Origin: cerebral cortex Termination: brainstem motor nuclei

Describe the origin and termination of the corticospinal tract

Origin: cerebral cortex Termination: spinal cord ventral horn

Generally describe outputs from the deep cerebellar nuclei

Originate from the deep cerebellar nuclei (the fine print: there is a small portion of cerebellar output that originates from cells in the cerebellar cortex; we'll discuss these later). Axons project out of cerebellum through superior and inferior cerebellar peduncles.

Describe the anatomy of the middle ear

Ossicular chain, middle ear muscles, round window and oval window. Function is auditory in nature.

Describe the outputs of the vestibular nuclei

Outputs. In additional to reciprocal connections with vestibulo-cerebellum and contralateral vestibular nuclei, outputs include: 1. spinal cord 2. oculomotor nuclei 3. reticular formation 4. nucleus of solitary tract

Describe the role of oxytocin in lactation

Oxytocin has multiple functions in lactation -oxytocin released by the pituitary causes contraction of myoepithelial cells of the breast to eject milk in the letdown reflex. -oxytocin promotes release of prolactin, increasing milk production

Describe how the release of oxytocin during lactation differs from its release during other times

Oxytocin release during lactation is controlled differently than at other times -For females that are not pregnant or nursing, glial processes isolate the oxytocin-containing PVN and SON neurons from each other -Just before and during lactation, hormonal conditions cause retraction of glial processes and formation of gap junctions among these cells. The cells become electrically coupled. When oxytocin cells are stimulated, the electrical coupling results in a large, coordinated release of oxytocin that is required for ejection of milk. -Oxytocin release is activated by sensory stimulation of the mother's nipple by the nursing infant.

Describe P-Type Ganglion Cells

P-type (parvicellular = small) cells: • smaller receptive fields • respond well to more sustained stimuli • color-sensitive • mediate color/form vision

What types of fibers do pain and temperature information travel in?

Pain and temperature information from free nerve endings is carried to the spinal cord by Aδ fibers (small, myelinated) or C fibers (unmyelinated). Both fiber types are thought to use substance P as a transmitter. Both types of axon can branch when they enter the cord. The branches can ascend or descend a few segments in Lissauer's tract (a.k.a. dorsolateral fasciculus) before terminating on second order neurons. The axons then terminate in laminae I and V (A-delta) and laminae I and II (C fibers) in the dorsal horn.

What is pain?

Pain is a sensation. Sensations that are intense, unpleasant, or distressing are described as being painful.

What is pain NOT?

Pain is not an excessive stimulation of the same receptors that generate the other somatic sensations. Pain perception uses specific receptors and pathways.

Is pain homogenous?

Pain is not homogeneous, but falls into three general categories: (a) Physiological pain (nociception) (b) Inflammatory pain (inflammation) (c) Neuropathic pain (pathological)

What is phantom pain?

Pain that occurs in a phantom limb (more in a subsequent lecture).

Generally describe sensory organ pathologies

Pathologies of peripheral receptor cells and their support structures are a major cause of failure of some sensory systems to function properly. It is common to distinguish two general types of peripheral pathologies: 1. Disorders resulting from improper function of receptor cell or sensory neurons 2. Disorders affecting support structures

Describe the effects of lesions in the pons and above on the ALS

Pathways conveying all modalities from the contralateral body travel together to the thalamus (VPL). Thus, small lesions cause hemianesthesia - loss of pain, temperature and discriminative touch for the contralateral half of the body (but see notes below about loss of pain).

How do peripheral nociceptors terminate?

Peripheral nociceptive axons terminate in unspecialized "free endings", in contrast to other somatic sensory receptors (e.g. Pacinian corpuscles etc).

Are photoreceptors distributed equally in the retina?

Photoreceptors and neural connections are distributed unequally in the retina

Describe the location of photoreceptors

Photoreceptors are located in the back of the retina. Light must pass through other retinal elements to reach photoreceptors

Describe the anatomy of the external ear

Pinna and associated structures, external auditory meatus, tympanic membrane. Function is auditory in nature.

Describe the anatomy and projections of the parasympathetic nervous system

Preganglionic neurons → Ganglionic neurons → Target Organs -Parasympathetic preganglionic neurons reside in five brainstem nuclei and sacral levels S2 to S4 in the spinal cord. In contrast to the sympathetic NS, these neurons have long efferents that synapse in ganglia that are close to their targets. The preganglionic fibers from the brainstem nuclei travel along with the cranial nerves.

Describe the anatomy and projections of the sympathetic nervous system

Preganglionic neurons → Ganglionic neurons → Target Organs -Sympathetic preganglionic neurons reside in the intermediolateral column of the thoracolumbar spinal cord. These neurons receive descending input from hypothalamus and other structures. -They send myelinated axons out via the ventral roots to synapse in one of three different kinds of sympathetic ganglia: 1) paravertebral (chain) ganglia, 2) prevertebral (collateral) ganglia 3) the adrenal medulla.

What is Presbyopia?

Presbyopia. Loss of lens's focusing ability with age. This is due to thickening of lens and loss of its ability to round. The ciliary muscles that act on the lens (via Zonule of Zinn fibers) are fine.

What happens to eye movements with cerebellar movements?

Problems mainly with smooth pursuit; possible nystagmus

Describe the role of efferent innervation on the regulation of frequency tuning by the cochlea

Projections from brainstem auditory nuclei contact inner and outer hair cells via olivo-cochlear bundle. This alters sensitivity of hair cells to sound.

What does a "Pyrimidal" motor pathway refer to?

Pyramidal - named because axons travel in the pyramids. Essentially the corticospinal (and corticobulbar) tracts.

The left side of the cerebellum modulates activity primarily in the (RIGHT or LEFT) motor cortex? Describe the relevant pathway.

RIGHT -Projections from cerebellum go to VL of thalamus by way of superior cerebellar peduncle. The SCP crosses in the midbrain (decussation of SCP), so left cerebellum projects to right VL, which projects in turn to right motor cortex.

Describe the Neural control in optokinetic nystagmus

Recall that nystagmus is a cyclical event consisting of a slow movement in one direction followed by a fast movement in the opposite direction. While it is somewhat of an oversimplification, we will consider nystagmus to be controlled by a combination of the smooth pursuit circuits and saccade circuits.

What subgroup does the basal ganglia belong to?

Recall that the term "extrapyramidal system" generally includes the basal ganglia.

What is the "Sensory Surface"?

Receptors are often arrayed on a surface, and their sensitivity often varies with position along that surface. (e.g. body surface, retina, basilar membrane).

What is hyposmia?

Reduced sense of smell

What is the clinical significance of reflexes?

Reflexes provide a means to assess the integrity of the nervous system. Commonly tested reflexes in clinical situations. a. Stretch (myotatic) reflex in skeletal muscle (e.g., patellar tap) b. Plantar reflex c. Pupillary light reflex, vestibular reflexes, postural reflexes

How does one assess reflexes?

Reflexes: compare sides and limbs; look for asymmetries

What is Argyll-Robertson pupil?

Result of syphilis infection of CNS i. Small pupils with weak pupillary light reflex. However, visual acuity is good and pupil can constrict in Accommodation to Near Vision (see below). ii. Lesion appears to be in pretectum or pathway to Edinger-Westphal nucleus, rather than in sensory pathway or in Edinger-Westphal nucleus.

What structures play a role in reward?

Reward: The nucleus accumbens plays a significant role in motivation and rewarding behaviors. It shows increased activity when positive emotions are expressed. Nucleus accumbens and dopaminergic inputs have been implicated in addictive states.

1. Sensory information from the right side of the body reaches which side of the cerebellum?

Right

Proprioceptive information from the right leg reaches which side of the cerebellum?

Right

Which cerebral artery supplies the right frontal eye field?

Right MCA

Describe the circuitry of rods

Rod > Bipolar cell >Ganglion cell i. Extensive convergence ii. Increases sensitivity but reduces visual acuity c. In peripheral retina, bipolar cells from rods and cones synapse on the same ganglion cell. d. Horizontal cells and some amacrine cells mediate lateral interactions important in receptive field properties

What tests are used to diagnose vestibular issues?

Rotatory and caloric tests diagnose vestibular disorders by activating vestibular reflexes.

What are saccades?

Saccades are quick darting movements that change the point of fixation. Saccades can be generated voluntarily, or they can occur as reflexes. Saccades are the primary means for voluntary movement of our eyes; the remaining types of eye movements are largely, or exclusively, reflexive (as for many other reflexes, one can voluntarily override some reflex eye movements).

Describe the dorsal column pain pathway

Second-order cells near central canal send visceral pain -carrying afferents through dorsal columns to synapse in dorsal column nuclei. Third-order cells in DC nuclei project to thalamus.

Describe the bilateral pairing of semicircular canals

Semicircular canals on the two sides of the head form functional pairs (see Figs. 9). When movement causes hair cell in one canal to be depolarized, hair cells in the other canal in the pair are hyperpolarized. The functional pairs are: -left and right horizontal canals -left anterior and right posterior canals -right anterior and left posterior canals

Describe the pathway for the corneal (blink) reflex.

Sensory neuron with free nerve ending in cornea, cell body in trigeminal ganglion, and central axon traveling in spinal V tract to terminate in ipsilateral spinal V nucleus, pars caudalis. Cells in spinal V nucleus project to reticular formation, which projects bilaterally to facial nuclei, which innervate orbicularis oculi muscles for each eye.

What are the neural control centers of saccades?

Several structures project to the gaze centers and are important for generating saccades. Control centers for saccades 1. superior colliculus 2. frontal eye field (FEF)- in Brodmann's area 8, a region of the frontal lobe just rostral to premotor cortex. 3. supplemental eye field-rostral end of supplemental motor area 4. parietal eye field-in inferior parietal lobule; especially important for visual attention

Why is there some ambiguity with distinguishing scents responses at the receptor level?

Since a single receptor sub- type can respond to more than one odor (although not many more) there is some ambiguity in telling odors apart at the level of the receptor cells.

What can be a secondary effect of issues with olfaction?

Since olfaction makes an important contribution to taste, olfactory changes can also contribute to food intake and nutritional issues.

What receptor mechanism allows us to recognize sour tastes?

Sour tastants are transduced by a proton-permeant, non-selective cation channel that is a member of the TRP channel family. In both cases, positive current via the cation channel leads to depolarization of the cell.

What is the basic idea of a myotactic reflex?

Stretch reflex. Stretch or lengthening of a skeletal muscle results in contraction that shortens that muscle.

2. Which area receives most of the input to the basal ganglia?

Striatum (caudate and putamen)

What is another term for the striatum?

Striatum = neostriatum

What is the Ventral pallidum?

Substantia innominata

Describe the success of surgical interventions for the treatment of chronic pain

Surgical intervention for the treatment of chronic pain has not been very successful. Surgical transection of pain pathways often produces temporary relief, followed by chronic pain that can leave the patient more disabled than before.

Pick True or False for the following statements regarding Purkinje cells: c. they are inhibitory

TRUE

Pick True or False for the following statements regarding Purkinje cells: e. some project directly to the vestibular nuclei

TRUE

Pick True or False for the following statements regarding Purkinje cells: some project directly to the deep cerebellar nuclei

TRUE

Pick True or False for the following statements regarding Purkinje cells: a. they receive direct input from climbing fibers

TRUE

What is Tabes dorsalis?

Tabes dorsalis is a condition associated with the tertiary stage of syphilis. It is characterized by slowly progressing degeneration of the dorsal columns (and, eventually, other pathways such as the optic nerves). A major symptom is ataxia, due to the loss of proprioception.

Where are taste receptors present?

Taste receptors are present on special receptor cells. These cells are non-neuronal. Numerous receptor cells are grouped with other cell types (support cells and basal cells) into taste buds. Taste receptor cells have an apical region with microvilli that contain the transduction apparatus. The microvilli are bathed in saliva, so only substances that dissolve in saliva can activate the taste cells.

Describe the Jaw jerk reflex

Tested by placing a couple of your fingers on the front of the patient's jaw and tapping to stretch jaw-closing muscles. Normal response is a closing "jerk" (this is sometimes hard to see). This is a monosynaptic reflex. Sensory and motor paths are both in trigeminal nerve; sensory link goes "through" mesencephalic V nucleus (where cell bodies are located) and terminates in motor V nucleus.

What type of somatosensation does the Anterolateral System convey?

The anterolateral system conveys information about PAIN and TEMPERATURE, as well as a CRUDE sense of TOUCH. Crude touch refers to a poorly-localized, poorly-identified sense that is not enough touch information to make up for a dorsal column/medial lemniscus problem. Damage to the anterolateral system causes deficits in both pain and temperature sensation. Given the clinical significance of pain, the rest of the notes will focus primarily on pain.

What forms the optic nerve and optic tract?

The axons of ganglion cells form the optic nerve and optic tract

What is the basal ganglia?

The basal ganglia include the caudate, putamen and globus pallidus, as well as a number of closely related nuclei. They influence motor systems primarily through projections to upper motor neurons.

Describe the interposed effects of mechanoreceptors in the case of reading braille

The different types of information conveyed by cutaneous mechanoreceptors is illustrated by comparing the responses of different afferents as a fingertip is moved across a row of Braille letters. All the afferent types are activated by this stimulation, but the information supplied by each type varies. Merkel afferents clearly recognize the details of the Braille pattern; Meissner afferents supply a slightly coarser version of this pattern. Details are lost in the Ruffini and Pacinian afferents, because these responses have more to do with tracking the movement and position of the finger than with the specific identity of the Braille characters.

Describe the role of the frontal eye field in eye movements

The frontal eye fields project directly to the gaze centers and also project to the superior colliculus. The figure shows this relationship for projections to the PPRF. Note that the right frontal eye field will affect the left PPRF. Like the rest of the right motor cortex, the right FEF is concerned with the left half of the world and will therefore drive eye movements to the left. Conversely, of course, the left FEF projects to the right PPRF.

What is the function of the myotactic reflex?

The functional role of this reflex is to regulate the tone of the innervated muscle a. Muscle length and tone influence the force of contraction that can be generated by a muscle b. Muscle length is a measure of joint position, so the reflex can control joint position

Is the method of transduction the same in all somatosensory afferents?

The fundamental mechanism of sensory transduction - the process of converting the energy of a stimulus into an electrical signal - is similar in all somatic sensory afferents.

What makes up the lentiform nucleus?

The globus pallidus and the Putamen

Describe the purpose of the gustatory system

The gustatory system detects ingested tastants, which are primarily water or fat-soluble molecules. Tastants provide information about quality, quantity and safety of ingested food.

Generally describe the enteric nervous system

The neurons of the gut are organized into 1) the Myenteric (Auerbach's) plexus, located beneath the longitudinal muscle layer, and 2) the Submucous (Meissner's) plexus, located beneath the circular muscle layer. The Myenteric regulates the musculature of the gut; the Submucous monitors chemicals and glandular secretion.

Describe the function of the spinal trigeminal nucleus

The nucleus has 3 parts. The most caudal part, "pars caudalis", processes pain and temperature (comparable to the ALS in the spinal cord). Another part of the spinal V nucleus projects to the cerebellum (equivalent to the spinocerebellar pathways, processing non-conscious proprioception).

Describe the purpose of the olfactory system

The olfactory system detects airborne molecules, called odorants. Olfactory information influences social interactions, reproduction, defensive responses and feeding behavior.

Compare and contrast the roles of medial and lateral descending motor pathways

The organization of the lower motor neurons establishes somatotopic organization: -Medial: control of trunk for posture and movement -Lateral: more distal muscles/limbs, especially important for fine motor control.

Briefly describe the anatomy of the ear

The outer ear includes the pinna, external auditory meatus, and tympanum. The middle ear includes the middle ear ossicles and muscles (not shown), oval and round windows. The cochlea is the auditory structure of the inner ear.

What are the major pathways through the basal ganglia?

The pathways through the basal ganglia can be grouped into a direct pathway and an indirect pathway. Understanding these pathways is key to understanding the results of damage to different regions. We will trace these pathways through their effects on VA/VL, and thus on motor cortex.

What is the current view of the hypothalamus's role in feeding behavior?

The role of the hypothalamus: current view is that arcuate nucleus, lateral hypothalamic area, and paraventricular nucleus play major roles

Why can a lesion of the sub thalamic nucleus produce unwanted movement?

The subthalamic nucleus is part of the indirect pathway. As stated above, the indirect pathway normally decreases movement; with a subthalamic lesion, the indirect pathway is non-functional, reducing inhibition of the thalamus and allowing for the spontaneous appearance of movements.

Describe the role of the superior colliculus in eye movements

The superior colliculus projects directly to the gaze centers. Recall that the superior colliculus gets direct input from the retina; since this input is post-chiasmatic, each superior colliculus receives information about the contralateral visual field. Thus, the left superior colliculus "sees" the right visual world, and is concerned with moving the eyes to the right. Accordingly, it projects to the right PPRF.

What is the task of the peripheral auditory system?

The task of the peripheral auditory system is to convert airborne mechanical waves into electrochemical activity in nerve cells.

What happens with a total loss of the visual cortex on one side?

TotallossofvisualcortexononesideresultsinContralateralHomonymous Hemianopia.

What types of eye movements does the vestibular system control?

The vestibular system works best for short and/or fast movements. If a large part of the visual scene moves, such as when one looks out the window of a moving car, our eyes tend to fixate on an object and to follow that object, with a smooth pursuit type movement. This is called the optokinetic reflex. When the object of fixation leaves our field of view, we make a saccade to a new object in the scene. The cycle of movements that occurs -- a slow movement in one direction followed by a quick saccade in the opposite direction -- is known as nystagmus.

What are the functional subdivisions of the cerebellum?

There are 3 functional subdivisions of the cerebellum, named for the part of the brain they are most closely related to: 1. Vestibulocerebellum 2. Spinocerebellum 3. Cerebrocerebellum

Describe the relationship between the vestibular nerve and the cerebellum

There is direct input from vestibular nerve to nodulus and fastigial nucleus of cerebellum (via juxta-restiform body).

Describe the cerebellar lobes

Three main lobes 1. Anterior lobe -----Primary fissure 2. Posterior lobe -----Posterolateral fissure 3. Flocculonodular lobe • nodulus (= nodule; single one on midline) • flocculus (1 on each side)

Where is the reticular formation located?

Throughout the core (tegmentum) of the brainstem

What is muscle tone?

Tone: level of resistance to passive stretch

What is anosmia?

Total loss of the sense of smell.

Describe the mechanism of urination

Urination occurs through coordinated parasympathetic activation [S2-S4 via pelvic nerve] and inhibition of sympathetic activity and crucially, the voluntary relaxation of the external urethral sphincter muscle a. Bladder contraction b. Internal sphincter relaxation c. External sphincter opening via relaxation of tonically active motor neuron input to striated sphincter muscle fibers

Describe the Thalamic Relay of the Dorsal Column System

Ventroposterolateral Nucleus

What are vergence movements?

Vergence movements allow the fovea of each eye to be aligned to objects at a specific distance. Convergent movements allow the eyes to fixate on a nearer object, while divergent movements allow the eyes to fixate on a more distant object. Vergence movements are non-conjugate, meaning that the eyes move in different directions from one another. In contrast, the other types of eye movements are conjugate, where the eyes move in the same direction.

Give a detailed explanation of sound transduction by cochlear hair cells

Vertical movement of the basilar membrane is translated into a shearing force that deflects the sterociliary bundles of the hair cells. The deflection of the stereocilia in the upward phase opens potassium channels, allowing potassium to enter and depolarize hair cells. The downward deflection of the basilar membrane closes channels and hyperpolarizes the hair cell. cells, two types of receptors cells called hair cells.

How are voluntary saccades initiated?

Voluntary saccades are initiated by the frontal eye fields. We also make reflex saccades to visual and non-visual targets; these reflex saccades are mediated by the superior colliculus. Retinal projections to the superior colliculus underlie the saccades to a flash of light or sudden movement. We also will make a saccade to a loud sound or to a touch. Both the auditory and somatosensory pathways project to the superior colliculus.

What are the characteristics of a receptive field?

a. Fundamental property of a sensory neuron (peripheral or central) describing the region of the sensory surface where an appropriate stimulus alters the firing rate of that neuron (Fig. 6A). b. Sensory neurons generally have limited receptive fields because they receive input from receptors on a specific part of the sensory surface. Sensory neurons therefore respond to stimuli on only a limited part of the sensory surface c. The size of receptive fields can vary substantially d. In CNS, 2nd order neurons may have receptive fields that are larger than peripheral neurons due to convergence.

Describe the auditory function of the outer ear?

a. Funnels sound vibrations to middle ear via tympanic membrane. b. Selectively amplifies some frequencies of sound. c. Role in locating sound sources.

What are the related forebrain structures of the limbic system?

a. Hypothalamus: mammillary bodies, ventromedial nucleus, lateral hypothalamus.

Describe the Structures associated with reward and motivation

a. Neocortical areas including orbitofrontal, medial prefrontal, and anterior cingulate cortices drive many motivated behaviors. b. The mediodorsal nucleus of the thalamus is most closely associated with these cortical areas. c. Rewarding behaviors are reinforced and regulated by a "basal ganglia" type circuit involving the ventral d. Interconnections between nucleus accumbens and amygdala assess the emotional content and valence (positive or negative affective nature) of stimuli. e. Habenula regulates activity of reward circuitry

Describe The visual field and the pathway to visual cortex

a. Projection of visual field onto retina b. Ascending pathway contains crossed and uncrossed components c. The optic tract, visual thalamus, and visual cortex contain an orderly, retinotopic representation of the contralateral visual field

Describe the distribution of analgesia following damage to complete transection of the cord at L5

bilateral, sacral dermatomes.

Describe the juxtarestiform body connection

connections of cerebellum with vestibular system

For the dorsal column-medial lemniscus pathway, identify the type of information carried

discriminative touch (2- point discrim.), vibration, pressure, conscious proprioception

Parkinson's disease results from loss of neurons containing what neurotransmitter? Where are those neurons located? Where do they normally project? What is the pathway called?

dopamine, substantia nigra pars compacta, project to striatum, nigrostriatal pathway

Describe the organ of Corti

i. Contains sensory hair cells and various supporting cells ii. In each cross-section, 1 inner hair cell and 3 outer hair cells iii. Hair cell stereocilia project in endolymph of scala media, contacting tectorial membrane.

Describe how the Habenula regulates activity of reward circuitry

i. Habenula neurons are activated by unpleasant events ii. Inhibits dopaminergic ventral tegmental neurons

Describe M-Type Ganglion Cells

i. M-type (magnocellular = large): • larger receptive fields • responds well to brief / moving stimuli • little or no color information • mediate motion an general spatial sensitivity

Describe the inner hair cells of the cochlea

i. Make contact with 95% of auditory nerve fibers ii. Supply most of the information from cochlea to CNS

Describe the central modulation of sensory information processing

i. Neurons at higher levels of sensory system project back to neurons at lower levels to modify/adjust (=modulate) their processing of sensory stimuli (e.g. auditory cortex projects back to auditory midbrain to enhance responses to certain stimuli). ii. Other brain centers connect to sensory systems to modify activity depending on a person's internal state (e.g., modulation of nociceptive input from brainstem) (Fig. 10).

Describe the maintenance of GnRH secretion in males

i. Pulsatile GnRH release is maintained in a more or less regular manner throughout sexual maturity, ii. The rate and amount of GnRH release is regulated by circulating testosterone and inhibin.

Describe the varying sizes of receptive fields

i. Receptive fields of sensory receptors in the skin are smallest on fingers/lips ii. Size of receptive field affects precision of sensory perception (Fig. 6B)

Describe the basilar membrane of the cochlea

i. Runs length of cochlea ii. Separates scala media and scala tympani iii. Sensory organ (organ of Corti) rests on basilar membrane

Describe how the middle ear transmits sound to the inner ear

i. Sound transmitted from tympanic membrane via the middle ear ossicles (the maleus, incus, and stapes) to the oval window. ii. The net effect of this ossicular chain is to convert airborne vibrations into fluid borne vibrations within the cochlea with little loss of energy.

Explain how the Primary visual cortex systematically represents the visual field

i. based on an orderly representation of the retina (but you should remember the visual field representation) ii. Macular representation at occipital pole is disproportionately large (central 10° of visual occupies about 50% of primary visual cortex) iii. upper visual field represented in lingual gyrus iv. lower visual field represented in cuneus v. lateral retinal field (extrafoveal region) represented rostrally along calcarine fissure

Describe the outer hair cells of the cochlea

i. contact only ~5% of auditory nerve fibers ii. regulate cochlear tuning, sensitivity of inner hair cells

What are the five cells types of the cerebellar cortex

inhibitory 1. Purkinje cells 2. basket cells 3. Golgi cells 4. stellate cells excitatory 5. granule cells

For the dorsal column-medial lemniscus pathway, identify the location and level at which the pathway crosses the midline

internal arcuate fibers in lower medulla

Describe the distribution of analgesia following damage to right anterolateral tract at the level of T4

left side of body, up to T4.

Hemiballism affecting the right limbs indicates damage where? (name nucleus and side).

left subthalamic nucleus.

What taste region is innervated by Glossopharyngeal

tongue - posterior third

What taste region is innervated by the Facial (chorda tympani)?

tongue - anterior 2/3

Describe the origins of the pyramidal tract

• 1/3 from M1 (primary motor cortex) • 1/3 from premotor areas (premotor and supplementary motor cortex) • 1/3 from primary somatosensory cortex • NOT entirely motor in function

Describe the pigment epithelium of the retina

• Absorbs stray light • Provides metabolic support for photoreceptors

What are the main targets of upper motor neurons?

• alpha motor neurons • gamma motor neurons • interneurons

How do tone and reflexes figure in clinically?

• both vary between individuals • both vary at different times for single individual • frequently altered with damage to CNS

What does Parkinson's Disease lead to?

• decreased activity in the direct pathway, and • increased activity in the indirect pathway. Both effects lead to greater inhibition of VA/VL and hypokinetic symptoms (the resting tremor is an exception, of course; there are thoughts on this, but we won't go into them.)

What types of lesions can cause spasticity?

• lesion cortex or internal capsule - contralateral limbs spastic. • Lesions at subcortical levels (in diencephalon and extending into the midbrain or beyond) can cause particular patterns of spasticity. These lesions affect reticular formation and often alter consciousness.

What are the main targets of the corticobulbar tract?

• motor V nuclei • facial nuclei • nu. ambiguus • hypoglossal nuclei

Generalize the inputs and outputs of the cerebellum

• the inputs are mossy and climbing fibers • the outputs are from deep nuclei

Give an overview of cerebellar clinical signs

1. "cerebellar signs" result from damage to the cerebellum or its inputs or outputs. 2. damage to the cerebellum causes ipsilateral deficits. 3. lesions of the deep cerebellar nuclei are more severe than lesions in the cortex

What are the common features of a sensory system?

1. A receptor cell that responds to an appropriate stimulus 2. A neural network that processes information about the stimulus 2. Projections to brain centers that can act on the information (e.g., motorneurons, autonomic neurons, higher brain centers)

What are some tests of hearing function?

1. Audiometry 2. Simple screening tests -Rinne's test -Weber's test 3. Sophisticated clinical test, e.g., auditory evoked potentials

Describe the ascending neural connections of the hypothalamus

Ascending neural projections to amygdala, hippocampus, thalamus, and cerebral cortex -Projections contribute to emotional expression, reward systems in brain, memory -Projections influence coordinated behaviors such as: 1. temperature regulation (moving out of sun, putting on coat) 2. feeding (going to kitchen to prepare meal) 3. social interactions (pair and parental bonding, social interactions) 4. somatic expression of emotion

What does central sensory dysfunction usually result from?

Central dysfunction usually results from circulatory, tumor, or degenerative disorders

Describe the function of the chief sensory nucleus of V

Chief (or "principal" or "main") sensory nucleus - discriminative touch / conscious proprioception (comparable to the dorsal column/medial lemniscal system).

Describe taste sensory activation in the cortex

Each taste elicits focal activity in the taste cortex. Thus, from the taste receptors, which are different for each taste, to the cortex where taste processing localized to different regions, tastes are segregated throughout the taste system.

What does emotional expression also rely upon?

Emotional expression also depends on cingulate, orbitofrontal and medial prefrontal cortex, based in part on direct and indirect projections of amygdala. This was basis of frontal lobotomy: by severing inputs to prefrontal cortex, it was hoped that undesirable aspects of mood would be eliminated.

What inputs and outputs are common to all areas of the cerebellum?

In addition to their "dominant" input, each area of the cerebellum receives input from the inferior olive (i.e., all areas have climbing fibers). Also, all subdivisions send output to the VL and thus to motor cortex. Differences in the details of this projection allow different types of information to be sent from the different parts of cerebellum to different parts of cortex (as you might imagine, information about planning is most closely related to "higher" motor cortical areas such as supplementary and premotor areas).

Do all nociceptors produce pain?

In addition to their role in pain sensation, nociceptors also transduce stimuli that don't cause a sensation of pain, such as low levels of heat or cold that don't cause tissue damage (non-noxious stimuli), and therefore don't produce pain. Thus, while pain results from nociceptor activation, not all nociceptor activation produces pain.

Besides learning, emotion, and reward, what does the limbic system participate in?

Limbic structures are also related to olfaction, to features of personality, and to attention. Many psychoactive therapeutic drugs act on neurons of the limbic system.

What is analgesia?

Loss of pain and temperature sensation

Describe the central control mechanisms involved in the sexual response

Medial preoptic area [integrates hormone and sensory information]; amygdala, bed nucleus of stria terminalis and thalamus [execution and reward]; hypothalamus [erection and lordosis]; Additional hypothalamic nuclei are integrative centers for sexual response, preference and gender identity.

Is the area of the cortex that makes up regions of the homunculus proportionate?

No. The homunculus in cortex is disproportionate - areas of the skin with a greater density of receptors (like the fingers) have more cortical space dedicated to them. This is another factor (along with receptive field size) that determines two-point discrimination thresholds.

Give the General principles of nociception

Nociceptors (pain receptors), like other somatic sensory receptors, arise from cell bodies in dorsal root ganglia (or in the trigeminal ganglion), and send one axonal process to the periphery and the other into the spinal cord or brainstem.

How are nociceptors classified?

Nociceptors can be classified according to their responsiveness to diferent stimuli. For example, mechanonociceptors are activated by mechanical tissue damage, thermonociceptors are activated by tissue damage caused by heat or cold, and chemonociceptors are activated by tissue damage caused by chemicals

Describe hypothalamic output that takes place via the pituitary

Output to and via the pituitary (Fig. 7): Hypothalamus is the master gland 1. Parvicellular (small-sized) neurons in the hypothalamus control secretion by the anterior pituitary 2. Magnocellular (large-sized) neurons of the supraoptic and paraventricular nuclei send axons via supraoptico-hypophysial tract to terminate on the capillary bed of the posterior pituitary

What is Bradykinesia?

Poverty of movement". Movements are slower and smaller than normal.

Describe the traveling pressure wave in the cochlea

Pressure applied by stapes at oval window increases pressure in scala vestibuli. This pressure wave causes vibration of the basilar membrane. This vibration moves along the basilar membrane from base to apex as a Traveling Wave.

What is hypotonia?

Reduced muscle tone is a common consequence of cerebellar damage. This reduced tone is believed to be responsible for pendular reflexes. Normally, after the patellar reflex is elicited, the leg returns to its resting position. With cerebellar damage, the leg may swing back and forth, "like a pendulum" after the reflex jerk.

What is Hypogeusia?

Reduced taste (lesions that affect peripheral or central pathways (e.g., Bell's palsy; lateral medullary syndrome).

What is a reflex?

Reflexes are specific, stereotyped motor responses that occur as a result of specific sensory input.

What do retinal ganglion cells respond best to?

Retinal ganglion cells respond optimally to contrast in their receptive fields. These receptive fields are represented here as circular patches on the retina; the receptive field is composed of "center" and "surround" regions with different responses to light. The figure shows the responses of an ON-center cell to five light stimuli falling on different parts of the receptive field (the stimulated portion of the receptive field is shown in white). The pattern of action potentials fired by the ganglion cell in response to each stimulus is shown. Duration of illumination is indicated by a bar above each record.

A patient has difficulty walking. How would you distinguish damage to the spinocerebellar pathways from damage to the dorsal columns?

Romberg test - see description in notes; 2-point discrimination etc to test dorsal columns.

Describe symptoms of spinocerebellar lesions

Spinocerebellum - vermis/fastigial nucleus: body sway, dysarthria - paravermis/interposed nu: limb ataxia, intention tremor

Describe the Receptor Surface of the Dorsal Column System

Skin, Joints, Skeletal Muscle

What is athetosis?

Slow, writhing, worm-like movements

Describe how the firing of the vestibular nerve depends on the bending direction of the hair cells.

The firing of vestibular nerve fibers depends on the direction in which the hairs are bent. Bending toward the kinocilium causes hair cells to depolarize and produces an increased rate of firing in the afferent fibers. Bending away from the kinocilium causes the hair cells to hyperpolarize and produces a decreased rate of firing in the afferent fibers.

Describe the limbic inputs of the hypothalamus

The hypothalamus receives multiple inputs from the limbic system (Fig. 10). These inputs are related to 1) emotions and their expressions, e.g., the amygdala, 2) learning and memory, and 3) olfactory information. To some extent, the limbic system input can be viewed as a modulator of the integration performed by the hypothalamus. In some situations, however, the limbic system is the master controller, orchestrating a broad range of responses to a highly significant sensory stimulus.

What is the main excitatory loop of the cerebellum?

The main excitatory loop includes the input fibers and the deep cerebellar nuclei, which project out of the cerebellum to descending motor systems. Both the inputs and the cells of the deep nuclei are excitatory; hence the name of the circuit.

How does the motor cortex project towards lower motor neuron?

The motor cortex projects both directly and indirectly to lower motor neurons

Describe the motor projections of the superior colliculus

The projections from the superior colliculus constitute the tectospinal tract, which has the job of turning our gaze to an area of interest. The tectospinal tract takes care of any head movements that could be required. As you might expect, this involves only muscles innervated by the upper cord, and the tectospinal tract terminates only in cervical regions. We will consider this tract again in the lecture on eye movements.

Why doesn't the corticobulbar tract target CN XI?

The spinal accessory nucleus is in the spinal cord, so corticospinal projections take care of it, not corticobulbar projections.

Describe the purpose of the chemical trigeminal system

The trigeminal system provides information about irritating or noxious molecules that come into contact with the skin and mucous membranes of the eyes, nose or mouth.

Describe sensory disorders affecting support structures

These prevent a stimulus from properly acting on receptor cells (e.g., conduction deafness, cataracts).

What does the transduction of sound to the inner ear depend on?

Transduction depends on transmission of sound energy from middle ear to inner ear. -Pressure applied by stapes at oval window increases pressure in scala vestibuli. -This pressure wave causes vibration of the basilar membrane, which activates hair cells.

What is the vestibular system?

Vestibular system: Designed to sense and analyze orientation with respect to gravity, linear acceleration, and angular acceleration (associated with head turning).

How can a ganglion cell become sensitive to contrast?

When the center cone has one type of color sensitivity (e.g.,green) and the surrounding cones have a different kind of color sensitivity (e.g., red), a ganglion cell becomes sensitive to color CONTRAST between the center and the surround of its receptive field. This is essential for color vision.

Do humans use pheromones?

While many mammals have a vomeronasal system that mediates pheromone- induced signaling, there is insufficient evidence that such a system is functionally present in humans

What are the contributions of the Trigeminal Nerve to taste?

While not often considered in this context, the trigeminal system makes an important contribution to what we call "taste". Nociceptive receptors in the mouth are sensitive to irritants such as capsaicin, which provides the "zing" in hot peppers, or ethanol (hence the burning sensation of strong liquor). This "pain" information is relayed to cerebral cortex along with other trigeminal pain and temperature information (i.e., via spinal V tract and nucleus, ventral trigeminothalamic tract, etc.).

Describe inhibition within the olfactory glomerulus

Within each glomerulus, the dendrites of periglomerular cells form inhibitory synapses with mitral cell dendrites (these are dendro-dendritic synapses). The secondary dendrites of mitral and tufted cells form excitatory synapses on the dendrites of granule cell interneurons. The granule cells make inhibitory synapses on other dendrites. This forms a "curtain of inhibition" that must be overcome by a strong enough excitation caused by an odorant binding to a receptor cell. And since only strong responses to odorants at the receptor level can overcome this inhibition curtain, odorant responses are recorded at the level of the glomerulus as excitatory "peaks". These discrete peaks of excitation that stand out from the surrounding inhibition are what confer odorant- selectivity to the responses.

What neurotransmitters do the basal ganglia circuits use?

a) glutamate b) GABA c) acetylcholine (in striatal interneurons) d) dopamine (in SNc neurons that project to striatum)

Describe the organization of the utricle

a. Large chamber at the confluence of the three semicircular canals. b. Receptor surface is called the macula; composed of hair cells, support cells, and nerve fibers. c. Sitting on top of hair cells is the otolithic membrane, a gelatinous mass with calcium carbonate crystals.

Describe disorders resulting from improper function of receptor cell or sensory neurons

a. genetic receptor deficits: e.g. color blindness, specific anosmias b. acquired deficits due to infection, drugs, trauma, etc.

10.Spinal shock includes a. hypotonia b. hypertonia

a. hypotonia

What are the fiber pathways of the limbic system?

anterior commissure, cingulum, fornix, mammillo-thalamic tract, medial forebrain bundle, stria medullaris, stria terminalis, ventral amygdalofugal pathway

For the dorsal column-medial lemniscus pathway, identify the locations of first-, second- and third-order neurons

dorsal root ganglion; dorsal column nuclei; VPL

What lesser symptoms may be associated with cerebellar damage?

hypotonia, ataxia may be present with any cerebellar damage. Signs of cerebellar damage are discussed in more detail below.

What is Papilledema?

i. "choked disk" visible in ophthalmic inspection is result of swollen optic disk at the head of the optic nerve arrow shows the path of light through the optical apparatus. ii. results from high intracranial pressure, exerted through subarachnoid space to eye iii. damage to axons of optic nerve can lead to partial or complete loss of vision in the affected eye

Which limb of the internal capsule carries pain and temperature information from the body?

posterior limb

Where else do Purkinje cells project to?

• Also, the Purkinje cell project to vestibular nuclei (Vestibulocerebellum: Source of outputs) is the "fine print" reference earlier about a small amount of cerebellar output originating from something other than the deep nuclei) .

Describe neuroimmune pain

• Changes induced by spinal cord injury (SCI) caudal to the lesion illustrate neuroimmune signaling remote from the primary injury site at the thoracic level. • Immune cells rapidly infiltrate the CNS at lesion level, whereas below lesion, microglia switch from resting (Micr) to active (Mica) phenotype, concomitant with increased levels of PGE2 and CCL21 and contribute to the accumulation of proinflammatory molecules and other neuroexcitatory signals that impact the function of nociceptive relay neurons in the dorsal horn. • Microglia in the VPL nucleus become hyperactive owing to, at least in part, remote signaling from lumbar spinal cord levels below thoracic SCI mediated by CCL21, resulting in abnormal burst activity in the thalamus and amplified gain in the cerebral cortex (Cc) mediating an abnormal sensation of pain.

Give an overview of the parts of the motor system

• Lower Motor Neurons - coordinate functions (e.g., reflexes, locomotion) • Upper Motor Neurons - influence LMNs. - esp. muscle tone, posture • Motor cortex - initiate, plan, coordinate voluntary movement

Describe the inputs of the basal ganglia

• Most inputs enter the striatum. • The majority of inputs come from cerebral cortex. Additional inputs come from intralaminar nuclei of the thalamus. • These inputs are excitatory.

Why doesn't the corticobulbar tract target CN XI, VI, IV, or III?

• Other pathways target these nuclei- axons travel with corticobulbar, but are not part of UMN system (we'll discuss these later): • corticorubral (to red nucleus) • corticopontine (to pontine nuclei)

Describe the photoreceptor cells of the retina

• Rods, Cones • Contain photopigments, which absorb photons o Photopigment = light-absorbing chromophore retinal (an aldehyde of vitamin A) which is coupled to an opsin protein o The type of opsin tunes the wavelength sensitivity of the photopigment; different opsins across rods and cones

Describe the role of the suprachiasmatic nucleus on vision

1. Receives input via retino-hypothalamic tract from photoreceptive ganglion cells 2. Regulates circadian rhythms

What are the four classes of mechanoreceptors specialized for tactile information in the skin?

(a) Merkel cell afferents (b) Meissner afferents (c) Pacinian afferents (d) Ruffini afferents

Where is the horizontal gaze center? What is it called?

-in medial pons, near abducens nucleus. -paramedian pontine reticular formation.

Describe Optic radiation (geniculo-striate or geniculo-calcarine pathway)

1. Axons of LGN cells project to Area 17, primary visual (striate) cortex a. Area 17 in occipital lobe b. Located along superior and inferior banks of the calcimine fissure, occipital pole 2. Neurons in dorsomedial part of LGN send axons to superior bank of calcarine fissure (cuneus) a. These LGN axons join the internal capsule, passing laterally and caudally over the inferior horn of the lateral ventricle, and underneath the parietal lobe b. These neurons carry information about the lower part of the contralateral visual field. 3. Neurons in ventrolateral part of LGN send axons to inferior bank of calcarine fissure (lingual gyrus) a. These LGN axons first sweep rostrally, into the white matter of the temporal lobe, passing over or in front of the inferior horn of the lateral ventricle. They then pass caudally to the lingual gyrus. The path these fibers take through the temporal lobe is called Meyer's loop. b. These neurons carry information about the upper part of the contralateral visual field.

Describe the mechanism of the vestibulo-ocular reflex (when the head turns to the left).

1. Head turns to left. 2. Fluid motion in left horizontal canal causes shearing of hair cells to depolarize them (fluid motion in right horizontal canal hyperpolarizes its hair cells). 3. Vestibular nerve fibers on left side increase activity. 4. Increased activity in left vestibular nuclei. 5. Activation of motorneuron controlling right lateral rectus and left medial rectus. 6. Eyes move to right, at a angular rate equal to head movement.

What clinical issues are associated with the hypothalamus?

1. Homeostatic mechanisms: diabetes insipidus, fever, body fat composition 2. Control of visceral motor function: e.g., Horner's syndrome 3. Endocrine disorders

Describe the role of the lateral vestibulo-spinal pathway

1. Mediates lateral vestibulo-spinal reflex: head/body tilt or linear acceleration in any direction activates antigravity muscles to counteract the tilt or acceleration 2. Functional Role: To maintain posture and an upright position (with respect to gravity). 3. Net effect is to increase tone in axial and extensor musculature.

Describe the structure of the cochlea

A. The cochlea, part of the inner ear, consists of three fluid-filled tubes that extend throughout its entire length of ~ 35 mm. B. Cross section of cochlea shows arrangement of the three tubes. The oval window, against which the stapes pushes in response to sound, communicates with the scala vestibuli. The scala tympani is closed at its base by the round window, a thin, flexible membrane. Between these two compartments lies the scala media, an endolymph-filled tube whose epithelial lining includes the hair cells resting on the basilar membrane. The organ of Corti contains, in addition to many support cells, two types of receptors cells called hair cells.

Describe the orientation and function of the semicircular canals in the head

A. The horizontal canals on both sides lie in the same plane and therefore are functional pairs. B. The anterior duct on one side and the posterior duct on the opposite side lie in the same plane and are therefore functional pairs. C. Superior view of the horizontal semicircular canals illustrates how the paired canals work together to signal head movement.

Describe the Primary Cortical Area of the Dorsal Column System

Areas 3 and 1 (Post-central gyrus)

What gives rise to somatic sensation?

Arises from information provided by a variety of receptors distributed throughout the body.

Describe action potential generation by sensory receptors.

At some more central site, the graded receptor potential causes action potentials in the sensory neuron

Where does trigeminal sensory information cross the midline?

At the level of each second order neuron; i.e., from lower spinal V nucleus all the way up to chief sensory V nucleus.

Describe auditory brainstem responses

Auditory brainstem responses (ABR) obtained in response to repeated brief sounds. Each peak represents the activity of many neurons, usually within a particular anatomical region. For instance, peak "I" is thought to represent activity in the auditory nerve. Loss or delay of these peaks can help diagnose type of hearing loss. The ABR in response to sound in the right ear indicates a pathology between the auditory nerve and the auditory brainstem. This is consistent with a vestibular schwannoma ("acoustic neuroma").

What is Babinski's Sign?

Babinski's sign is an abnormal response to stroking the lateral plantar surface of the foot. Normally, the toes plantar-flex in response. Following damage to the corticospinal tract, the same stimulus produces a dorsiflexion (plantar extension) of the big toe. This is Babinski's sign (sometimes called a positive Babinski sign). It is normally absent, except in babies, in whom the corticospinal tracts have not completely myelinated. Other than in babies, a Babinski sign indicates corticospinal damage.

What are some clinical considerations regarding memory, or lesions of the hippocampus?

Clinical issues: lesions of hippocampus or related nuclei diminish consolidation of short-term memory to long-term memory 1. Ischemia affects hippocampal formation 2. Korsakoff's syndrome 3. Traumatic or surgical lesions of diencephalon or temporal lobe 4. Alzheimer's disease:

What is the effect of dopamine on basal ganglia pathways?

DA excites the direct pathway (D1 receptors) and inhibits the indirect pathway (D2 receptors).

Describe the effect of lesions of the somatosensory cortex on pain.

Damage to somatosensory cortex (e.g., from a stroke) can reduce the ability to localize pain, but does not eliminate the ability to feel pain. Note that a stroke of MCA (middle cerebral artery) could knock out much of primary somatosensory cortex, but would leave anterior cingulate cortex intact (which, as you recall, is a target of the spinoreticulothalamic pathway).

Give an example of a limbic input and its effect on the hypothalamus

Example: a stress-inducing visual stimulus (e.g., a large predator). Emotional reaction to that stimulus is based on neural processing in the amygdala. In turn, the amygdala projects to the hypothalamus to generate some of the responses to stress: widespread activation of the sympathetic nervous system and secretion of corticotrophins through the anterior pituitary.

What is rigidity?

Excessive muscle tone. This is different from spasticity, seen after UMN lesions, because spasticity includes hyperreflexia, which is not present in rigidity.

What receptor is used to bitter tastants?

For bitter tastants, a distinct set of G-proteins, the gustducins, are involved in the transduction process. Gustducin is not found in sweet or amino acid receptor-expressing taste cells.

What is the role of the superior colliculus in motor function?

For control of certain eye movements; to be described in an upcoming lecture.

What is the relationship between axon diameter and muscle innervation?

Generally, smaller motor axons contact fewer muscle fibers and larger axons contact greater number of muscle fibers.

What acts as the final sensory receptor for sound transduction?

Hair Cells

Describe the horizontal gaze centers

Horizontal eye movements are coordinated by the horizontal gaze centers. Each horizontal gaze center (one on each side of the brain) is located in the paramedian pontine reticular formation, or PPRF, which is near the midline in the pons close to the abducens nucleus.

What is Horner's Syndrome?

Horner's syndrome is a loss of sympathetic input to the eye/face characterized by: • pupillary constriction (miosis) • partial ptosis (drooping eyelid) • anhidrosis (loss of facial sweating)

Describe Hyperkinetic Disorders

Hyperkinetic disorders (Huntington's) are characterized by dyskinesia (unintentional movements). The movements take various forms, from • Ballism (large-amplitude, violent movements of an entire limb) to • Chorea (jerky, dance-like movements) to • Athetosis (worm-like writhing movements). -Note that there are not necessarily strict distinctions between these types of movements; a common term is "choreoathetosis", indicating a combination traits.

What are Hypokinetic Disorders?

Hypokinetic disorders are characterized by reduced movements

Describe the hypothalamic control of pituitary secretion

Hypothalamic control of pituitary secretion. A. Parvicellular neurons of hypothalamus terminate onto a special capillary bed in the median eminence to release factors (e.g., TRH, GnRH) that control synthesis and release of hormones in the anterior pituitary. B. Magnocellular neurons from paraventricular and supraoptic nuclei send axons into the posterior pituitary to release the hormones ADH (vasopressin) and oxytocin.

What monitors internal temperature?

Hypothalamus is primary homeostatic regulator of internal temperature. 1. Internal temperature is detected by neurons of the anterior hypothalamus a. Cold-sensitive neurons b. Warm-sensitive neurons

Describe the general concepts of hypothalamus function

Hypothalamus receives sensory inputs and contextual information, compares to internal standards ("setpoints"), then sends information for responses by several systems.

Describe the cohlear nucleus, including its function, input, and output.

INPUT: Auditory Nerve OUTPUT: Trapezoid Body 1. Input from ipsilateral auditory nerve 2. Tonotopic organization in each of the cochlear nuclei. 3. Dorsal and ventral nuclei differ in responses • VCN analyzes timing of sounds • DCN analyzes vertical sound localization

Describe the primary auditory cortex, including its input and function.

INPUT: Auditory Radiation (of Internal Capsule) 1. Location on the superior surface of the temporal lobe, buried within the lateral sulcus (TRANSVERSE TEMPORAL GYRUS) 2. Area 41 is the primary auditory cortex, organized tonotopically. Low frequency sounds are analyzed rostro-laterally, and high frequency sounds analyzed caudo-medially. 3. Analysis of complex features: the frequency content of sounds, temporal patterns, cues used in localizing sounds, speech patterns. 4. Secondary auditory cortical areas in superior temporal gyrus.

Describe the medial geniculate body, including its function, input, and output

INPUT: Brachium of the inferior colliculus OUTPUT: Auditory Radiation 1. Receives ascending inputs from IC (mostly ipsilateral) via brachium of inferior colliculus. 2. It projects to several areas of auditory cortex 3. Serves as gateway to auditory cortex: Descending input from auditory cortex modifies activity of MGB neurons.

Describe the inferior colliculus, including its function, input, and output

INPUT: Lateral Lemniscus OUTPUT: Brachium of the Inferior Colliculus 1. Receives converging ascending input from all lower nuclei. A major function is to integrate information from all the brainstem auditory centers. 2. Input from contralateral side is dominant, but some input also occurs from ipsilateral side. Basis for contralateral representation of sounds in IC and higher auditory centers. 3. It is tonotopically organized.

Describe the superior olivary nucleus, including its function, input, and output.

INPUT: Trapezoid Body OUTPUT: Lateral Lemniscus 1. Principal nuclei of the SO are the first in the ascending auditory pathway to compare input to the two ears. These neurons analyze either time differences in the arrival of sounds at the two ears, or intensity differences in sounds arriving at the two ears. Both are important cues for locating a sound source. 2. Accessory nuclei project to cochlear nuclei and to cochlea. Olivo- cochlear pathway modulates cochlear function.

What happens if there is a total lesion of the optic radiation?

If total :loss of vision in the contralateral hemifield for each eye, termed Contralateral Homonymous Hemianopia.

What is agraphesthesia?

Inability to identify letters drawn on the skin

What is astereognosia?

Inability to name objects held against the skin

What will happen to eye alignment as a consequence of problems with eye muscles?

Lesions that interfere with actions of eye muscles (damage to the muscle, nerve or nucleus that innervates them) will usually cause the two eyes to be misaligned. This has two consequences. First, the patient's complaint isn't usually about eye movements per se, but double-vision. Second, the patient may, often subconsciously, adopt a head position that compensates for the deficit. For example, if the patient's left eye is deviated laterally, she may turn her head to the right, to bring the images from the two eyes into alignment.

What is a phantom limb?

Loss of a limb may lead to a "phantom limb", a phenomenon in which the patient feels that the limb is still present. Presumably the cortical representation of the lost limb remains intact for a considerable period of time, and in this period the limb remains associated with the mental image of the body.

Describe homonymous and heteronymous visual deficits

Many lesions cause deficits in the same part of the visual field viewed by both eyes, i.e., they are congruent. These deficits are termed homonymous. Lesions that are not in the same part of the visual field for each eye are termed heteronymous.

Describe the mechanism of muscle spindle involvement in the stretch reflex

Mechanism of spindle involvement in the stretch reflex: when the muscle is stretched, the intrafusal fibers are deformed. This deformation initiates action potentials by activating mechanically gated ion channels in the afferent axons innervating the spindle.

Describe the role of the medial premotor cortex

Medial premotor (a.k.a. supplementary motor) cortex is important in initiating and coordinating internally generated movements (self-initiated movements, "I want to reach over and pick up the cup"; movements from memory) as opposed to a movement elicited by sensory stimuli [e.g., visually-guided movements]).

What are the other reticular functions of the midbrain and medulla?

Midbrain: arousal Medulla: life support

What is the middle cerebellar peduncle connection called?

Middle cerebellar peduncle = brachium pontis -INPUT FROM PONTINE NUCLEI

Describe the anatomy of the taste pathway of CN X

N.X - vagus - pharynx - nodose ganglion.

What are the differences between the pain pathways?

One theory suggests that the spinoreticulothalamic system contributes to states of attention, awareness and levels of consciousness. Another theory suggests that this pathway is concerned with the emotive/affective aspects of pain sensation (remember the projections to the limbic system - insula and anterior cingulate). One fact is clear: when the spinothalamic tract is cut surgically to alleviate pain, it is usually successful only for a short period of time (few months to a year). Gradually the pain will return. It is assumed that this return of pain is due to the redundancy in the nervous system (i.e., the multiplicity of pain pathways) and that perhaps the bilateral spinoreticulothalamic system becomes increasingly activated.

What supplies blood to the visual cortex? What does a loss of this supply cause?

Posterior cerebral artery supplies all of visual cortex except occipital pole (where macula is represented). Some part of macular representation is supplied by middle cerebral artery. A loss of supply from the posterior cerebral artery results in Contralateral Homonymous Hemianopia (with macular sparing).

What gives sound its frequency?

Rate of air pressure change is its frequency, measured as cycles per second or Hertz (Hz). We perceive sound frequency as the pitch of sound. We are sensitive to sounds with vibrations of 20-20,000 Hz. Most mammals are sensitive to frequencies higher than 20,000 Hz, e.g., bats.

Describe the relative absorption spectra of the three types of cones and the one type of rod

Relative absorption spectra for rods and three types of cones. Comparison of activity in S, M, and L cones provides the basis for color vision. S-cones contain a photopigment most sensitive in the blue range. M-cones contain photopigment most sensitive in the green-yellow range. L- cones contain photopigment most sensitive in the red range. Rods do not contribute to color vision.

What makes up the striatum?

The caudate nucleus and the Putamen

What electronic device does the cerebellum act as?

The cerebellum acts as a comparator. It compares the intended movement (information it gets from cerebral cortex) to the actual movement (information it gets from sensory systems). It then sends corrective signals into the descending motor paths. It effects virtually all movements, and thus is important in balance, locomotion, simple and complex movements, eye movements, etc. It is essential for learning new motor skills and for adjusting movements to changing sensory inputs.

Describe the neural control of smooth pursuit

The circuits for smooth pursuit are not well understood, but appear to include many or all of the areas involved in saccades. Of course, since smooth pursuit movements are directly "tied" to a visual stimulus, these movements rely on a functional visual sensory pathway and visual cortex, which work in concert with the frontal and parietal eye fields. Recall that the striate - extrastriate - parietal stream of information flow - the so-called "where" stream - is concerned with motion analysis and the spatial aspects of vision. Damage to the parietal stream results in deficits of smooth pursuit. The cerebellum has also been implicated in smooth pursuit. Damage anywhere in the pursuit pathway, including the cerebellum, causes a patient to "track" a moving target with a series of short saccades.

What is the Sensorineural system?

The cochlea transduces sound, analyzes sound frequency, and controls the sensitivity of transduction and frequency analysis

Describe neurons in the enteric nervous system

The enteric nervous system is a network of neurons in the digestive tract that operates virtually autonomously from the central nervous system, with small inputs from the sympathetic and parasympathetic NS. It manages digestion. It is estimated there are more neurons in the enteric NS than in the spinal cord.

Where do the initiating events of the chemosensory transduction of taste occur?

The initiating events of chemosensory transduction occur in the taste cells, which have receptors on microvilli that emerge from the apical surface of the taste cell. The synapses that relay the receptor activity are made onto the afferent axons of the various cranial nerves at the basal surface. The apical surfaces of individual taste cells in taste buds are clustered in a small opening (about 1 mm) near the surface of the tongue called a taste pore. Taste cells have a lifetime of only about 2 weeks and are normally regenerated from basal cells.

What are Mechanoreceptors?

These receptors respond to mechanical deformation of tissue (stretch, touch, pressure, vibration). They are cutaneous receptors (found in the skin) and their behavior has best been studied in glabrous (hairless) portion of the hand (palm and fingertips). These regions of the skin surface are specialized for providing a high-definition neural image of manipulated objects.

Describe the role of the medial and dorsal prefrontal cortex in memory

These structures are implicated in multiple functions of the limbic system: learning/memory, emotions, olfaction

Describe transduction in the Vanilloid Receptor

Transduction by nociceptors is similar to those by other somatosensory receptors, except that the triggers for activating nociceptors can be different. Under resting conditions, the pore of the VR receptor channel is closed. When activated by capsaicin or high heat, the channel opens, allows an influx of sodium and calcium ions, depolarizes the membrane of the nociceptor cell, and causes action potentials to be generated in the nociceptor fibers - i.e. the A-Delta or C nerve axons.

What is intention tremor?

Tremor that occurs during movement; generally most severe at the end of the movement. This type of tremor is not present at rest. (Contrast this with the resting tremor seen in Parkinson's disease.) Intention tremor is usually most apparent in arm movements.

Where are the cell bodies of the primary afferent neurons for the trigeminal nerve?

Trigeminal ganglion (discrim touch, pain and temp) and mesencephalic trigeminal nucleus (proprioception).

Describe the difference in projection between the tufted an mitral olfactory cells

Tufted cells project mainly to the anterior olfactory nucleus and the olfactory tubercle, while mitral cells in the accessory olfactory bulb project only to the amygdala.

What are the signals for decreased fluid volumes in the hypothalamus?

Two signals for decreased fluid volume 1. Stretch receptors of the right atrium and aortic/carotid arches sense change in fluid volume. These receptors project via the vagus to the nucleus of the solitary tract (NST). 2. Decrease in blood flow to kidney activates renin angiotensin system. Angiotensin II (a potent vasoconstrictor) acts on cells of the subfornical organ in the forebrain (SFO). SFO, like nearby OVLT, has no blood brain barrier, so angiotensin II can directly activate SFO neurons.

What is two-point discrimination?

Two-point discrimination is defined as the minimum inter-stimulus distance required to perceive two simultaneously applied stimuli

Describe the differences in two point discrimination over the body

Two-point discrimination varies across the skin surface. For example, on the fingertips, two stimuli 2 mm apart can be distinguished, while on the forearm, the stimuli have to be at least 40 mm apart before they can be distinguished.

24 year old man, car accident victim; paralyzed from the waist down. Wheelchair-bound; cannot stand without support.

UMN

Trigeminal information is sent to which nuclei in the thalamus?

VPM and, for pain information, intralaminar nuclei as well.

Describe how vertical movements of the eye are accomplished, and how they can be modified by other muscle groups.

Vertical movements are more complicated. Elevation is controlled predominantly by the superior rectus and inferior oblique. Referring to the figure below (next page), note that when the eye is directed straight ahead, the superior rectus lies at an angle to the visual axis (about 23 degrees). If the right eye is directed to the right about 23 degrees, the superior rectus provides a simple elevation. In contrast, if the eye is strongly adducted, the action of the superior rectus would be more rotational (intorsional). In the adducted position, elevation is accomplished primarily by the inferior oblique. In any other position, elevation of the eye is accomplished by a combined effort of both superior rectus and inferior oblique. Depression of the eye is similarly controlled by a pair of muscles - the inferior rectus and the superior oblique - whose actions also depend on eye position.

What are the subcortical portions of the limbic system?

a. amygdala b. septal nuclei c. ventral striatum (including nucleus accumbens) d. ventral pallidum (including substantial innominata, locus of the basal nucleus of Meynert)

Damage to the left spinal trigeminal tract will produce what deficit (nature and distribution of loss).

analgesia for left face.

Nystagmus could result from damage to which of the following (more than one may be correct): a. frontal eye field b. cerebellum c. vestibular nuclei d. mediallongitudinalfasciculus e. oculomotor nerve

b. cerebellum c. vestibular nuclei d. medial longitudinal fasciculus

13.Spasticity, as we are defining it, includes which two of the following signs? a. hypotonia b. hypertonia c. hyporeflexia d. hyperreflexia

b. hypertonia d. hyperreflexia

9. Spinal shock refers to a condition resulting from damage to a. lower motor neurons b. upper motor neurons

b. upper motor neurons

What taste region is innervated by Vagus?

epiglottis, esophagus

Which part of the internal capsule carries trigeminal sensory information?

posterior limb (or adjacent genu).

Describe the role of the medial vestibulospinal tract

• originates from medial vestibular nucleus • driven by semicircular canals • terminates medially in ventral horn of cervical sp. cord • reflex action on neck muscles to control head position

Describe the role of the lateral vestibulospinal tract

• originates from lateral vestibular nucleus • driven by otolith organs • terminates medially in ventral horn of sp. cord • reflex action on trunk and proximal limb muscles, especially extensors

What is Ménière's disease?

• recurrent episodes involve bouts of tinnitus, hearing loss, and fullness in the ear • over the long term, there is progressive loss of low frequency hearing as hair cells die

What are the receptors of the dorsal column system?

Receptors: information from encapsulated receptors and hair shafts.

Describe the Vestibulocerebellum

Region: flocculo-nodular lobe Major input: Vestibular nerve and nuclei Other inputs: Inferior olive Source of outputs: Fastigial nu., Purkinje cells direct to vestib. nuclei Part of motor system targeted: UMNs of medial pathways, (Vest-sp; reticulospinal), VOR circuits (via mlf) Add'l Projections: VL Function: balance, eye movements Major signs of damage: staggering or falling nystagmus

Describe the Cerebrocerebellum

Region: lateral hemispheres Major input: Cerebral cortex via pontine nuclei Other inputs: Inferior olive Source of outputs: Dentate nu. Part of motor system targeted: Motor cortex (via VL) Add'l Projections: Red nu. (to inferior olive) # Function: coordination and planning of voluntary movements Major signs of damage: decomposition of movement

Describe the Spinocerebellum

Region: vermis and paravermal zone Major input: Spinal cord and trigeminal system Other inputs: Inferior olive Source of outputs: Interposed nu. Part of motor system targeted: UMNs of medial and lateral pathways Add'l Projections: VL Function: motor execution. (regulate tone, posture, locomotion) Major signs of damage: staggering gait intention tremor

What are the regions away from the fovea dominated by?

Regions away from fovea are rod-dominated(Fig.10A). • highly convergent (many rods onto one bipolar) • results in low spatial resolution but good detection of light (summation of the inputs of many rods into ganglion cells increases the likelihood of response)

Briefly describe the regulation of GnRH secretion

Regulation of GnRH neuronal activity is complex 1. Pulsatile release of GnRH may depend in part on communication among hypothalamic GnRH neurons 2. Inputs to GnRH neurons can modulate the timing and amplitude of pulsatile GnRH release -GnRH neurons receive inputs from many neuromodulator neurons -Variety of factors act through these neurons to alter GnRH secretion: e.g., nutritional state, growth, stress, light cycle and environmental cues all affect GnRH production/release.

Describe the lower-center inputs of the hypothalamus

Sensory inputs from lower centers arrive via the dorsal longitudinal fasciculus (dlf), medial forebrain bundle (mfb), and other pathways 1. Viscerosensory inputs associated with solitary nucleus (arterial baroreceptors, G-I tract input (e.g., distention of stomach and intestines), taste 2. Thermoreception from skin (via spino-reticular tract to midbrain, then to hypothalamus via mfb, dlf, etc 3. Mechanoreception from reproductive tract associated with childbirth, sexual stimulation 4. Visual inputs via optic tract

What are the two main types of sensory receptor structures?

Sensory receptors are either modified neurons or non-neural receptor cells

Describe the concept of receptor selectivity

Sensory receptors respond to a limited range of physical sensory energy. Selectivity is based on intrinsic properties of receptors (Fig. 4A), on adjacent support structures, or on where the receptor is located on a sensory surface

What types of transduction are used by sensory cells?

Sensory receptors use one of two general types of transduction mechanisms: 1. Direct transduction: the stimulus energy acts directly upon channels for various ions (used by mechanoreceptors and some taste receptors). 2. Indirect transduction: stimulus energy acts on receptor proteins that activate a 2nd messenger system, which in turn opens or closes an ion channel (e.g. olfactory chemoreceptors, photoreceptors).

What are sensory systems designed to do?

Sensory systems are designed to selectively receive information about the surrounding environment or about the internal state of our bodies, and to provide information upon which we can act.

What is transduction?

Sensory transduction is the process by which a physical stimulus triggers a change in membrane potential in the receptor cell. This receptor potential is graded; it is proportional to the strength of the stimulus. Sensory receptors use one of two general types of transduction mechanisms

What is sensory transduction?

Sensory transduction is the process of converting natural stimulus energy into neural activity. Each sensory system employs specific receptor organs.

Describe the transduction of somatosensory information

Somatosensory afferents convey information from the skin surface to central circuits. Somatic sensory receptors arise from cell bodies in dorsal root ganglia (or in the trigeminal ganglion), and send one axonal process to the periphery and the other into the spinal cord or brainstem. Action potentials generated in afferent fibers by touch, pain, pressure etc in the skin or muscle propagate along the fiber, past the cell body in the dorsal root ganglia until they reach the terminals, which are located in the CNS.

What is the Gate Control Theory of Pain?

The gate control theory of pain, put forward by Ron Melzack and Patrick Wall in 1962, is the idea that physical pain is not a direct result of activation of pain receptor neurons, but rather its perception is modulated by interaction between different neurons. The interplay among these connections determines when painful stimuli go to the brain. This theory doesn't tell us everything about pain perception, but it does explain some things. If you rub or shake your hand after you bang your finger, you stimulate normal somatosensory input to the projector neurons. This closes the gate and reduces the perception of pain.

Describe the mechanism of pain that is explained by the Gate Control Theory

When no input comes in, the inhibitory neuron prevents the projection neuron from sending signals to the brain (gate is closed). Normal somatosensory input happens when there is more large-fiber stimulation (or only large-fiber stimulation). Both the inhibitory neuron and the projection neuron are stimulated, but the inhibitory neuron prevents the projection neuron from sending signals to the brain (gate is closed). Nociception (pain reception) happens when there is more small-fiber stimulation or only small-fiber stimulation. This inactivates the inhibitory neuron, and the projection neuron sends signals to the brain informing it of pain (gate is open). Descending pathways from the brain close the gate by inhibiting the projector neurons and diminishing pain perception.

Describe the neurons of the DRG

(A) The cell bodies of somatosensory afferent fibers conveying information about the body reside in a series of dorsal root ganglia (DRG) along the spinal cord. Those conveying information about the head lie in the trigeminal ganglia. (B) Pseudo-unipolar neurons in the DRG give rise to a peripheral processes that ramify within the skin or muscle and central processes that synapse with neurons in the spinal cord and higher levels of the nervous system.

Describe Descending Control of Pain as explained by the Gate Control Theory

(A) The descending systems that modulate pain originate in the somatosensory cortex, the hypothalamus, the periaqueductal gray matter of the midbrain, the raphe nuclei, and other nuclei of the rostral ventral medulla. Modulation of pain occurs at each of these sites as well as in the dorsal horn. (B) Activation of mechanoreceptors modulates the transmission of nociceptive information to higher centers (gate theory of pain). (C) Enkephalin containing local circuit neurons exert descending control on nociceptive signal transmission.

Compare and contrast the center-surround representation of bits of visual information and the visual cortex representation.

(A) Unlike center-surround receptive fields in cells of the LGN that respond to spots of light (or dark), primary visual cortex (Area 17) contains many "simple" cells that respond to light (or dark) bars. (B) Receptive fields of several simple cells depict how they have different orientation selectivities. (C) A single Area 17 cortical cell responds best to a bar of a specific orientation. (D) Orientation-selectivity is created via convergence of LGN neurons with center-surround receptive fields (orange) onto a "simple" cell (purple) of visual cortex

What types of axons are associated with nociceptors?

(a) A-Delta group of myelinated axons (also called Group III fibers): conduction velocity 5-30 m/s; respond to dangerously intense mechanical or thermal stimuli. (b) C-fiber group of unmyelinated axons (also called Group IV fibers): conduction velocity < 2 m/s; respond to thermal, mechanical and chemical stimuli - referred to as polymodal. -Thus, there are fast (5-30 m/s) and slow (< 2 m/s) pain pathways.

Describe Pacinian Afferents

-Rapidly adapting fibers; make up 10-15% of mechanosensory innervation of the hand. Located deep in the dermis or in subcutaneous tissue. They resemble a small onion, with concentric layers of membrane surrounding a single afferent fiber. -Pacinian corpuscles adapt more rapidly than Meissners, are very sensitive - they can respond to skin displacements as small as 10 nanometers. -Their receptive fields are very large. -Pacinian corpuscles are well suited to detect vibrations transmitted through objects that contact the hand or are grasped.

Describe temporal discrimination of somatosensory input

-Adaptation of sensory afferents -Sensory afferents are differentiated by the temporal patterns of their responses to sensory stimulation. Some afferents fire rapidly when a stimulus is first presented, then fall silent with continued stimulation. Other afferents generate a sustained discharge in the presence of an ongoing stimulus. -Rapidly adapting afferents (those that become quiescent during a continued stimulation) are thought to be effective in conveying information about changes in ongoing stimulation, such as those produced by stimulus movement. -In contrast, slowly adapting afferents are better suited to provide information about the spatial attributes of a stimulus, such as size and shape.

Describe some of the pathologies that can affect the function of the vestibulo-ocular reflex

-Basic elements of the horizontal vestibular-ocular reflex. The reflex shown here is activated by a head turn to the left that causes fluid movement in the horizontal canals. The response is a movement of the eyes to the right. -The reflex depends on the integrity of the medial longitudinal fasciculus (mlf): Fibers in mlf coordinate action of lateral rectus muscle (n. VI) and medial rectus muscle (n. III. Damage to mlf (e.g., multiple sclerosis) results in disconjugate movements of left and right eyes (internuclear

Discuss the role of the cerebellum in eye movements

-In the lecture on cerebellum, we discussed its role in guiding somatic movements, and mentioned that the accuracy of eye movements is affected by cerebellar lesions. We see here that the cerebellum is a critical part of the circuits for smooth pursuit. A patient with a cerebellar lesion will have difficulty tracking an object. Recall that fixation is also an active neural process. The patients will have difficulty maintaining fixation; their eyes tend to wander. As you might predict, saccades often overshoot or undershoot the target (recall dysmetria?), requiring additional corrective saccades. When smooth pursuit is effectively eliminated by a cerebellar lesion, the patient is left with only inaccurate saccades to track moving targets. Finally, damage to the vestibular parts of the cerebellum can lead to spontaneous nystagmus. -From the types of deficits that follow damage, we can infer that the cerebellum normally makes important contributions to many aspects of eye movements. Indeed, all of the major types of eye movements -- smooth pursuit, saccades, optokinetic, vergent and vestibulo-ocular movements -- rely on the cerebellum for accuracy in amplitude and timing.

Describe the pathway of ALS fibers in the internal capsule

-In the posterior limb of the internal capsule, the fibers associated with the ALS travel along with those of the dorsal column/medial lemniscus system (we are focusing here on the fibers from the VPL, which go to the postcentral gyrus and account for localization of pain). These fibers, then, show the same topography in the internal capsule as do the DC/ML fibers. -A "capsular stroke", which can lesion just a small portion of the internal capsule, can eliminate the sensory fibers associated with part of the body, leaving other fibers intact. Knowing the topography can help localize the lesion. Of course, the information that reaches the postcentral gyrus is also organized topographically; recall the sensory homunculus described above. A lesion of the "arm" region of the postcentral gyrus can reduce a patient's ability to localize a painful stimulus affecting that arm (this is discussed in more detail below). **Note that the present discussion applies to the spinothalamic portion of the ALS, which is responsible for precise localization of pain and temperature. Topography and localization are not so precise in the spinoreticular and spinomesencephalic pathways.**

Describe the transduction of light energy in rods

-Light HYPER-POLARIZES photoreceptor 1. Arrangement of elements in rod; photopigments on membrane--bound discs in outer segment 2. Resting (dark) state: partial depolarization (-40 mV) maintained by cGMP- activated Na+ channels, elevated release of neurotransmitter (glutamate) 3. Absorption of light energy leads to G-protein-coupled processes, closure of Na+ channels, thus hyperpolarization 4. Hyperpolarization suppresses glutamate release (onto bipolar cells) 5. Rods operate at low-to-moderate light levels 6. Huge signal amplification: Single photopigment absorbs photon-->100's of transducer molecules-->~6 cGMP molecules-->closes ~200 ion channels (2% of open channels)

What muscles does CN III innervate?

-Medial Rectus -Superior Rectus -Inferior Rectus -Inferior Oblique

6. Name the vestibulospinal tracts. State whether each tract is associated with medial or lateral descending motor systems. Which vestibular organs and nuclei "drive" the pathways? Describe their patterns of termination in the spinal cord.

-Medial and Lateral vestibulospinal tracts -Medial vestibulospinal tr: from medial vestibular nu., driven by semicircular canals, terminates in cervical spinal cord (medial ventral horns) to control head position. -Lateral vestibulospinal tr: from lateral vestibular nu., driven by otolith organs, terminates in medial part of ventral horn throughout length of cord to control axial and proximal limb muscles, especially extensors (i.e., antigravity).

Describe the mediation of vascular dilation during the sexual response

-Mediation of vascular dilation to cause erection (penis or clitoris) -Parasympathetic activation dilates vessels and relaxes the smooth muscles of the venous sinusoids [nitric oxide as signaling molecule]; sinusoidal spaces expand and blood and pressure increases. This is the erection. Note: there is some sympathetic contribution to these stages.

Where are nociceptors found? How does location dictate function?

-Nociceptors are found in the skin, muscles, joints and visceral organs, and mediate pain in each of these regions. -Nociceptors in joints mediate joint pain due to conditions such as arthritis, for example, while pain receptors in the heart may be activated by ischemia (Haines, Table 18.2, Page 283). The density of free nerve endings at the peripheral end of a nociceptive neuron can vary in different regions of the body and contribute to different pain intensities.

Describe olfactory signal transduction

-Olfactory signals are transduced through G-protein coupled receptors. Each olfactory receptor type is linked to a G protein-coupled receptor. Binding of an odorant molecule to the receptor causes the activation of a G-protein coupled receptor. -This interaction causes the release of the G-protein's GTP-coupled A-subunit, G-olf, which stimulates adenylyl cyclase to produce cAMP. -Increased cAMP levels cause the opening of cyclic- nucleotide gated cation channels, which causes membrane depolarization and action potentials in the sensory axon, and transmission of signals to the olfactory bulb.

8. Contrast eye movement deficits following VI nerve damage versus PPRF damage.

-PPRF lesion: neither eye moves toward side of lesion. At rest, eyes likely deviated away from lesion. -VI nerve lesion: ipsilateral eye deviated medially; unable to abduct.

Describe visceral pain

-Painful signals that arise in the visceral organs of the pelvis, abdomen etc, enter the CNS via a visceral pain pathway in the dorsal column-medial lemniscal system. -Primary visceral afferents from the pelvic and abdominal viscera enter the spinal cord and synapse on second-order neurons in the dorsal horn of the lumbar-sacral spinal cord. Some of these second-order neurons give rise to the anterolateral systems that contribute to visceral pain. Other neurons synapse upon neurons in the intermediate gray region of the spinal cord near the central canal. These neurons send axons not through the anterolateral white matter (as might be expected for the pain pathway), but THROUGH DORSAL COLUMN NEAR THE MIDLINE. These second order neurons then synapse in the dorsal column nuclei of the caudal medulla, where neurons give rise to arcuate fibers that form the contralateral medial lemniscus that eventually synapse on thalamocortical projection neurons in the ventral-posterior thalamus.

Describe the Projections out of somatosensory cortex

-Primary somatosensory cortex encompasses Brodmann's areas 3a, 3b, 1 and 2. (There are differences between the areas, but we will not go into them.) Information obtained by the primary somatosensory cortex is sent to other cortical areas: e.g., to posterior parietal cortex for integration with other senses and the building of an internal representation of the body and the outside world; to motor cortex for the execution of voluntary movements. -Cortical cells in primary somatosensory cortex also project to subcortical levels (e.g., spinal cord dorsal horn, dorsal column nuclei, VPL). This descending pathway can modify the processing and flow of sensory information.

Describe the effects of differing lesions on the pupillary light reflex

-Pupillary light reflex pathways. If the optic nerve is partially damaged (A), shining a light into that eye will produce a diminished direct and consensual response (left); but both will be present when the undamaged side is illuminated (right). This is termed a relative afferent pupillary defect. -A total lesion at A would produce a blind eye, which would induce neither a direct nor a consensual response when illuminated. -If the lesion occurs in the optic tract (B) or pretectum, neither response is lost. Although the reflexes may be weaker, this is not easily discerned clinically. However, a large lesion in the posterior (dorsal) midbrain (e.g., pinealoma) would weaken pupillary responses bilaterally. -If the lesion occurs in the oculomotor nucleus or nerve (C), both direct and consensual responses will be lost in the eye on the lesion side, but they will be present in the other eye.

Describe Meissner Afferents

-Rapidly adapting fibers; innervate skin more densely than Merkel afferents, account for ~40% of mechanosensory receptors in the hand. Enriched in tips of dermal papillae adjacent to the primary ridges and closest to the skin surface. -They are elongated receptors, and formed by a connective tissue capsule comprising lamellae of Schwann cells. -Meissner afferents are much more sensitive than Merkels to skin deformation; however their receptive fields are larger than Merkels so they transmit signals with less spatial resolution.

Describe the make-up of the cerebellum

-The cerebellum includes a cortex on the surface, white matter beneath the cerebellar cortex, and deep nuclei within the white matter. -The deep nuclei are the major source of output from the cerebellum. There are 4 on each side; from medial to lateral, they are the fastigial, globose, emboliform, dentate nuclei. The globose and emboliform are sometimes considered together as the interposed nuclei.

Describe how to determine whether a somatosensation deficit is due to peripheral nerve or CNS damage.

-Recall that a dermatome is the area of skin supplied by a single dorsal root. Dermatomes overlap, so damage to a single dorsal root causes a rather small area of sensory loss. Peripheral nerves generally contain sensory fibers associated with more than one dorsal root. Careful mapping of a deficit will allow you to determine whether the deficit matches the distribution of a peripheral nerve, or the distribution of dermatomes; the former indicates peripheral nerve damage, whereas the latter indicates damage to the dorsal roots or pathways within the CNS. -Of course, the real world is more complicated than just peripheral nerve versus CNS. For example, in peripheral neuropathy, the longest axons are the most susceptible to damage and so the deficits are associated with the distal limbs. This is referred to as the "stocking and glove" effect.

What are sensory receptors often associated with in order to carry out their functions?

-Receptors are often associated with non-neural structures that help to make the receptor cell responsive to particular sensory stimuli (e.g, Fig. 2). Many pathological states alter function of the support structures, not the receptor cells. -This applies to both specialized sensory receptors and modified neuron receptors.

Describe how the cornea and the lens bend light rays to focus an image on the retina

-Refractive power is the degree to which an object (cornea or lens) can bend light rays, and is expressed in diopters. -Most refractive power is in the cornea, but the shape of the cornea is fixed, while the lens can change its refractive power. -Curvature of the lens alters bending of light rays: under parasympathetic control. 1. Relaxed ciliary muscle, lens has less curvature. 2. With contracted ciliary muscle, lens has more curvature. (Contraction relaxes the zonule fibers which normally pull the lens taut) 3. Activity of ciliary muscle of eye is controlled by Edinger-Westphal nucleus in midbrain (part of cranial nerve III nuclear group), projecting to neurons of ciliary ganglion, projecting to ciliary muscle.

Describe how GnRH secretion can change throughout the life of a female.

-Sex steroids generally have a negative feedback effect on secretion of GnRH and gonadotropins 1. Ovulatory cycle: during later stages of follicle development, high levels of sex steroids have a different, positive effect, triggering the LH surge that results in ovulation. 2. Pregnancy: Increasing levels of sex steroids during pregnancy reduce GnRH output from hypothalamus, although GnRH appears to be produced by the placenta. 3. Nursing: For women who nurse an infant, sensory or hormonal feedback to the hypothalamus delays the cyclic GnRH secretion that is necessary for the resumption of regular ovulatory cycles. 4. Menopause: As a woman nears menopause, GnRH release from the hypothalamus increases, because estrogen and progesterone production is reduced and has less negative feedback effect on the hypothalamus and pituitary. It is the decline in sex steroid production that is responsible for many of the physiological effects of menopause.

What are the two main categories of pain perception?

-Sharp first pain and dull second pain. -A noxious stimulus first activates the large diameter A-Delta fibers, giving rise to a sharp first pain. If the stimulus intensity is increased, then the unmyelinated smaller diameter C- fibers are activated, giving rise to a slower, more delayed second pain (panel A below). If the A-Delta or C fibers are selectively anesthetized, a stimulus will cause either a sharp pain or a dull pain, but not both (panels B and C below).

Describe smooth muscle contraction during the sexual response

-Smooth muscle contraction of vas deferens during ejaculation or rhythmic vaginal contractions during orgasm in females -Sympathetic activation -Contractions of the perineal muscle that accompanies orgasm in both males and females -Sympathetic activation

Describe the alternate mechanism that they eye contains for measuring ambient light

-Some ganglion cells are photoreceptive. They contain the pigment melanopsin (which absorbs light in the blue range) and use a G-protein coupled phototransduction process. Their response to light is sluggish compared to rods and cones; they measure overall light intensity. Increased light levels are depolarizing, causing increased numbers of action potentials in the ganglion cell. -These ganglion cells function in pupillary light reflex, circadian rhythms, and probably conscious perception. Explains why circadian rhythms persist when form vision is lost due to rod/cone degeneration.

What is spasticity?

-Spasticity is an increase in muscle tone and an increase in stretch reflexes (often called "deep tendon reflexes"). The combination can make the limbs quite stiff. Because the patient is also paralyzed (or weak, if paralysis is incomplete), this condition is often called spastic paralysis, which contrasts with the flaccid paralysis associated with damage to lower motor neurons. -An interesting phenomenon is sometimes seen when a spastic limb is passively flexed. After a brief period of applied force, the resistance suddenly collapses (the way the blade of a pocket knife, or clasp-knife, closes). This is probably due to strong drive of the Golgi tendon organs, which fire more as the muscle is stretched, and which lead to inhibition of the motor neurons that innervate the muscle being stretched. You may see descriptions of a patient that includes "clasp-knife" reflex; "clasp-knife" is an older term for "pocket knife".

What are some of the "other" upper motor neuron areas?

-Superior colliculus -Red Nucleus (and rubrospinal tract)

Where are sweet, umami, and salty receptors located on the tongue?

-Sweet, umami, salty: tip of tongue. -Behaviors activated: mouth movements, salivary secretion, insulin release, swallowing.

What cranial nerves are attached to each taste receptor? What tastes to each nerve respond best to?

-Taste receptor cells are innervated by cranial nerves VII, IX and X. 1. The chorda tympani branch of the facial nerve responds best to NaCl and sucrose (that is, for a given concentration a greater number of action potentials and axons are activated by these substances) 2. The glossopharyngeal nerve responds best to acid and quinine 3. The superior laryngeal branch of the vagus nerve responds best to acid and water. -These observations imply that a single peripheral neuron might respond best to sucrose, for example, but could also be activated by fructose, NaCl, and acetic acid. The information contained in the patterns of action potentials elicited by each of these tastants arises from the activation of the several taste cells that synapse with the primary afferent neurons. This peripheral convergence has the effect of increasing the sensitivity of the taste system to a given stimulus.

What is the purpose of having both the direct and indirect pathways in the basal ganglia? Describe its center surround organization.

-The direct pathway activates an intended motor program while the indirect pathway suppresses competing motor programs. -Normal function requires proper balance between direct and indirect pathways.

Describe the direct pathway of the basal ganglia

-The direct pathway goes from striatum to GPi (see diagram). This pathway uses GABA, which inhibits another GABAergic projection (GPi to VA/VL). This sets up disinhibition, whereby cortical activity excites the direct pathway, which leads to a reduction in GPi activity, which releases the VA/VL from inhibition. Thus, activity in the direct pathway leads to increased motor cortex activity and increased movement. -Direct pathway inhibits BG output, so increases movement

Compare and contrast the direct and indirect pathways of the basal ganglia

-The direct pathway increases movements, whereas the indirect pathway decreases movements. Normal motor behavior requires a balance between the direct and indirect pathways. -Direct : drives movement -Indirect : inhibits movement

Describe the dorsal spinocerebellar tract

-The dorsal spinocerebellar tract carries information from the trunk and leg. -The second order cells are located in Clarke's nucleus (also known as nucleus dorsalis or dorsal nucleus of Clarke), which is present in the spinal cord from C8 to L2. Between C8 and L2, primary afferents enter the cord and synapse with a second order neuron in Clarke's nucleus. -Primary afferents that enter the cord below L2 must ascend in the fasciculus gracilis until Clarke's nucleus is reached. The axons from the second order neuron in Clarke's nucleus form the dorsal spinocerebellar tract. -These axons enter the cerebellum via the inferior cerebellar peduncle.

Describe the regulation of prolactin secretion

-The hypothalamus is the primary regulator of prolactin secretion -Prolactin, a hormone of the anterior pituitary has several reproductive functions, including: 1. promotion of development of breast tissue for lactation 2. promotion of milk synthesis in alveoli of the breast 3. blockade of gonadotropin release during nursing, preventing normal ovulatory cycling and creates a temporary infertility. -Prolactin inhibiting factor (dopamine) is released by hypothalamic neurons (in arcuate nucleus) onto the portal system in the median eminence, reducing prolactin release. -Other hypothalamic releasing factors and hormones affect prolactin release (e.g., oxytocin, TRH) (Fig. 9)

Describe the inhibitory cortical loop

-The inhibitory cortical loop modulates the activity in the deep cerebellar nuclei. Mossy fibers and climbing fibers are the inputs to cerebellar cortex. Climbing fibers contact the Purkinje cells directly. Mossy fibers contact granule cells in synaptic zones called glomeruli. The granule cells then contact Purkinje cells. -Thus, output of cerebellar cortex, which arises from Purkinje cells, depends on both mossy fiber and climbing fiber inputs. -The three remaining cell types (Golgi, basket, and stellate) are inhibitory interneurons that alter granule cell or Purkinje cell activity. The Purkinje cells form the output from cerebellar cortex. They are inhibitory, and their primary targets are the deep cerebellar nuclei and vestibular nuclei.

Describe the two types of afferent axons in muscle spindles

-The muscle is innervated by Group Ia and Group II afferent axons. -The Group Ia afferents are myelinated axons with the largest diameter, they adapt quickly during changes in muscle length, and provide information about lib dynamics - velocity and direction of movement. -The Group II afferents produce sustained responses to constant muscle length and provide information about the static position of the limb.

Describe the broad organization of olfactory epithelium

-The olfactory epithelium is organized into a few large zones. Each zone contains receptor cells that express a unique subset of receptor genes. Three different groups of genes are mapped in this figure. Each group has a different zone of distribution and does not overlap with any other group. -Within each zone of receptor sub-type, individual receptor cells are scattered randomly. In the figure below, each receptor cell with a unique receptor protein is color coded.

Describe the peripheral process of an olfactory cell. How does it interact with chemicals?

-The peripheral process of an olfactory receptor cell is short, extending from the cell body to the surface of the olfactory epithelium, where it gives rise to cilia. The cilia are bathed in a layer of aqueous mucus that is produced by Bowman's glands and the supporting cells. -Odor molecules must dissolve in the mucus layer and diffuse to receptors on the microvilli. Hydrophilic (water-soluble) molecules can dissolve and diffuse in the mucus directly. Hydrophobic molecules do not easily dissolve in the mucus. The mucus contains a large number of odorant-binding proteins that can transport hydrophobic molecules through the mucus layer. The odorant molecule can then bind with receptors on the olfactory receptor cell cilia and set up a signal that is conducted to the brain by the central process of the olfactory receptor cells.

Describe the somatotopic organization of the motor cortex

-The primary motor cortex and the fibers of the pyramidal tract are arranged somatotopically. Notice that, roughly speaking, the fibers associated with the leg are located posteriorly in the posterior limb of the internal capsule and fibers for the arm are located anteriorly (near the genu) fibers for the face are in the genu. Recall that the posterior limb of the internal capsule also contains somatosensory fibers (projections from VPM and VPL to cortex). -Consequently, even a relatively small "capsular" stroke (the most common type of intracerebral stroke) can decrease or eliminate both sensory and motor function. (Will that region be ipsilateral or contralateral to the lesion?). In the brainstem and spinal cord, "leg" fibers are located laterally and "arm" fibers medially (details are different in the anterior corticospinal; the differences are not relevant clinically and we will not discuss them).

Describe the forms of immediate-onset transcription-independent central sensitization (CS) in dorsal horn neurons

-There are three forms of immediate-onset transcription-independent central sensitization (CS) in dorsal horn neurons: (i) windup, which is homosynaptic and manifests only during the trains of stimuli that elicit it (arrows). (ii) activity-dependent classical central sensitization that outlasts the initiating stimulus and is predominantly heterosynaptic - low-threshold Aβ inputs normally elicit no response but begin to do so after the C-fiber conditioning input. (iii) An LTP-like enhancement of excitatory postsynaptic potentials (EPSPs), the induction phase of which is transcription- independent. This form of central sensitization is largely homosynaptic.

Describe the forms of late-onset transcription-dependent central sensitization

-There are two forms of late-onset transcription-dependent central sensitization: (i) an activity- dependent localized form, which can include late-phase LTP (ii) an activity- independent widespread form, both of which take hours to manifest and last for prolonged periods. -Two genes upregulated by activity in localized areas of the spinal cord are those encoding dynorphin (Dyn) and the neurokinin 1 (NK1) receptor, while the cytokine interleukin 1β (IL1β) produces a widespread induction of cyclooxygenase 2 (COX-2) in many areas of the CNS.

Why do somatosensory afferents differ in terms of response properties?

-These differences in response properties can arise from differences in the afferents themselves, or from differences in characteristics of peripheral receptor cells. -The largest diameter sensory afferents (Ia) supply the sensory receptors in muscles. Smaller diameter fibers (A) convey information about touch/tactile system. -The smallest diameter fibers (A, C) supply information about pain and temperature.

What are the major vestibulo-spinal pathways? What are their function?

-Vestibular reflex pathways: control of posture, head and eye position. 1. Lateral vestibulo-spinal pathway 2. Medial vestibulo-spinal pathway

Describe the central taste pathways as information is relayed from the rostral solitary nucleus to the forebrain

-Within the thalamus, taste information is processed in a subdivision of the ventral posterior medial nucleus distinguished by small cells. This taste subdivision is therefore called the parvicellular part of the VPM (VPMpc. "Parvicellular" means "small- cell"). The remainder of the VPM takes care of the remaining general sensory information from the head (primarily from the trigeminal system). -The primary taste area in cortex is located near the representation of the tongue in somatosensory cortex (which is represented in the inferior somatosensory cortex in the postcentral gyrus) and extends into the frontal operculum and the anterior insula. "Higher order" cortical areas are located in orbitofrontal cortex that integrate gustatory, olfactory, visual and somatosensory inputs to give rise to the appreciation of flavor and food reward. -Some of this information ultimately reaches the hypothalamus and the amygdala (both of which help control feeding behavior).

What are the motions of horizontal movement?

-adduction (toward the nose) -abduction (away from the nose)

What are the motions of vertical movement?

-elevation -depression

What are the motions of rotational movement?

-intorsion (top of eye toward the nose) -extorsion (top of eye away from the nose)

Match the pairs of semicircular canals

-left and right horizontal canals -left anterior and right posterior canals -right anterior and left posterior canals

Describe Benign Paroxysmal Positional Vertigo (BPPV)

-vertigo and nystagmus for brief periods, often recurrent, often after sleeping -cause: otoconia become lodged in semicircular canal

Describe the method of transduction of somatosensory afferents

1. A stimulus alters the permeability of cation channels in the afferent nerve endings, generating a depolarizing current known as a receptor (or generator potential). 2. If the receptor potential is large enough, it reaches the threshold for action potential generation in the afferent fiber. The rate of action potential firing is roughly proportional to the magnitude of the depolarization, which is proportional to the magnitude of the stimulus (i.e. the intensity of pressure, pain etc). 3. At the transduction site of the sensory neuron, the receptor potential is graded with stimulus magnitude. At the trigger zone in the axon, all-or-none action potentials are generated, riding on top of the receptor potential. 4. Further along the axon, receptor potentials are not observed, but the action potentials remain, travel to the nerve terminals of the sensory neuron, which are located in the spinal cord or other regions of the CNS, where they result in transmitter release. 5. The larger the sensory stimulus, the larger the magnitude of the receptor potential, the higher the frequency and number of action potentials, and the greater the amount of transmitter release. Thus, by way of changing the amount of transmitter released from the sensory neuron nerve terminal, information about the duration and magnitude of the sensory stimulus (i.e. stretch, pressure, pain etc) is conveyed to the next neuron, which lies in the CNS.

Describe how the autonomic nervous system responds to a rise in blood pressure

1. Activate baroreceptors that inhibit tonic activation of sympathetic preganglionic neurons in spinal cord. Baroreceptor input (re: increased pressure) stimulates activity of parasympathetic preganglionic neurons in nucleus ambiguus and dorsal motor nucleus of vagus that influence heart rate. 2. Reduction in stimulatory noradrenergic sympathetic innervation on cardiac pacemaker and cardiac muscle 3. Decreased output of catecholamines from adrenal medulla 4. Decreased vasoconstrictive effects on sympathetic innervation of peripheral blood vessels 5. Activation of parasympathetic heart innervation (via extensive ganglia in and near heart) decreases cardiac pacemaker discharge 6. Heart rate reduced via lowered atrial-ventricular contraction, peripheral arteriole dilation

Describe the activity of A-Gamma fibers

1. Activity of A-gamma fibers and contraction of intrafusal muscle fibers regulates the sensitivity of muscle spindle afferents to muscle length/stretch. 2. More A-gamma activity >> more contraction of intrafusal fibers >> greater sensitivity of spindle afferents to muscle stretch 3. A-gamma motorneurons help to regulate the tone of muscles by regulating the sensitivity of the stretch reflex 4. A-gamma motorneurons maintain sensitivity of stretch reflex during muscle contraction (shortening) 5. Activity of A-gamma motorneurons is modulated by higher centers.

Describe the monosynaptic reflex arc

1. Also referred to as stretch, deep tendon or myotatic reflex. 2. Basis of knee, ankle, jaw, biceps and triceps response tested in routine examination. 3. The simple reflex arc consists of the afferent sensory neuron, the local circuit neuron and the two motor neurons. The reflex does not require higher brain control to work, but since the local circuit interneuron can receive descending inputs from higher brain regions, reflexes can be modulated.

Describe the mechanism of ocular dominance

1. Area 17 neurons are activated more strongly by stimuli in one eye or the other eye (= ocular dominance). 2. Area 17 is organized into adjacent groups of cells that respond best to one or the other eye. These are called ocular dominance columns, because they are composed of groups of cells extending from the cortical surface to the deepest cortical layers. Adjacent ocular dominance columns look at the same part of the visual field, but with different eyes. 3. Inputs from eye-specific layers of LGN remain segregated when they enter visual cortex layer 4. Within cortex, signals from the two eyes converge within other layers (Fig. 18A). Outside layer 4, the relative strength of input from the two eyes varies in a columnar fashion, with monocular responses over the centers of ocular-dominance columns, and binocular responses at the borders between columns. (Fig. 18) 4. The information from the two eyes, looking at the same part of the visual field, is compared to produce binocular depth perception.

Describe the vestibular pathway to the cerebral cortex

1. Ascending projection forms basis of conscious perception of vestibular stimuli. 2. Pathway: Lateral, and superior vestibular nuclei ascend bilaterally outside of mlf to the ventroposterior nucleus and posterior nuclear group. These ascend to parietal cortex, near somatosensory representation of the face (area 3A) and to posterior parietal cortex (area 5).

What cortical areas beyond the primary auditory cortex are associated with audition?

1. Association auditory cortex includes areas 42 (on transverse temporal gyrus) and 22 (superior temporal gyrus. 2. These areas appear to perform more specialized analyses of sounds, receiving input from primary auditory cortex. -Some association cortex may analyze sound location cues -The posterior part of area 22 in the left cerebral cortex appears specialized for speech processing. Additional speech areas are in the adjacent inferior parietal lobule (areas 39 and 40). These areas in the left cerebral cortex are called Wernicke's area. Occlusion of branches of the middle cerebral artery can damage these areas, impairing language comprehension while leaving analyses of non- speech sounds unaffected.

Describe the systematic representation of the frequency of sounds within the cochlea and each ascending auditory nucleus.

1. Auditory hair cells are responsive to particular frequencies of sound, due to their location on the basilar membrane (Fig. 19). 2. The highly organized connection from the hair cells to auditory nerve fibers to the auditory brainstem nuclei. This connection preserves the cochlear frequency organization throughout the brain.

Describe how Sensory information is communicated to the CNS by the cardiovascular system

1. Baroreceptors in heart and major vessels sense mechanical pressure; activated by deformation 2. Chemoreceptors in carotid bodies located in carotid artery bifurcation and aorta sense levels of oxygen and carbon dioxide 3. CN9 (glossopharyngeal) and CN10 (vagus) carry baroreceptor and chemoreceptor information to NST which relays to hypothalamus and reticular formation 4. Sympathetic innervation to heart is T1 to T5 intermediolateral column 5. Visceral motor neurons for heart are in thoracic paravertebral and prevertebral ganglia of the cardiac plexus 6. Parasympathetic preganglionic innervation for heart are in nucleus ambiguus and dorsal motor nucleus of the vagus

What are the Sensory Input to CNS from Viscera?

1. CN 9 (glossopharyngeal) and CN 10 (vagus) synapse in nucleus of the solitary tract 2. Second-order visceral afferents in the spinal cord 3. Preganglionic neurons of the sympathetic and parasympathetic divisions 4. Dorsal column pain pathway

Briefly describe upper motor neurons

1. Cell bodies located in brainstem centers (superior colliculus, vestibular nuclei, reticular formation) and in cerebral cortex. 2. Axons of upper motor neurons contact local circuit neurons in brainstem and spinal cord, which then contact lower motor neurons.

What key roles does the hypothalamus play?

1. Central control of homeostatic mechanisms 2. Central control of endocrine systems 3. Coordinator of visceral motor function 4. Contributor to complex systems

Besides the motor neurons, what are the other components of the motor control system?

1. Cerebellum: no direct access to lower motor neurons or local circuit neurons. -Cerebellum acts via its efferent pathways to regulate activity of upper motor neurons. 2. Basal ganglia: located in forebrain. Affects upper motor neuron circuits.

Describe the flow of information in the Papez Circuitry

1. Cingulate cortex, receiving inputs from broad areas of cerebral cortex (pre-frontal, pre-motor, sensory association areas) and from anterior nucleus. It provides the major conduit for highly processed information to hippocampus. • It projects via cingulum to: 2. Entorhinal cortex. In addition to input from cingulate cortex, it receives input from widespread areas of cortex. (A part receives input from olfactory bulb) I • It projects to: 3. Hippocampal formation. It forms associations among multiple inputs, primarily from cingulate or entorhinal cortex, through the process of long-term potentiation. •It then projects via fornix to: 4. Mammillary body (of hypothalamus). •Mammillary neurons project via mammillo-thalamic tract to: 5. Anterior nucleus of thalamus. This nucleus provides main gateway by which processing in hippocampal formation reaches cingulate cortex. •It then projects to: 6. Cingulate cortex. The cingulate cortex provides an output to other areas of cerebral cortex that is a comparison of its inputs from both anterior nucleus of thalamus and from widespread neocortical areas.

What hypothalamic areas differ between males and females?

1. Control of ovulatory cycles in females -Anteroventral paraventricular nucleus (you don't need to remember) regulates GnRH and Prolactin Inhibiting Factor release, controlling ovulatory cycles. -The nucleus is larger in females than in males. Transiently high levels of testosterone in male fetuses cause cell death. 2. A part of the preoptic area contributes to male copulatory behavior. The nucleus is largely absent in females. During fetal development, cell death occurs in the absence of testosterone. 3. Overall, many elements of sexual/copulatory behavior and responses in both males and females are controlled by the preoptic area of the hypothalamus.

Generally describe the autonomic nervous system functional network

1. Coordinated Activity -Parasympathetic activity increases metabolic and other resources when circumstances allow rest and digestion. This includes pupil constriction, slowed heart rate, increased peristalsis, bladder voiding. Other signs of rest, including dilation of blood vessels in the skin and gut, and relaxation of piloerector muscles, are possible through diminished activity in the sympathetic system as opposed to activity in the parasympathetic. 2. Sympathetic innervation only: adrenal medulla, sweat glands, piloerector muscles, arterial blood vessels.

What are two types of spastic rigidity?

1. Decerebrate (Upper Pontine Damage)--Hands at sides 2. Decorticate (Upper Midbrain Damage)--Hands on chest -Patients are usually unconscious due to damage of ARAS

What makes up the basal ganglia?

1. Dorsal Basal Ganglia -Putamen -Globus Pallidus -Caudate Nucleus 2. Ventral Striatum -Nucleus Accumbens 3. Ventral Pallidum -Substantia innominata

What were the early views of satiety control?

1. Early view related to lateral and medial hypothalamic syndromes a. Feeding and satiety centers b. Hypothalamic syndromes associated with lesions of lateral area (diminished feeding) and ventromedial area (excessive feeding)

Describe the structure and innervation of the Golgi Tendon Organ

1. Encapsulated afferent nerve ending. 2. Located at the junction of the muscle and a tendon. 3. Each tendon organ is innervated by a single Group Ib sensory axon. 4. Arranged in series with extrafusal muscle fibers.

Describe the mechanism behind fever

1. Endogenous or exogenous substances (pyrogens) reach endothelial cells of hypothalamic capillary beds , stimulating production of cyclooxygenase-2 (COX-2), which induces prostaglandin E2. This compound increases the "thermostatic set point" of hypothalamic neurons 2. This results in activation of mechanisms associated with heat gain, including vasoconstriction, increased heart rate, shivering, and behavioral adjustments. 3. Fever is a controlled increased in internal temperature that can establish an unfavorable environment for infectious agents 4. Anti-pyrogens block the "set-point" adjustment by pyrogens, leading to lowered temperature. Given the adaptive nature of fever, is this always desirable?

Describe the neurons of the crossed-extensor reflex

1. Excitatory interneurons terminate on extensor A-alpha motorneurons. These activate extensor muscles to support the body. 2. Inhibitory interneurons terminate onto alpha motorneurons that supply the contralateral flexor muscles. The result is to suppress contraction in the flexor muscles that would oppose the action of the crossed extensor muscles.

Describe the anatomy of the primary visual cortex

1. Extends from the occipital pole (representing foveal visual field) along superior and inferior banks of the calcarine fissure on medial part of cerebral hemisphere (representing more lateral parts of visual field). 2. Distinctive in fiber-stained material due to heavy myelinated optic radiation fibers in Layer IV. Called stripe of Gennari. Due to these stripes (or striations), visual cortex is also called striate cortex. 3. Pattern of termination of geniculo-striate neurons -Neurons from M- and P-type LGN layers project to different layers and sublayers in visual cortex (maintains segregation of M- and P- pathways) (Fig. 16). -LGN neurons responding to inputs from one eye terminate in patches that are mostly separate from terminations of LGN neurons responding to inputs from other eye. Different parts of visual cortex are thus dominated by inputs from one or the other eye (Fig. 18).

Describe the roles and features of Gamma Motor Neurons

1. G motor neurons are smaller than A motor neurons. 2. G motor neurons control muscle spindles and modulate their excitability. 3. G motor neurons terminate on intrafusal fibers (of the spindle), and actgivation of y fibers causes intrafusal contraction. This contraction maintains tension on the middle of the intrafusal fibers, which is where the sensory axons terminate.

What is the function of the Golgi Tendon Organ?

1. GTOs are very sensitive to increase in muscle tension that arises from muscle contraction. Unlike spindles, GTOs are relatively insensitive to passive stretch. 2. The Ib sensory axons from GTOs contact inhibitory local circuit neurons in the spinal cord. These inhibitory local circuit neurons synapse on A-motor neurons that innervate the same muscle. 3. Therefore, the Golgi tendon circuit is a negative feedback system that regulates muscle tension. It decreases the activation of a muscle when very large forces are generated, and so protects the muscle. 4. This is called the inverse myotatic reflex - discussed later.

What causes sensorineural hearing loss?

1. Hair cell loss due to genetics, abuse, age, disease, ototoxic substances 2. Ménière's disease 3. Acoustic neuroma (=vestibular schwannoma) 4. The cochlea and vestibular structures of the inner ear and VIIIth nerve are often involved in pathological processes together, e.g., Ménière's disease, ototoxicity, acoustic neuroma

What are some main vestibular diseases?

1. Hair cell loss due to toxic substances: amino glycoside antibiotics 2. Benign Paroxysmal Positional Vertigo (BPPV) 3. Meniere's disease 4. Migraine cause severe vertigo and nystagmus. A link has been proposed between migraine and Meniere's 5. Occlusion of PICA in lateral medullary (Wallenberg) syndrome can cause vertigo and nystagmus 6. Vestibular neuritis

Describe otolith function

1. Hair cells in the utricle are arranged in a horizontal plane within the macula, but orientation and directional sensitivity differs (Fig. 6). The utricular macula thus converts head tilt or linear acceleration into selective activation of some of these hair cells. (Fig. 7) 2. Functional role: Utricle senses linear acceleration and position with respect to gravity. 3. Saccule is arranged roughly vertically. It also analyzes head tilt and linear acceleration

What are the general characteristics of the rod system?

1. High sensitivity to light -Lots of photopigment -Large signal amplification due to a single photon -High convergence onto ganglion cells 2. Operates best at low light levels -Greater sensitivity (activated by a single photon) -Saturation (cannot respond at moderate to high light levels) 3. Slow temporal response: Cannot distinguish between events that occur very closely spaced in time 4. Low acuity, not present in fovea 5. Highly convergent pathways 6. ACHROMATIC (no color contrast)

What are the three axes of eye muscle movement?

1. Horizontal 2. Vertical 3. Rotational

Describe sensory receptors that are sensory neurons with modified nerve endings

1. Individual neurons have specialized structures that sense particular types of stimulus energy 2. Examples: nociceptors, touch receptors, temperature receptors, olfactory receptors, photoreceptors

Give a general schema for hypothalamic control pathways

1. Information is received by humoral and neural routes 2. Connections within the hypothalamus distribute processing to many neurons 3. Outputs are to endocrine pathways and to neural pathways mediating both conscious and unconscious behaviors.

Describe Parallel processing in secondary and association visual cortices

1. Information received in primary visual cortex is processed, then sent to secondary visual cortex (Area 18). M and P pathways remain segregated in V2. 2. M and P channels diverge after V2

Describe how the autonomic nervous system responds to a fall in blood pressure

1. Inhibit parasympathetic activity and increase sympathetic 2. Sympathetic postganglionic terminals release NE, increasing cardiac pacemaker activity and cardiac contractility 3. Adrenal medulla releases NE 4. Sympathetic postganglionic NE release on arteriole smooth muscle increases vascular tone, shunting blood away from the periphery and to heart, brain and kidneys

What are the important characteristics of skeletal muscle contraction?

1. Initiated by "lower" motor neurons in spinal cord and brainstem. 2. Cell bodies of lower motor neurons (LMN) located in ventral horn of spinal cord and cranial nerves in the brainstem. 3. LMNs - also called alpha (A) motor neurons. 4. A motor neurons send axons directly to skeletal muscle through ventral roots of spinal cord, spinal peripheral nerves and via cranial nerves.

Describe the innervation of the facial muscles in regards to the innervation of the left side of the face

1. Left side of face: upper and lower parts receive separate innervation via CN VII 2. Lesion of CN VII yields paralysis/weakness of whole side of face 3. Lesion of corticobulbar path yields paralysis limited to lower face; upper face on affected side still innervated by intact corticobulbar tract

What are local circuit neurons of the spinal cord?

1. Local circuit neurons - neurons that interconnect lower motor neurons. 2. Connections both across the two sides of the spinal cord (commissural axons), as well as along its length, i.e. across spinal cord segments. 3. E.g. local circuit neurons coordinate rhythmic movements of upper and lower limbs. 4. Local circuits neurons ensure connections between the two sides of the body - ensure that groups of axial muscles on the two sides of the body act in concert to maintain and adjust posture etc. 5. Somatotopic arrangement of local circuit neurons; e.g. local circuit neurons that supply the medial region of the ventral horn are situated medially within the spinal cord grey matter.

What are the general characteristics of the cone system?

1. Lower sensitivity to light -Less photopigment -Less of a signal amplification to a single photon -Low convergence onto ganglion cells 2. Operates best at high light levels -Lower sensitivity (need >100 photons for activation) -Lack of saturation at moderate to high light levels 3. Fast temporal response: able to distinguish between closely spaced events in time 4. High acuity, located in fovea 5. Less convergent pathways 6. Achromatic OR Chromatic, 3 types of cones which are sensitive to different parts of the visible spectrum

Summarize the role of the hypothalamus in reproduction

1. Many parts of the hypothalamus contribute to reproductive function 2. Inputs to hypothalamus include hormonal influences, sensory information, input from other hypothalamic centers and other parts of the brain 3. Outputs include neuroendocrine output, autonomic output, and projections to other brain centers 4. Hypothalamic function is distinctive in several respects -Function can change radically over the lifespan -Control mechanisms can shift quickly from negative feedback to positive feedback -Many elements of hypothalamic structure and function are sexually dimorphic

Describe the role of the medial vestibulo-spinal pathway

1. Mediates vestibulo-colic reflex: rotation of body activates neck musculature to turn head in direction opposite the rotation of the body. 2. Functional Role: To maintain a consistent position of the head in space during movement of the body 3. Net effect is to stimulate appropriate neck musculature to turn head in direction opposite the rotation of the body (and to inhibit opposing muscles of the neck).

What happens in cases of lesions in the optic chiasm?

1. Most common are lesions of crossing fibers due to tumor of the pituitary or hypothalamus 2. Visual deficit is in the temporal visual fields on both sides. WHY? If loss is complete, termed Bitemporal Heteronymous Hemianopia. 3. Larger lesions can affect uncrossed fibers, resulting in total blindness 4. Usually accompanied by endocrine disorders

Describe the relationship between motor neurons and muscles. How are they spatially related in the body?

1. Orderly relationship between the location of motor neuron pools and the muscles they innervate. 2. This relationship occurs both along the length of the spinal cord and in the medial- lateral direction. 3. Therefore, this arrangement provides a spatial map of the body's musculature. 4. Each lower motor neuron innervates muscle fibers within a single muscle. 5. All the motor neurons innervating a single muscle (the motor neuron pool) are grouped together into a cluster that runs parallel to the long axis of the spinal cord, traversing more than one spinal cord segment. 6. This mapping is referred to as "topography". 7. From the spatial organization of the motor neuron pools in the ventral horn, you can understand how the body's musculature is controlled. 8. E.g. medial lower motor neurons that govern postural control and maintenance of balance - receive input from upper motor neurons through axonal tracts that run in the medial and anterior (ventral) white matter of the spinal cord. The lateral lower motor neuron pools that innervate distal extremities are controlled by projections from cerebral cortex that run through the lateral white matter of the spinal cord.

Describe oxytocin's role during childbirth

1. Oxytocin release is stimulated by cervical and uterine stretching during childbirth. Oxytocin then increases contractions of the uterus to promote childbirth.

Describe the intertwined nature of the vestibular and auditory components of CN VIII. How does this impact pathology?

1. Peripheral components of these systems are in close proximity, and share many anatomical and physiological features. 2. Pathologies in periphery can affect both systems. Proper diagnosis can depend upon examination of function in both systems.

Describe the detailed process of light hitting a photoreceptor

1. Photon absorbed by photopigment 2. 11-cis retinal converts to all-trans retinal 3. Activates intracellular messenger transducin 4. Activates phosphodiesterase (PDE) 5. PDE hydrolyzes cGMP 6. Lowers cGMP concentration 7. Closes Na+/Ca2+ channels 8. Less Na+/Ca2+ flowing in, leads to hyperpolarization 9. Hyperpolarization leads to less neurotransmitter (glutamate) release onto bipolar cell

Describe the pretectum pathway of the brachium of the superior colliculus

1. Pretectum a. Located in the rostro-dorsal midbrain b. Pretectum on one side receives direct input from parts of both retinas viewing contralateral visual field (input from contralateral eye crosses in optic chiasm). c. Functions in pupillary light reflexes and accommodation for near vision.

What are the main features of sensory neurons?

1. Primary sensory neurons convey information into CNS by action potential 2. Receptive Field 3. Inhibitory Mechanisms: Inhibitory processes play important roles in information processing of sensory neurons, especially in the CNS.

Describe the role of outer hair cells in the regulation of frequency tuning by the cochlea

1. Provide very little direct information to central nervous system. 2. Are essential for full sensitivity and frequency tuning of inner hair cells. 3. Outer hair cells are contractile: they shorten with each sound-induced depolarization. Because they are connected to both basilar and tectorial membranes, their contraction and relaxation greatly amplifies the vibration of the basilar membrane. The largest amplification occurs at the point on the basilar membrane where the vibration is largest. 4. Outer hair cells are particularly susceptible to noise damage and to aminoglycoside antibiotics, both of which can cause significant loss of hearing sensitivity and frequency selectivity.

Describe how the iris controls pupillary diameter

1. Pupillary dilation allows more light into the eye, but decreases depth of field. Dilation of pupil caused by contraction of pupillary dilator (radial) muscle of iris, under sympathetic control. 2. Pupillary constriction allows less light into eye, but increases depth of field. Constriction of pupil caused by contraction of iris sphincter muscle, under parasympathetic control.

Describe sensory receptors that are specialized receptor cells

1. Specialized cells that transduce a stimulus 2. Examples: auditory and vestibular hair cells, taste receptors, arterial O2 receptors 3. Release chemical transmitters onto a sensory neuron, which conveys info to CNS

Describe the varying locations of lesions that affect spasticity

1. Spinal cord -Spastic, but can't support weight 2. Midbrain-Diencephalic Junction -Increased spasticity, may be able to support weight 3. Brainstem -Greatest spasticity, with varying patterns -Lesions in the brainstem are particularly dangerous

What are the important aspects of light in relation to vision?

1. Visible light is a form of electro-magnetic radiation that can be sensed by your eyes. -Our eyes can see light having wavelengths in the range 400 - 700 nanometers. Our ability to distinguish colors is based on photopigments that are differentially sensitive to the wavelengths of light. 2. Light can be viewed as having wave-like features (e.g., frequency or wavelength) and particle-like features (behavior of photons). The brightness or intensity of light can be related to the number of photons striking photoreceptors.

Describe the timing of urination

1. Stretch receptors in bladder wall synapse on cells projecting to periaqueductal gray (PAG). Signals sent when urine reaches threshold (eg., 300 to 400mL urine) 2. Hypothalamus, amygdala, forebrain input to PAG [safety and suitability for micturition] 3. Periaqueductal gray activation of pontine micturition center (Barrington's nucleus) 4. Pontine micturition neurons project to parasympathetic preganglionic neurons and inhibitory local circuits; activation increases parasympathetic outflow/decreases sympathetic tone → bladder wall contraction → external sphincter muscle inhibition → voiding

Describe some features of the systematic organization of the auditory system

1. Systematic representation of the frequency of sounds (tonotopic organization) within the cochlea and each of the ascending auditory nuclei through primary auditory cortex (area 41). 2. Representation of contralateral sound field in the inferior colliculus and higher levels. 3. Brainstem inputs converge on inferior colliculus to fully represent sound. 4. Other response properties, such as cues for sound localization, may also be systematically represented.

Describe the general mechanism for hair cell transduction

1. The "resting" state is characterized by inward K+ leakage, outward pumping of K+, and some release of transmitter. 2. Hair cells activated by the deflection or shearing of the hair bundle 3. Shearing in direction of kinocilium causes depolarization. 4. This is basic mechanism of transduction by hair cells in vestibular and auditory systems (with some differences in auditory system).

Describe the afferents of the myotactic reflex

1. The afferent (sensory) neurons that control the stretch reflex have their cell bodies in the dorsal root ganglia, with a peripheral axonal branch that ends in a sensory ending (stretch receptor) in the muscle, and a central axonal branch that ends in the spinal cord. 2. Ia afferents: respond phasically to small stretches. Signal the velocity of stretch. 3. IIa afferents: respond tonically to sustained stretches. Signal the degree of stretch.

Describe how the cochlea distinguishes between frequencies

1. The greatest vibration of the basilar membrane occurs at one position along its length, determined by the frequency of the sound. High frequencies cause the point of maximum deflection to be close to the base. 2. Systematic change in stiffness of basilar membrane is a crucial feature of cochlea's response to sound. The basilar membrane is stiff near its base (near the oval window) and becomes less stiff toward the apex. As a result, high frequency sounds cause maximum vibration at the stiffer, basal end of the basilar membrane. Low frequencies cause maximum vibration at the floppier, apical end. 3. Due to this frequency sensitivity of the basilar membrane response, hair cells nearer the base respond maximally to high frequency sounds, while hair cells nearer the apex respond maximally to lower frequency sounds. The basilar membrane is tonotopically organized.

Describe the main categories of Modification of light by the eye

1. The optical image is inverted by path through the eye (left-right and up-down). 2. The cornea and the lens bend light rays to focus image on the retina. 3. The iris controls pupillary diameter. 4. Positioning/movement of eyes by eye muscles 5. Image focusing

What are some functional notes to keep in mind about the vestibulo-ocular reflex?

1. The reflex consists of movement of the eyes opposite to the direction of head rotation. 2. They occur with head turning in all directions and involve the functions of all three sets of semicircular canals. 3. This reflex is designed to maintain stability of the visual field. 4. VOR circuitry can adapt to changes in vestibular input. 5. The reflex depends on integrity of vestibular nuclei, flocculonodular lobe of cerebellum, and mlf. 6. Can be overridden by higher centers.

Describe the circuits responsible for horizontal saccades

1. The right PPRF causes rightward eye movements, via projections to the right abducens nucleus (to activate the right lateral rectus) and the left oculomotor nucleus (to activate the medial rectus). Leftward movements are handled by the left PPRF. 2. There are inhibitory "internuclear" neurons that inhibit the antagonistic muscles (shown in dotted lines). 3. The axons that extend between the oculomotor nucleus rostrally and the PPRF and abducens nucleus caudally travel in a pathway called the medial longitudinal fasciculus (MLF). The MLF is located along the midline in the dorsal tegmentum, in a position similar to the visual motor nuclei (i.e., III, IV, and VI).

Describe the feedforward and feedback mechanisms of muscle action

1. There is a central command for limb adjustment 2. Accompanying the central command is a subconscious command for anticipated postural readjustment. This is feedforward *(Cortico-reticulospinal paths (i.e., corticobulbar projections to reticular formation; reticulospinal tracts))* 3. Following muscular action, postural instability as relayed by afferent sensors tells the body to readjust posture in a different way is necessary. This is feedback (Vestibular system and vestibulospinal tracts)

Describe what happens in cases of Lesions of the optic tract or lateral geniculate nucleus

1. There is a loss of vision in the contralateral hemifield for each eye. If total, this is termed Contralateral Homonymous Hemianopia. 2. Macular vision is not spared by total lesions of these structures. 3. Note proximity of optic tract to cerebral peduncles: possible motor involvement 4. Optic tract may be affected by tumor, vascular, or CSF related problems. Blood supply is by anterior choroidal artery. 5. LGN lesions are rare

Describe the effects of central lesions on the auditory system

1. Unilateral lesions of the cochlea, auditory nerve, and cochlear nuclei cause deafness or impaired hearing in one ear. 2. Unilateral lesions above SOC do not result in unilateral deafness because information from both ears ascends on each side of the brain. However, hearing is impaired, particularly the ability to locate and analyze sounds in the contralateral sound field.

What are the receptor organs of the inner ear?

1. Vestibular apparatus 2. Hearing Apparatus (Cochlea)

What is the role of the vestibulo-ocular reflex?

1. Vestibulo-ocular reflex (VOR): head movement in any direction results in eye movements that are equal in angular rate but opposite in the direction of the head movement. 2. Functional Role: The function of the VOR is to stabilize the direction of gaze during movement of the head or body.

Describe symptoms of vestibulocerebellar lesions

1. Vestibulocerebellum - flocculonodular lobe: problems with balance, eye movements; "nystagmus" may be present

Describe the difference between passive stretching and active contraction on a muscle, and its effect on the golgi tendon organ

1. When a muscle is passively stretched, most of the change in length occurs in the muscle fibers since they are more elastic than the tendon. 2. But when a muscle actively contracts, the force acts directly on the tendon. This force increases the tension of the collagen fibrils in the GTO, and compression of the sensory receptors.

Describe the course of the axons of the corticospinal tract

1. corona radiata 2. internal capsule, posterior limb 3. cerebral peduncle (crus cerebri, basis pedunculi) 4. fiber bundles in base of pons 5. Pyramid: a. to pyramidal decussation and lateral corticospinal tract, -OR- b. continuing uncrossed in ventral corticospinal tract

Describe the course of the axons of the corticobulbar tract

1. corona radiata 2. internal capsule, posterior limb 3. cerebral peduncle (crus cerebri, basis pedunculi) 4. fiber bundles in base of pons 5. pyramid -Travel with corticospinal fibers until they are close to level of target, then split off and travel across brainstem to target area.

From medial to lateral what are the cerebellar nuclei?

1. fastigial 2. globose 3. emboliform 4. dentate Globose+Emboliform=Interposed Nuclei

Describe the locations of the gaze centers

A number of "accessory" nuclei are associated with the oculomotor nucleus. These nuclei include the interstitial nucleus of Cajal, the nucleus of Darkschewitsch, and the nuclei of the posterior commissure. These are located in the midbrain close to the oculomotor nuclei. They are involved in many aspects of visual motor function, including various types of eye movements and associated head movements, eyelid control, and possibly the light reflex. As you might expect, their connections are complex; we will not consider them further.

What is a scotoma?

A scotoma refers to a part of the retina with reduced or non-functional vision.

Describe the steps taken for accommodation for near vision

A series of motor responses required to view nearby objects a. Stimuli include retinal disparity (mismatched images on the two retinas) and blurred retinal images (images without sharp contours) b. Pupillary constriction c. Lens accommodation d. Convergence of the eyes, via contraction of medial rectus muscles (external eye muscles)

How many muscle fibers does a single motor neuron innervate?

A single motor neuron innervates several muscle fibers. The motor neuron and the muscle fibers it contacts define the motor unit.

What are the bony and membraneous labyrinth of the inner ear?

A system of bony tubes (the bony labyrinth) and, within it, membranous tubes (the membranous labyrinth) encased in temporal bone. Fluids within these tubes are essential for proper function: 1. Perilymph is located just inside bony labyrinth. It is similar in composition to cerebrospinal fluid (high sodium). 2. Endolymph is located within the membranous labyrinth. -similar in composition to intracellular fluid (high potassium) -created by specialized epithelial cells -drains into venous sinuses through endolymphatic sac

What is plegia?

A term that you will see frequently is "plegia" or, more often, its variants "monoplegia", "hemiplegia" and "quadriplegia". Plegia means paralysis; monoplegia affects one limb; hemiplegia affects one side of the body and quadriplegia affects both sides (all four limbs; hence the "quadri"). "Paraplegia" is paralysis of the lower half of the body. You also will likely see the term "palsy", as in III nerve palsy, referring to paralysis of the muscles associated with the III (oculomotor) nerve. In general, palsy means paralysis, although in popular usage it can have additional connotations.

What is tremor at rest?

A tremor that disappears during intentional movements. Common in Parkinson's disease. IMPORTANT distinction from the intention tremor associated with cerebellar lesions.

What are the actions of FSH and LH in the male?

Actions of the anterior pituitary hormones in males (review) i. LH stimulates Leydig cells to produce testosterone ii. FSH stimulates Sertoli cells to promote spermatiogenesis

Describe the connections and functioning of the circuits for horizontal eye movements to the right. Include the appropriate gaze center and follow the circuits to the relevant muscles.

Activation of right PPRF leads to: 1. activation of right VI nu. and right lateral rectus. Right VI inhibitory neuron inhibits right III nu. via projection in mlf 2. Right PPRF also projects via mlf to left CN III nucleus to activate left medial rectus.

What causes affective disorders?

Affective disorders involve altered function of the amygdala. i. Implicated in post-traumatic stress disorder: through a traumatic event, normally benign sensory stimuli become evoke strong negative emotional responses. ii. Depression involves increased activation of the amygdala and associated regions including the orbitofrontal and medial prefrontal cortex and the mediodorsal nucleus of the thalamus

What is Huntington's Disease?

Also known as Huntington's chorea, HD is an inherited disease (autosomal dominant) that usually manifests in midlife. HD is characterized by a loss of striatal neurons. The major motor symptom is chorea (unintentional dance-like movements) affecting the limbs, face and mouth. Dementia appears as the disease progresses. The gene responsible for HD has been identified. It produces a protein, huntingtin, that appears to play an essential role in striatal cell death; however, this role is not yet understood. There is currently no treatment for HD.

What should one keep in mind when discussing the substantia nigra and sub thalamic nucleus as components of the basal ganglia?

Anatomically, the subthalamic nucleus and substantia nigra are not part of the basal ganglia, but since they are so closely tied together functionally, they are often included as "honorary" parts of the basal ganglia.

What is anopia?

Anopia means lack of vision. It refers to the part of the visual field lacking vision, e.g., left/right (sometimes contralateral/ipsilateral), and upper/lower visual field

What can cause anosmia?

Anosmia can result from numerous things, including blockage of the airways, infection, or a tumor affecting the olfactory tract or bulb. The fibers of the olfactory nerve can be broken as a result of head trauma; a car accident, for example, can force the brain forward and shear off the olfactory nerve fibers as they leave the cribriform plate.

Pick True or False for the following statements regarding Purkinje cells: b. they receive direct input from mossy fibers

FALSE

Pick True or False for the following statements regarding Purkinje cells: d. some project directly to the thalamus

FALSE

TRUE or FALSE: Paralysis is a common result of basal ganglia disorders.

FALSE

Where do anterior hypothalamic temperature sensing cells AND pre optic cells project to?

Anterior hypothalamic cells (including medial preoptic neurons) project to the lateral hypothalamic area, to cells with widespread projections that initiate somatic responses 1. If temperature decreases, activate heat gain mechanisms -Shiver (involuntary) -Seeking warmth (voluntary) 2. If internal temperature increases beyond hypothalamic "set point", heat loss mechanisms are activated that include motivated behaviors to avoid heat (e.g., turn on A/C)

What is apraxia?

Apraxia - refers to a "dis-coordination" of movement, usually seen as an inability to perform a complex series of movements, such as that required to tie one's shoelaces. Apraxia commonly results from damage to the "higher" motor cortical areas, i.e., lateral premotor cortex or supplementary motor cortex. [contrast with damage to primary motor cortex, which produces paralysis]. In the lecture on the cerebellum, we will discuss a different term, ataxia, which is also a motor deficit.

What is the function of the arcuate nucleus?

Arcuate nucleus: hypothalamic sensor of leptin levels (Fig. 3) 1. αMSH- and CART- containing neurons are stimulated by a rise in leptin levels. Their activation suppresses appetite and increases energy utilization (anorexigenic). 2. NPY (neuropeptide Y)- and AgRP- containing neurons are also sensitive to leptin. Presence of leptin reduces secretion and release of NPY and AgRP by their arcuate neurons. NPY and AgRP neurons are active at low levels of leptin and insulin, and their activation increases appetite and decreases energy utilization (orexigenic). 3. Arcuate nucleus neurons project to both hypothalamic and non-hypothalamic regions

Describe the unique importance of the hypothalamic arterial supply

Arterial supply allows hypothalamus to directly sample endocrine and homeostatic variables 1. Feedback regulation of pituitary release (e.g., sex steroids, cortisol, thyroxin) 2. Homeostatic variables -Blood osmolality and fluid volume (angiotensin II) -Temperature -Body fat content (e.g.,circulating leptin), hunger and satiety signals

Generalize the sensory receptors and pathway of the spinocerebellar system

As described in a previous lecture, proprioceptive information is gathered by muscle spindles, Golgi tendon organs, and joint receptors. The primary afferent (i.e., first order) neurons have cell bodies located in dorsal root ganglia, and central processes that enter the spinal cord. Within the cord, there are numerous pathways for this information to be carried to the cerebellum. All the pathways include second orders cells in the spinal cord, whose axons terminate in the cerebellum on the same side. Two of the pathways are larger and most relevant clinically. These two pathways, dorsal (posterior) spinocerebellar tract and the cuneocerebellar tract, are described in detail, followed by a brief description of the other pathways.

Describe the location of the pathways for vertical eye movements

As noted above, the vertical gaze center is located in the midbrain reticular formation in an area known as the rostral interstitial nucleus of the medial longitudinal fasciculus (riMLF). As suggested by the name, the cells of this nucleus are intermingled with the fibers of the MLF, near its rostral end (adjacent to the III nucleus). Projections from this area extend to the appropriate motor neurons for vertical eye movements. The circuit is more complex than that for horizontal movements; we will not deal with the details!

Describe the support structures of the cochlea

Auditory receptor (inner hair cell) is surrounded by a very complex structure in cochlea that contributes to sensory transduction.

What do basal ganglia disorders normally cause?

Basal ganglia disorders do not cause paralysis. They are generally characterized by lack of movements (hypokinetic disorders) or the appearance of involuntary movements (hyperkinetic disorders). Muscular rigidity without increased stretch reflexes is common.

6. Contrast the deficits associated with lesions of the basal ganglia vs. UMNs or LMNs

Basal ganglia: Hyperkinetic - dyskinesia (ballism, chorea, athetosis) Hypokinetic - lack of movements Specific diseases Hyperkinetic: Hemiballismus - ballism in affected limbs Huntington disease - chorea. In later stages, dementia Hypokinetic: Parkinson disease - akinesia, bradykinesia, rigidity, resting tremor

58 year old man, has spontaneous movements of arms and legs that he can't suppress.

Basal ganglia: Huntington's disease

69 year old woman, not paralyzed, but general lack of movements; blank expression; "shuffling gait".

Basal ganglia: Parkinson's disease

What is the role of brainstem upper motor neurons?

Brainstem UMNs superimpose their influence on LMN circuits. They have a strong influence on muscle tone, and maintain posture while still and during movement.

What are the related brainstem structures of the limbic system?

Brainstem structures: periaqueductal gray matter and reticular formation nuclei, various autonomic nuclei (e.g., periaqueductal gray, parabrachial nucleus, solitary nucleus, dorsal motor nucleus of vagus), ventral tegmental area, interpeduncular nucleus

Describe the clinical significance of gustatory and olfactory deficits

Clinically, the gustatory (taste) and olfactory (smell) systems are relatively unimportant in comparison to the other senses. Assessment of taste, when carried out, is usually done more for testing of the cranial nerves than for specific interest in the gustatory system itself. Loss of taste is often accompanied by more significant neurological deficits. Loss of smell, however, can occur in the absence of other deficits. A decrease in ability to smell is a common feature of aging. Whether a consequence of normal aging or a result of trauma, loss of smell can lead to decreased appetite. Loss of taste or smell can become a significant issue in patients who are compromised nutritionally.

Describe the brachium of the superior colliculus and its targets

Composed of optic nerve fibers that bypass the LGN, continuing dorsally and medially. Two targets of the brachium of the superior colliculus: 1. Pretectum 2. Superior Colliculus

Describe the circuitry of cones

Cone> Bipolar cell > Ganglion cell i. In fovea: little convergence of cones > bipolars > ganglion cells ii. Outside fovea: more convergence means that a ganglion cell will be activated by light shining on several cones

What is sensory receptor adaptation?

Decrease in response of receptor to stimulus that has been present for a period of time (Fig. 5). Some receptors only respond to the onset of a stimulus.

Describe what results from damage to the primary motor cortex

Damage to primary motor cortex causes weakness (paresis) or paralysis. The remaining motor areas of cortex are involved in initiating and coordinating more complex movements, and damage to these areas leads to more complex deficits, such as the inability to perform a series of movements or to make movements appropriately guided by sensory input. A general term that covers a range of "higher order" motor deficits is "apraxia". A patient may recognize an object, and be able to describe its use, but "forget" how to use it. For example, a patient should know how to use a comb, or how to use a key to open a door. A patient with apraxia may have difficulty with such tasks despite having normal muscle strength.

What does damage of the PPRF cause?

Damage to the right PPRF causes a paralysis of rightward gaze. Contrast this deficit with that obtained following damage to the right abducens nerve: right nerve damage eliminates rightward movement of the right eye only, whereas damage to the right PPRF eliminates rightward gaze of both eyes.

What is dark adaptation?

Dark adaptation: increased sensitivity due to reduced exposure to light -Photopigments that are bleached must recover. Involves reconstitution of photopigments onto outer segment membranous discs. Slow - 10's of min.

What is the signal sent by decreased leptin levels?

Decreased leptin levels signal negative energy balance: adipose cells have low lipid levels -Increased activity of cells in arcuate nucleus that contain NPY and AgRP -Actively promote feeding via outputs of lateral hypothalamic zone and diminish energy usage via effects on endocrine and visceromotor system.

Describe the significance of decreased cutaneous reflexes

Decreased superficial (or cutaneous) reflexes. There are two superficial reflexes that can be altered by UMN damage. The abdominal reflex is elicited by stroking the side of the abdomen. The normal response is a contraction of abdominal muscles, causing the umbilicus to deviate toward the stimulated side. The cremasteric reflex is elicited by stroking the inside of the thigh, producing elevation of the testicle. The superficial reflexes are not always easy to see in a normal individual (the cremasteric reflex is reportedly useful in babies). UMN damage tends to decrease superficial reflexes, so if they may be absent normally, how could they be useful diagnostically?! The reference to babies may give you a clue, as you can readily imagine that a baby's inability to follow directions can make neurological assessment more difficult. As with all reflexes, the key is to look for symmetric (i.e., normal) vs. asymmetric (abnormal) responses on the two sides of the body.

Describe the efferents of the myotactic reflex

Efferents: A motor neurons - cell bodies in ventral horn of spinal cord. Receive input from Group Ia and II afferents. Synapse onto extrafusal muscle fibers. -The centrally projecting branch of the sensory afferent forms monosynaptic excitatory connections with the A-motor neurons in the ventral horn of the spinal cord that innervate the same (homonymous) muscle. This A-motor neuron also connects with local circuit neurons that form inhibitory connections with the A-motor neurons that innervate antagonistic (heteronymous) muscles. This arrangement is known as reciprocal innervation.

What structures play a role in emotion and expression?

Emotions and their expression: The amygdala is critically involved in emotions and their overt expression. Circuits related to the amygdala help to establish the emotional significance of sensory stimuli, while others connect to effector systems that mediate internal and external emotional expression. Lesions of the amygdala and/or related structures have a range of effects, altering the ability to recognize emotionally significant sensory stimuli and altering the expression of emotional states.

Where are the emotional aspects of odor processed?

Emotive aspects of olfactory sensation arise from the limbic projections which involve the amygdala and hypothalamus.

What do "Extrapyramidal" motor pathways include?

Extrapyramidal - the other descending pathways plus the basal ganglia and their connections.

Describe why eye movement nuclei are not targeted by the corticobulbar tract

Eye movements are controlled by cranial nerves III, IV and VI, projections from the frontal lobe certainly underlie our ability to make voluntary eye movements. However, the projections originate from a special region of motor cortex, and terminate NOT in the cranial nerve nuclei themselves, but in specialized regions of the reticular formation (called gaze centers) that coordinate the movements of the two eyes. We will discuss these issues in a lecture devoted to eye movements.

Describe the general concept of eye movements?

Eye movements are essential for vision. There are a limited number of types of eye movements, each controlled by specific neural circuits. These circuits converge on the origins of cranial nerves III, IV and VI, which in turn activate the eye muscles.

What are the main types of eye movements?

Eye movements fall into six main classes. We will discuss first the functions associated with each type of movement and then consider the circuits that give rise to the movements. Six basic types of eye movements: 1. saccades 2. smooth pursuit 3. vergence 4. vestibulo-ocular 5. optokinetic 6. fixation

Describe the somatotopic organization of the internal capsule

Face in genu; Body in posterior limb of capsule, With leg most posterior.

What is farsightedness?

Farsightedness (hyperopia). Light-focusing apparatus is too weak for shape of eye. Nearby images are focused behind the retina, blurring the retinal image. Distant images are focused correctly on the retina.

Describe how saccades are interrelated with head-turning

For a voluntary saccade, the frontal eye fields can trigger the motor cortex to initiate a head turn coordinated with the eye movements. The motor cortex would accomplish the head turn via corticospinal projections to the cervical cord, where there are lower motor neurons for muscles of the neck that turn the head. What about reflex saccades that need a head turn? Recall that the superior colliculus projects to the cervical spinal cord via the tectospinal pathway

What receptors are used to sweet, amino acid, bitter. and umami tastants?

For sweet, amino acid (umami) and bitter tastants, different classes of G-protein coupled receptors mediate taste transduction

What is HEMIBALLISM?

HEMIBALLISM (= hemiballismus) is a condition characterized by large involuntary movements of one side of the body caused by lesion of the subthalamic nucleus. The condition often occurs with sudden onset, resulting from a stroke that knocks out function of the subthalamic nucleus. Unintentional movements appear contralateral to the side of damage (why?). A bilateral lesion of the subthalamic nuclei would produce ballism, with bilateral symptoms. However, damage to the subthalamic nucleus almost always is unilateral, resulting from a stroke. The symptoms are unilateral; hence "hemiballismus".

What is Glaucoma?

Glaucoma:one cause is increased fluid pressure in the eye i. Aqueous humor produced in posterior chamber, flows into anterior chamber thru pupil; drains via meshwork at junction of iris and cornea. ii. Inadequate drainage can increase intraocular pressure.

What are some other terms for the globus pallidus?

Globus pallidus = pallidum = paleostriatum.

1. What region(s) is/are the major source of output from the basal ganglia?

Globus pallidus internal and substantia nigra pars reticulata.

Describe the cycle of GnRH secretion from the hypothalamus

GnRH is released in a coordinated, pulsatile manner from hypothalamic neurons.

Describe the basal ganglia circuits in Huntington's Disease

HD and basal ganglia circuits. In advanced stages, the striatum can be reduced in size by 90% or more. Certainly, the loss of so many striatal neurons would affect both direct and indirect pathways. In the early stages of the disease, the characteristic chorea may result from preferential loss of striatal cells involved in the indirect pathway (which normally inhibits movements).

Generalize the structure of the limbic system

Highly interconnected regions in vicinity of medial and basal surfaces of the telencephalon Consists of: 1. Neocortical structures (6 layered cortex): 2. Paleocortex (3-5 layered cortex): entorhinal cortex 3. Archicortex (3 layered cortex): hippocampal formation 4. Subcortical forebrain structures: 5. Diencephalic structures

Review the components of the cerebellar peduncles: source and target of each pathway; where does the pathway cross the midline (if it does). Relate the crossings to the side of the cerebellum vs. the side of the body on which the muscles are being controlled.

INPUTS. Sources are listed in the "Summary of Functional Subdivisions" table near end of lecture. The inputs target both the deep nuclei and the cerebellar cortex. In the cortex, the climbing fibers contact the Purkinje cells and the mossy fibers contact the granule cells. -Pontine fibers target the lateral hemispheres (cerebrocerebellum). -Spinal pathways target the vermal/paravermal regions (spinocerebellum). -Vestibular nerve and nuclei target flocculonodular lobe and midline vermis (vestibulocerebellum). -Inferior olive targets all parts of cerebellum. OUTPUTS: sources and targets are listed in the "Summary of Functional Subdivisions" table near end of lecture. Midline crossings (see also the "Cerebellar crossings" schematic from lecture): -Ascending paths from spinal cord - functionally uncrossed. -Inferior olive - fibers leave the inf. olive, cross the midline immediately and enter the cerebellum via ICP. -Pontine nuclei - fibers immediately cross the midline in ventral pons and collect to form the middle cerebellar peduncle.

What happens when the internal temperature rises above the set point?

If internal temperature is higher than the "set point" of hypothalamic cells, mechanisms for heat loss are activated 1. Endocrine: decreased release of TRH by hypothalamus, reducing TSH release by anterior pituitary 2. Visceromotor: vasodilatation, sweating,

What happens when the internal temperature drops below the set point?

If internal temperature is lower than the "set point" of hypothalamic cells, mechanisms are activated to prevent heat loss and to initiate heat production 1. Endocrine: release of TRH (thyroid releasing hormone by hypothalamic neurons, causes release of TSH (thyroid stimulating hormone) by anterior pituitary, which stimulates thyroxin release by thyroid gland 2. Visceromotor: vasoconstriction, piloerection, increased heart rate

Describe the interactions between odorants and different olfactory receptors

If you were to record the pattern of firing in different receptor cells caused by exposure to the same odor molecule, you would get different responses. For example, the citrus odor causes strong firing in the green receptor cell type, weak firing in the blue cell and no firing in the red cell. The almond odor causes no firing in the green receptor, very weak firing in the blue receptor and strong firing in the red receptor. If you look at the combination of firing patterns in each of the three receptor types to the four different odors tested, it looks as though the green receptor cell is selective mainly to citrus, less strongly to peppermint and least selective to the almond and floral odors.

Describe the receptive fields of second order neurons of the CNS versus peripheral neurons

In CNS, 2nd order neurons may have receptive fields that are larger than peripheral neurons due to convergence.

What do off-center cells respond best to?

In OFF-center ganglion cells, the effects of light stimulation are opposite to those of ON-center ganglion cells, except that diffuse illumination of the entire receptive field elicits little or no discharge because the center and surround oppose each other.

Describe joint receptors

In addition to muscle spindles and golgi tendon organs, proprioception also occurs from mechanoreceptors in and around joints. These joint receptors resemble many of the receptors found in skin, including Ruffini endings and Pacinian corpuscles. These joint receptors don not make a large contribution to sensing features such as limb position, but they do signal information about the position of fingers, for example.

Describe how lesions of the corticobulbar tracts can be mitigated

In general, the corticobulbar projections are bilateral. Consequently, the deficits following damage to cortex or corticobulbar tract on one side are minimized by the bilateral projections from the intact side. For example, projections to the motor trigeminal nucleus are bilateral. If the right motor cortex is damaged, both left and right trigeminal motor nuclei will still have functional input from the left motor cortex.

Describe the spatial organization of sacral VS rostral dermatomes in the ALS of the spinal cord.

In the anterolateral cord, the ALS fibers are organized with sacral information carried most laterally. Information from successively more rostral dermatomes is carried in more medial fibers (see figure above).

Describe the firing of a Pacinian Corpuscle

In the bottom figure, deformation of a mechanosensory afferent (a Pacinian corpuscle) leads to a stretching of the membrane of the afferent fiber, increasing the probability of opening stretch sensitive cation channels in the membrane. Opening the channels leads to depolarization of the afferent fiber (the generator potential), which if large enough, produces action potentials (spike potential).

What is lack of check?

Inability to stop a limb as resistance is suddenly removed.

What is dysdiadochokinesia?

Inability to sustain rhythmical, alternating movements. Distinctive sign of cerebellar damage.

What is nystagmus?

Includes fast/slow oscillation (as we have discussed nystagmus previously); can also refer (imprecisely, but still commonly used) to "jerky" eye movements (think "intention tremor" of eye movements).

Describe the function of the reticular formation

Includes nuclei associated with • autonomic system (basic life support) • coordination of reflexes mediated by brainstem cranial nerves • modulation of pain pathways • "chemical" pathways (recall earlier lecture on this topic) • regulation of sleep, wakefulness and arousal • motor control -eye movements (upcoming lecture) -reticulospinal pathways

Describe the anatomy of the inner ear

Includes vestibular and auditory functions. 1. A system of bony tubes (the bony labyrinth) and, within it, membranous tubes (the membranous labyrinth) encased in temporal bone. 2. Receptor organs 3. Vestibulo-cochlear (VIIIth) nerve (Fig. 3)

What signal is sent by increased leptin levels?

Increased leptin levels signal positive energy balance: adipose cells have enough lipid -Activation of αMSH and CART containing neurons. -Projection to lateral hypothalamic area results in decreased feeding behavior. -Projection to paraventricular nucleus results in endocrine, visceral, and somatic effects that increase energy utilization

Describe the higher-center inputs of the hypothalamus

Inputs from higher centers arrive via fornix, mfb, stria terminalis, ventral amygdalofugal pathway (vafp) and others pathways 1. As a generalization, amygdalar inputs convey olfactory information (via stria terminalis) or emotional state (via vafp) 2. Fornix connects hippocampal formation and mammillary bodies as part of memory-related functions of limbic system.

Describe the inputs of the vestibular nuclei

Inputs. All nuclei receive inputs, to varying degrees, from: 1. vestibular organs (semicircular canals and otolith organs) 2. contralateral vestibular nuclei 3. fastigial nucleus and floccular-nodular lobe of cerebellar cortex 4. proprioceptive information from neck and postural muscles

Describe the inputs, outputs, and function of the central nucleus

Inputs: 1. Basolateral amygdala (processed sensory/emotional state information) 2. Viscerosensory input (e.g., solitary nucleus) Outputs (via amygdalofugal pathway) (Fig. 4) 1. Hypothalamus 2. Brainstem centers, e.g. midbrain periaqueductal gray, parabrachial nucleus, solitary nucleus, dorsal motor nucleus of vagus 3. Basal nucleus of Meynert (part of cholinergic forebrain). Functions: 1. mediates a variety of autonomic and behavioral responses to learned emotional stimuli 2. through basal nucleus of Meynert, projections to neocortex plays role in arousal and modification of responses in association sensory cortex (Fig. 5)

Describe the inputs, outputs, and function of the corticomedial nuclei

Inputs: Olfactory bulb Output (via stria terminalis) to ventromedial nucleus of the hypothalamus Function: mediates olfactory-based regulation of feeding behavior

Describe the inputs, outputs, and function of the basolateral nuclei

Inputs: modality-specific sensory information from the dorsal thalamus and sensory association cortices, limbic association cortex Outputs: 1. medial dorsal nucleus and limbic association cortex (cingulate, orbitofrontal, and prefrontal cortical areas). Function: mediate cognitive/conscious features of emotions 2. nucleus accumbens. Amgydala and nucleus accumbens mediate complementary emotional responses 3. central nucleus of the amygdala. This major output provides basis for expression of emotional states. Functions: 1. the basolateral nuclei combine specific sensory inputs with other information to establish the emotional significance of sensory stimuli. -after learning occurs, the basolateral nuclei match incoming stimulus with appropriate emotional state.

What is central hearing loss?

Interruption or alteration of ascending auditory pathways can result in hearing loss and/or tinnitus. Causes include tumors, cardiovascular accidents, and age-related changes. Central changes can result from conductive or sensorineural hearing loss.

Describe the role of the lateral premotor cortex

Lateral premotor cortex is located in the lateral part of area 6. It is particularly important in sensorimotor integration (i.e., selecting and controlling movements based on sensory inputs). It receives input from the frontal eye fields (area 8) and is important for visually-guided movements. It also receives input from language areas. Lesions can lead to a patient being unable to make a particular movement in response to verbal command or visual cue, but capable of making the movement (and interpreting the commands and cues). Such deficits highlight the role of premotor cortex in selecting appropriate movements.

Which side of the body would show ataxia following damage to the left inferior cerebellar peduncle?

Left [left ICP carries information from left limbs]

Describe the somatotopic organization below the internal capsule

Leg represented laterally, neck medially.

Review your brainstem cross-sections, and identify a level (section) and the region of the section in which a lesion could cause alternating hemianalgesia.

Lesion in lateral medulla that damages spinal V tract/ nucleus and spinothalamic tract (ALS).

Describe lesions in the dorsal column system

Lesions in the dorsal columns will lead to ipsilateral signs whereas lesions above the decussation in the medulla will lead to contralateral signs.

Describe the effect of lesions of the ventral posterior thalamus on pain.

Lesions involving the ventral posterior thalamus can cause a period (usually months) of analgesia followed by chronic pain. This is known as thalamic pain syndrome, and is an example of "central pain" (pain that results from a problem within the central nervous system).

Describe the effects caused by lesions of the corticospinal tract

Lesions lead to weakness or paralysis Deficits are most obvious in limbs • innervated by single side of cortex (lateral system of projections) • trunk (medial system) innervated bilaterally DEFICIT LOCATION IS... • contralateral to a lesion above the pyramidal decussation • ipsilateral to a lesion below the pyramidal decussation

What is hemianopia and quadrantanopia?

Lesions may affect half of the visual field (hemianopia) or one quarter of the visual field (quadrantanopia)

Describe distal lesions of the pathways connected to the primary visual cortex

Lesions of pathways beyond primary visual cortex can have specific, sometimes very unusual effects. 1. Lesions of the dorsal pathway: diminish motion perception and smooth pursuit eye movement 2. Lesions of the ventral pathway: diminish form and color vision

What happens in cases of lesions of the optic nerve?

Lesions of the eye or optic nerve: partial or total blindness in one eye. 1. A variety of sources: -Retinal detachment -Papilledema : increased intracranial pressure can block axoplasmic flow in retinal ganglion cell axons. -Retina supplied by the central retinal artery, entering through optic disc. 2. Retinal damage causes deficit in opposite part of visual field, because the eye's optics result in an inverted image on the retina.

What happens with lesions to the superior colliculus?

Lesions of the superior colliculus would be expected to cause a deficit in saccades. In practice, damage to the superior colliculus in humans often damages the nearby vertical gaze center. Thus, conjugate vertical movements, either elevation, depression or both, associated with saccades or smooth pursuit are usually affected following damage to the superior colliculus and its surrounding structures. 1. Can't move eyes toward side of lesion. 2. At rest, eyes deviated away from side of lesion.

Describe the effects of lingual and cuneus lesions on the visual cortex.

Lesions restricted to the cuneus or lingual gyrus a. Cuneus: contralateral lower homonymous quadrantanopia b. Lingual gyrus: contralateral upper homonymous quadrantanopia c. Macular sparing is possible in these lesions, if loss is due to occlusion of branches of posterior cerebral artery. The macular is not spared if lesion is due to other causes (e.g., trauma).

What is light adaptation?

Light adaptation: decreased sensitivity to an ongoing light stimulus -Photopigments (e.g. retinal and opsin) dissociate and can no longer affect transducin (photopigments are bleached)

Describe the function of local circuit neurons

Local circuits in the spinal cord act as central pattern generators. They are involved in patterned or rhythmic locomotion, e.g. swimming, walking, running, flying. They control the timing and coordination of rhythmic movement. Rhythmic patterns are not dependent on sensory input, and they are not dependent on input from descending projections from higher centers.

Describe the Inferior salivatory nucleus

Location: Medulla Cranial Nerve: CN IX Postganglionic Synapse: Otic ganglion

Describe the Dorsal motor nucleus of vagus

Location: Medulla Cranial Nerve: CN X Postganglionic Synapse: Ganglia near heart, vessels, lungs,

What is the role of the red nucleus and rubrospinal tract in motor function?

Losing it's status as a source of upper motor neurons. THIS IS A CHANGE FROM DESCRIPTIONS IN PAST YEARS OF BMB. Growing evidence indicates that, in humans, the red nucleus is almost exclusively associated with the cerebellum (and thus still plays a motor role), but has little or no component of upper motor neurons.

For the dorsal column-medial lemniscus pathway, identify the nature of the deficit if the pathway is damaged

Loss of discriminative touch (2- point discrim.), vibration, pressure, conscious proprioception. Deficit is ipsilateral if lesion is below internal arcuate fibers; contralateral if above internal arcuate fibers.

What is anesthesia?

Loss of pain, temp and fine touch

Describe the analysis of visual form and motion in the visual cortex

M (motion/location) and P (color, form) type LGN neurons synapse on separate populations of cortical neurons (Fig. 16). Thus, M and P pathways remain largely separate throughout visual cortex.

Which cerebral artery supplies the cortical area for sensation from the face?

MCA (middle cerebral artery)

What is the Macula Lutea?

Macula lutea ("yellowspot") is special region on retina

What is macular degeneration?

Macular degeneration: loss of photoreceptors in and around fovea

Where is the medial motor system located?

Medial system in ventral (anterior) funiculus

Describe the anatomy of the taste pathway of CN VII

N. VII - Facial - anterior tongue - Geniculate ganglion.

Describe the anatomy of the taste pathway of CN IX

N.IX - Glossopharyngeal - lateral posterior tongue - petrosal ganglion

What is Nearsightedness?

Nearsightedness (myopia). Light-focusing apparatus is too strong for shape of eye. Nearby images are focused correctly on the retinal surface, but distant images are focused in front of retina, blurring the retinal image.

Describe the secondary taste pathway

Neocortical area: caudolateral oribitofrontal cortex. This region acts as an integration site for taste, visual and olfactory cues that are associated with the ingestion of food.

What produces pain hypersensitivity?

Neural plasticity in the somatosensory system produces pain hypersensitivity.

Where do temperature sensing neurons project to?

Neurons in medial preoptic area receive inputs from temperature- sensing neurons in anterior hypothalamus, and from thermal receptors in skin and mucous membranes. It coordinates endocrine and visceromotor responses.

Describe the neurons of the parasympathetic nervous system

Neurons in the parasympathetic NS innervate portions of the head and the lower body cavity. These cells are located in the brainstem and the sacral spinal cord. The parasympathetic NS governs homeostasis, largely encouraging processes that build up energy stores during rest, eg., digestion, glucose uptake, reduced heart rate, blood vessel dilation, pupil constriction.

Describe neuropathic pain of an injured limb

Neuropathic pain in an injured limb. Peripheral nerve injury triggers local neuroinflammation and invasion of immune cells that become activated. Immune cells also invade the DRG and the dorsal horn of the spinal cord. Neuroinflammation is associated with increased levels of proinflammatory molecules released from damaged tissue, immune cells, peripheral glia such as Schwann cells and central glia. Several proinflammatory molecules are sprouting factors and trigger sympathetic sprouting from neighboring sympathetic ganglia (SG) or sprouting of large myelinated Aβ into superficial laminae of the dorsal horn. Here, the STT ascends towards the VPL nucleus of the thalamus. Proinflammatory molecules also mediate the transition of neuronal activity from normal to hyper-responsiveness in response to peripheral stimulation. Thus, sprouting and neuronal sensitization, controlled by remote neuroimmune signaling, contribute to an abnormal sensation of pain interpreted by the cerebral cortex (Cc).

Describe neuropathic pain

Neuropathic pain is pain initiated or caused by a primary lesion or dysfunction in the PNS or CNS. Most common neuropathic symptoms include allodynia and hyperalgesia, or pain evoked by a normally non-painful stimulus and exaggerated pain evoked by normally moderately painful stimulus, respectively. Neuropathic pain is usually chronic, and can exceed six months. Neuropathic pain can be accompanied by hyper- responsiveness to applied stimuli, and nerve sprouting, which results in structural remodeling and anatomical reorganization of neuronal circuitry. Sprouting can occur at the peripheral injury site or remotely in the DRG and spinal cord.

What is the mechanism of phantom pain?

Nociceptors can cause chronic pain if they are damaged. (A) In the normal state, if a nociceptor is activated by a noxious stimulus, the nerve cell transmits the information via the sensory system to create a painful sensation in the brain. (B) If the nociceptor is damaged, it can start firing randomly and activate other nerves that eventually cause phantom pain. (C) If the nociceptor was an inhibitory nerve, its inactivation through damage could activate other nerves in the sensory network that eventually cause phantom pain.

Via which center is oblique eye movement accomplished?

Oblique eye movements are accomplished by activation of both vertical and horizontal centers, with the trajectory of movement specified by the relative contributions of the different gaze centers.

Describe the regeneration of olfactory cells. What makes their surrounding mucous?

Olfactory epithelium with major cell types. Bowman's glands produce mucus. Olfactory cells are generated continuously from dividing stem cells.

When can olfactory hallucinations occur?

Olfactory hallucinations can also occur, most often as a result of an epileptic seizure that involves the anterior medial temporal lobe (where the pyriform and entorhinal cortices are located).

Generally describe the targets of the central olfactory tract

Olfactory information diverges quickly to reach multiple areas in the forebrain. Many of these targets are components of the limbic system. This close association between the olfactory and limbic systems underlies the many and varied effects an odor can elicit, from the "adrenaline jolt" that can accompany the smell of smoke to the long-ago memory of grandma's kitchen that comes forth with the smell of cookies baking.

In what regions of the cortex is olfactory information processed? What carries this information?

Olfactory information is processed in several regions of the cortex. Information from the olfactory bulb is transmitted by the mitral and tufted relay cells, whose axons travel in the lateral olfactory tract.

What do olfactory receptor cells synapse with?

Olfactory receptor cell axons synapse with mitral cell apical dendritic tufts and periglomerular cell processes within glomeruli. Granule cells and mitral cell lateral dendrites constitute the major synaptic elements of the external plexiform layer.

What are olfactory receptors?

Olfactory receptor cells are located in the olfactory epithelium in the nasal cavity (see figure). The receptor cells are neurons that, like other primary sensory neurons, have a peripheral and a central process. The central processes of the receptors together make up the olfactory "nerve", although a single nerve is never formed. Rather, the nerve consists of many separate fascicles that penetrate the adjacent cribriform plate of the ethmoid bone and then immediately enter the olfactory bulb.

Describe Opiate Control of Pain

Opium derivatives such as morphine are analgesics. Opiates act on most or all sites involved in the descending control of pain. Some CNS regions susceptible to opiate drugs periaqueductal gray matter, opiate -sensitive neurons in dorsal horn of spinal cord. Endogenous opioids include enkephalins, endorphins and dynorphin. All three are found in the periaqueductal gray matter. Enkephalins and dynorphins are also present in the rostral ventral medulla and in spinal cord regions. Enkephalin-containing local circuit neurons in the dorsal horn synapse with axon terminals of nociceptive afferents, which in turn synapse on dorsal horn projection neurons (see Figure on Page 14). The release of enkephalin onto nociceptive terminals inhibits the release of transmitter from these terminals onto the projection neuron. This reduces the amount of activity traveling to higher CNS centers.

Describe Parkinson's Disease

PARKINSON'S DISEASE (PD) results from degeneration of dopaminergic neurons in the substantia nigra, pars compacta that project to the striatum (the nigrostriatal pathway). PD is characterized by akinesia (few movements, because of difficulty initiating movements), bradykinesia (slow, shortened movements), rigidity, and resting tremor. Shuffling gait, flexed posture, and mask-like facial expression are common. The symptoms of PD are in many ways the opposite of those in HD. In fact, prolonged drug treatment or too high a dose of drug for treating PD can induce HD-like chorea.

Describe the roles of the direct and indirect pathways in Parkinsons's Disease

PD appears to affect both direct and indirect pathways. In the striatum, different cell types give rise to the direct and indirect pathways. Both of these cell types receive dopaminergic input from the SN pars compacta (see schematic of pathways; DA - dopamine). The key is that these cell types have different DA receptors. For the direct pathway, DA excites the striatal cells. Loss of DA leads to decreased activity in the direct pathway. For the indirect pathway, DA normally inhibits the striatal cells; loss of DA leads to increased activity in the indirect pathway.

Describe the clinical significance of pain. What is the pain threshold?

Pain alerts the brain to the presence of noxious or potentially threatening stimuli. Pain threshold, which is the minimum stimulus that elicits pain, is an indication of the health of the receptors, nerves, and pathways in the pain system. Pain tolerance, which is the degree of pain a subject can tolerate before experiencing physical or emotional impairment, may be altered by medications as well as by physical, cognitive and affective interventions and training. Pain hypersensitivity, a pathological condition arising from changes in signal transduction in the peripheral or central nervous system, can cause great emotional distress.

Describe the parallel organization of somatic sensory pathways

Parallel organization of somatic sensory pathways. Dorsal column/medial lemniscus pathway carries information from cutaneous mechanoreceptors and proprioceptors. Anterolateral/spino-thalamic system carries information primarily from thermoreceptors and nociceptors.

What is the function of the paraventricular nucleus?

Paraventricular nucleus activates endocrine and autonomic responses associated with increased energy use 1. Receives projections from arcuate nucleus 2. Some cells contain releasing factors for anterior pituitary hormones (e.g., TSH, ACTH) 3. Other cells project to brainstem and spinal autonomic nuclei, e.g., sympathetic neurons that increase metabolism

Information about discriminative touch is carried in which limb of the internal capsule?

Posterior

What is the primary motor cortex?

Primary motor cortex, Brodmann's area 4, M1; these terms are synonyms, and frequently the term "motor cortex" is used to mean the same thing. However, many areas of cerebral cortex contribute to motor function.

What are the roles of the premotor cortex and the primary motor cortex?

Primary motor cortex: precentral gyrus and paracentral lobule Execution of fine, skilled movements. Premotor cortex: frontal lobe, anterior to primary motor cortex Planning and selecting movements.

Describe the structure of primary somatosensory neurons

Primary somatosensory neurons are generally pseudo-unipolar neurons, and are specialized to respond to one (and sometimes more) of many different types of stimuli - e.g. touch, pain, temperature, pressure. The cell bodies of somatosensory receptor cells are located in the dorsal root ganglia, which is outside the central nervous system. The peripheral axon innervates targets in the periphery (skin, muscle etc) from where it receives information about the particular sensory stimulus. Action potentials are generated in the node of Ranvier in the axon at the peripheral region, then travel down the peripheral axon towards the cell body, and through into the portion of the axon that ends in the nerve terminal in either the spinal cord or other regions of the central nervous system.

Describe the autonomic control of the cardiovascular system

Principal task: Maintain oxygenated blood supply 1. Sensory information communicated to CNS 2. Control Rise in blood pressure 3. Control Fall in blood pressure

Describe the pathway of lens accommodation

Process by which shape of the lens is actively controlled to allow the eye to focus on objects within 10-20 feet. a. Afferent path: Retinal ganglion cells-->LGN-->Visual Cortex-->? b. Efferent path: Midbrain center near E-W-->Edinger-Westphal Nucleus--> Ciliary ganglion-->Contraction of ciliary muscle-->Lens thickening

What feeds-back to GnRH release?

Products of gonadal function (sex steroids and inhibiting factor (Inhibin)) affect GnRH release -GnRH neurons have few steroid hormone receptors -Effects of sex steroids appear to act through the variety of neuromodulator inputs -Regulation of gonadotropin release also occurs at the level of the pituitary -Reproductive quiescence during childhood appears to depend on highly sensitive negative feedback response to gonadal steroid hormones -Around puberty, the feedback control of GnRH release is reduced. This and other changes in the control of GnRH neurons cause increased GnRH release that results in adult gonadal function

Compare a lesion in the right Gracilis to a lesion in the right medial lemniscus

RIGHT GRACILIS: Would the deficit affect the upper body or the lower body? (A: lower). Left side or right side? (A: right). What would be different in a second patient who had a lesion that completely cut her right medial lemniscus (and nothing else)? Answer: Two main differences - first, the deficit would be on the left side of the body (the fibers in the right ML carry information from the left side of the body. The pathway crossed the midline via the internal arcuate fibers). Second, the deficit would involve both limbs and the trunk.

Describe the connections of the Central Control of Visceral Motor Function

Rostral NST passes visceral sensory information to rostral reticular formation, hypothalamus and parabrachial nucleus. Caudal NST passes sensory information to the preganglionic neurons in brainstem and spinal cord.

Are rotational eye movements voluntary?

Rotational eye movements are not a major component of voluntary eye movements; rather, they are (relatively small) movements that compensate for the angle, or side-to- side tilt, of the head. Head tilt is detected by the vestibular system which, among other things, provides for compensatory eye movements.

What are rotary tests?

Rotatory tests examine performance of a functional pair of semicircular canals. (see Nystagmus Worksheet) - Each functional pair can be tested separately, determined by the orientation of the patient's head relative to the axis of the rotation.

What are smooth pursuit movements?

Smooth pursuit movements are tracking movements that keep a moving stimulus on the fovea. This type of movement is reflexive in the sense that we cannot generate such a movement without following a moving target. (Watch a friend's eyes while you ask her to track your moving fingertip. Then ask her to repeat the eye movement without having a finger to follow. Without a moving target, the eye movements should be either one fast movement or a series of small movements, each of which are saccades). Of course, we usually choose whether or not to track a moving stimulus, so one might say this is a "voluntary reflex".

How are somatosensory afferents classified?

Somatic sensory afferents differ in their response properties and can be classified based on a set of characteristics. These characteristics are: (1) Axon diameter (2) Size of receptive field (3) Adaptation during a stimulus (4) Selectivity in responses to different somatic stimuli.

What is the function of the trigeminal pathway?

Somatosensory input from the head includes the same three types of information that we've discussed for the body: discriminative touch (as a broad term including all types of info carried by DC/ML paths), pain and temperature, and proprioception. Once again, the different types are handled by pathways that are separate below the thalamus. Most of the sensory information is carried by the trigeminal nerve, which carries information from the face, oral and nasal cavities, cornea, meninges and cranial blood vessels. Recall that there are also small sensory components (General Somatic Afferent) of cranial nerves VII, IX and X. All this GSA information is processed by the trigeminal nuclei.

Describe the structure of somatosensory receptors

Somatosensory receptors are the modified distal processes, sometimes called dendrites, of primary sensory neurons that have their cell bodies in dorsal root ganglia. These distal processes may be encapsulated endings, or free nerve endings. It is these endings that receive sensory stimuli.

What GnRH neurons do not project to the median eminence?

Some GnRH neurons do not project to the median eminence. These project to other brain regions to modulate reproductive function or behaviors.

Describe the function of the mesencephalic trigeminal nucleus

Some reflexes. This nucleus is unique in that it is the one place where the cell bodies of primary afferent fibers are located within the central nervous system (all other cases have the cell bodies in peripheral ganglia, i.e., dorsal root ganglia or cranial nerve ganglia).

What is sound?

Sound is a propagated disturbance of molecules within a medium (usually air). Periodic stimulus moves air molecules back and forth. This movement causes minute, rhythmic changes in air pressure.

Describe the specific parasympathetic descending pathway of the hypothalamus

Specific descending output: to parasympathetic division -direct (hypothalamo-medullary and -spinal pathways) and indirect (dorsal longitudinal fasciculus) projections to visceral motor nuclei in the brainstem and sacral spinal cord mediate parasympathetic output -functions include: decreased heart rate and blood pressure, increased G-I tract motility and secretions, pupillary constriction and lens accommodation, erection of sexual organs

Describe spinal shock. What is its cause?

Spinal shock. UMN syndrome usually does NOT appear immediately following sudden damage to the descending motor pathways. Rather, there is a period of flaccid paralysis with lack of reflexes. This is called spinal shock, and may last for days to months before a transition to UMN syndrome. Damage with a slower onset, such as that produced by a tumor, may lead directly to UMN syndrome without a period of spinal shock.

Describe the spinocerebellum

Spinocerebellum-vermis + paranormal parts of hemispheres, interposed nuclei (emboliform and globose). The main function is execution of movement. This area compensates for changes in load, regulates muscle tone, guides limb movement, helps maintain posture.

What is the function of the spinotectal tract?

Spinotectal (or spinomesencephalic) tract is similar to the spinoreticular, except that the ascending axons terminate in the tectum of the midbrain (in periaqueductal gray and superior colliculus).

Distinguish between the information in A-delta fibers and C fibers of the ALS.

The A-delta fibers convey "fast pain", which is sensed very quickly and has the quality of sharp, well-localized stabbing pain. In contrast, the C fibers convey "slow pain" which has a dull aching or burning quality and which is sensed with a noticeable delay (compared to fast pain). The A-delta fibers feed into the spinothalamic pathways while the C fibers feed mostly into the spinoreticular (and other) pathways.

What is the Romberg Test?

The Romberg test also assesses proprioception. The patient stands with eyes open and feet together, and the examiner assesses how much the patient sways. The patient then closes his eyes. If swaying increases, inadequate proprioceptive input is reaching the brainstem (with eyes open, the visual system was compensating for loss of proprioceptive input). The Romberg test is positive if swaying increases with eyes closed, indicating damage to dorsal column system.

Describe the adrenal medulla as a ganglion

The adrenal medulla is a specialized ganglion. Each adrenal medulla receives preganglionic input from axons that ran within the chain then exited to synapse in the medulla. These synapses release acetylcholine on small postganglionic cells that, in turn, release epinephrine or norepinephrine into the bloodstream. Release of these neurotransmitters into the bloodstream produces widespread, generalized sympathetic activation.

Discuss the role of the amygdala in relation to the basal ganglia

The amygdala is not a part of the basal ganglia functionally, but it can be considered part of the basal ganglia anatomically (defined as a gray matter/nuclear region in the deep telencephalon; i.e., deep to cerebral cortex). The Sylvius Atlas is among the sources that list the amygdala as part of basal ganglia (it is highlighted in the image set showing the basal ganglia). Functionally, the amygdala is associated with the limbic system and emotion, and we will discuss it in a later lecture. The current lecture focuses motor functions of the basal ganglia (as defined above) and the functionally related structures.

Describe the effects of anterior white commissure lesions on the ALS

The anterior white commissure looks quite small in the standard cross section, but of course the commissure is present through the entire length of the cord. The clinical significance is that a lesion of this commissure usually affects one or a few spinal levels. The deficit would be limited to the associated dermatome, and would be bilateral. A lesion, say, of the commissure from T3-T6 would cause a "ring" of diminished pain (and temperature) sensitivity around the chest. Pain sensation from lower levels would be unaffected. (Note: the deficits would include both pain and temperature sensations; clinically, we usually test and discuss only the pain).

Describe the interplay of the basal ganglia and the limbic system

The basal ganglia also contain circuits closely tied to the limbic system. The ventral striatum and ventral pallidum have connections with the cingulate cortex, amygdala, and prefrontal cortex. These circuits play important roles in motivation and reward. Lesions can lead to both motor and psychiatric disorders. These pathways are believed integral to addiction, including drug and alcohol abuse.

Discuss the role of the basal ganglia in eye movements

The basal ganglia also contribute to eye movements. The figure shows the similarities of circuits for body movements and eye movements. Big picture: cortex projects to striatum, which projects to pallidum/s. nigra pars reticulata, which projects to thalamus, and back to motor areas of cortex. Note that for the oculomotor loop, the cortical areas that project to striatum include FEF and supplementary eye field, as well as posterior parietal cortex (area of parietal eye field). Focus on the parallels here; you do not need to learn the detailed differences between these two loops. The value of recognizing the similarities lies in the opportunity to see similarities in deficits after lesions. Recall that the basal ganglia contribute to the initiation and termination of movements. Patients with Parkinson's disease generally exhibit fewer spontaneous body movements than normal, often complaining of a difficulty in starting a movement. These patients often have a paucity of eye movements as well. Patients with hyperkinetic disorders may also have abnormal eye movements.

What are the non-motor roles of the basal ganglia?

The basal ganglia have important non-motor functions, including contributions to cognition and emotion. As discussed above, Huntington's disease includes dementia as well as motor deficits. The dementia is not explained by damage to the motor circuits that we have traced, but to other circuits that we will not consider in detail.

Describe a basic cerebellar circuit

The basic circuit includes a main excitatory loop and an inhibitory cortical loop.

Describe the origin of the central taste pathways

The central processes of the taste fibers terminate in the rostral portion of the solitary nucleus (nucleus of the solitary tract; nucleus tractus solitarius). The rostral part of the solitary nucleus is sometimes called the gustatory nucleus. The gustatory nucleus gives rise to the central taste pathways. The gustatory nucleus projects to the ipsilateral thalamus: the taste pathway remains ipsilateral from tongue to cortex. The ascending taste fibers from the gustatory nucleus travel in the central tegmental tract.

Describe the role of the cerebellum in learning motor skills.

The cerebellum is also important for learning motor skills. Presumably, practicing a motor skill is associated with long term cellular changes in the cerebellum. If you are interested, you may enjoy topics covered in the textbook in Figures 19.12 and 19.13 and associated text (pages 428-9) Finally, there is increasing evidence that the cerebellum also contributes to cognitive tasks. Such tasks include those requiring judgment of elapsed time or velocity.

What is the cerebellum, and where is it in the motor hierarchy?

The cerebellum is essential for normal movement. It is "above" the UMNs in the motor system hierarchy, meaning that the cerebellum exerts its effects on motor behavior by affecting UMNs. It is essential not only for normal movements but also in learning new motor skills.

Where does the conscious discrimination of odors most likely occur?

The conscious discrimination of odors most likely occurs in the orbitofrontal and frontal cortices (neocortex region). These two regions receive olfactory information via two separate projections - one through the thalamus and one directly to the neocortex.

Describe higher order somatosensory processing that is accomplished by the cerebral cortex.

The cortex combines information from different receptor types onto cortical neurons with complex properties to form "higher order" perceptions, allowing us to identify qualities such as size, shape, and texture. Sensory deficits resulting from somatosensory cortex lesions can include agraphesthesia (inability to identify letters drawn on the skin) and astereognosia (inability to name objects held against the skin). These deficits are quite different from analgesia (loss of pain and temperature sensation) and anesthesia (loss of pain, temp and fine touch) which describe losses due to lesions at lower levels (peripheral nerves to thalamus). You will hear more about conditions that include somatosensory deficits in a later lecture on higher order cortical functions.

What is the purpose of the corticobulbar tract?

The corticobulbar tract has the same general origins and course as the corticospinal tract, but axons terminate in the cranial nerve nuclei. Targets include sensory and motor nuclei; we focus on the motor projections. The cortical projections are responsible for voluntary movement. Lesions can cause problems with movements of the jaw, tongue, facial muscles, eyes and throat. Speech requires coordinated control of the tongue, lips and vocal cords, so speech could be affected by damage to corticobulbar projections to a number of cranial nerve nuclei, including VII, XII and nucleus ambiguus.

What is the corticospinal tract?

The corticospinal tract (a.k.a. pyramidal tract) comprises a large lateral corticospinal tract and a much smaller anterior (or ventral) corticospinal tract. The lateral corticospinal tract includes roughly 90% of the corticospinal fibers. These fibers cross in the pyramidal decussation and descend in the lateral funiculus of the spinal cord. They terminate laterally in the ventral horn and thus are part of the lateral pathways.

Describe the cuneocerebellar tract

The cuneocerebellar tract carries information from the upper trunk and arm. Primary afferents enter the cord and pass directly into the ipsilateral fasciculus cuneatus. The axons ascend to the ipsilateral external cuneate nucleus (also known as lateral cuneate or accessory cuneate nu.), in the medulla. Neurons of the external cuneate nucleus project to the ipsilateral cerebellum via the inferior cerebellar peduncle.

Describe Somatotopic organization in the DC/ML pathways

The dorsal column/medial lemniscus pathway is organized topographically throughout its course. Because this organization reflects the body surface, it is called somatotopic organization. The significance is that damage to a portion of the pathway produces deficits associated with a specific, and predictable, part of the body. In other words, knowing the detailed distribution of a deficit can give insights into the location of a lesion.

Describe the autonomic nervous system

The first order neurons of the autonomic NS originate in the brainstem or spinal cord. They synapse in ganglia on the second order neurons that innervate the glands or visceral organs. Though there can be descending input to autonomic NS activity (the hypothalamus is the locus of control for the ANS), the system controls involuntary activity related to homeostatic function. Efferents have many synaptic terminals, sometimes separated by hundreds of microns from postsynaptic receptors.

What types of neuron do the sub modalities of somatic sensation use? Are they the same neuron?

The four sub-modalities share a common class of sensory neuron - the dorsal root ganglion neurons. But, different dorsal root ganglion neurons have different specializations of their peripheral nerve terminals (the peripheral somatosensory receptors), and each modality is mediated by a distinct system of receptors.

Describe the golgi tendon organ

The golgi tendon organ (GTO) is a mechanoreceptor that is located in tendons and informs the central nervous system about changes in muscle tension. The GTO is formed by branches of Group Ib afferents distributed among the collagen fibers that form the tendons. Each GTO is arranged in series with a small number of extrafusal muscle fibers. Taken together, the population of GTOs for a given muscle provides an accurate sample of the tension that exists in the muscle.

What is the function of the hippocampal formation?

The hippocampal formation and its connections contribute to consolidation of short-term memory into long-term memory, underlying "declarative memory". The "Papez circuit" describes circuitry linking the hippocampus to several structures. It includes: cingulate cortex, to entorhinal cortex, to hippocampal formation, to mammillary body of hypothalamus, to anterior nucleus of thalamus, to cingulate cortex (Fig. 2).

What is the limbic system?

The limbic "system" is a diverse set of brain centers involved in a diverse set of functions, including emotions, reward, learning and memory. These centers do not function as a single system. However, the structures are highly interconnected and their functions are interrelated. Additionally, some limbic structures receive input from the olfactory system, and some olfactory structures are considered part of the limbic system. (Fig. 1)

What is phantom pain?

The loss of a limb generates an internal mismatch between the brain's representation of the body and the pattern of peripheral input that reaches the neocortex. The consequence is the illusion that the missing body part is still present and functional. The phantom sensation occurs almost immediately after limb loss and decreases with time. Amputees can sometimes develop phantom pain, which they ascribe to the missing limb. Phantom pain can develop from a functional re-organization of the cortex following damage to the nociceptors that occurs following limb loss (the phantom is indicated by a dashed line with the colored regions showing the most vividly experienced parts).

Describe the function of the lower areas of the reticular formation

The lower areas - in caudal pons and medulla - include the origins of the reticulospinal tracts

Describe the intertwined structure and function of the macula of the utricle

The macula of the utricle is organized structurally to detect tilt of the head in any direction. The mechanoreceptive sensory cells are called hair cells. Their bundles of stereocilia extend into the otolithic membrane. This membrane is a gelatinous material into which calcium carbonate crystals (otoconia) are embedded. Nerve fibers from the VIIIth nerve make contact with hair cells.

Describe the mechanism of accommodation

The mechanism of lens accommodation. When looking at a nearby target, the ciliary muscle contracts (blue arrows), releasing tension in the zonule and allowing the anterior surface of the lens to round up (blue arrow) owing to its own elasticity. When looking at a distant target, the ciliary muscle relaxes, and the unopposed tension in the zonule (red arrows) flattens the lens (red arrow).

Distinguish between lesions of the different levels of the trigeminal tract

The most important issue with regard to CNS lesions is the spatial separation of the spinal trigeminal nucleus and tract from the rest of the trigeminal system. Lesions to the lateral medulla will interrupt pain and temp information from the ipsilateral face, while leaving discriminative touch pathways intact. A lesion in the lateral mid-pons would interrupt the chief sensory nucleus or the first order axons heading to this nucleus; however, this lesion would probably also knock out pain and temperature sensation because these fibers also enter at the mid-pons and must travel to the medulla via the spinal trigeminal tract. Thus, a lateral pons lesion is likely to eliminate BOTH discriminative touch and pain/temp sensation in the ipsilateral face.

Describe the paravertebral ganglia

The paravertebral ganglia run alongside the spinal cord from the cervical to the sacral region of the vertebral column. Preganglionic axons leave the spinal cord via the ventral root, enter the paravertebral ganglia via the white communicating ramus and then run rostral or caudal within the chain before 1) synapsing on a postganglionic neuron within the chain 2) exiting the chain to synapse in a prevertebral ganglion -OR- 3) exiting the chain to synapse in the adrenal medulla. -The preganglionic axons use acetylcholine as their neurotransmitter on the postganglionic neurons. -Postganglionic axons exiting the paravertebral ganglia do so via the gray communicating ramus and they travel with peripheral nerves to reach their targets, which include the eye, salivary glands, blood vessels, skin, heart, lungs, kidney, rectum, bladder, and reproductive organs.

What can block the placebo effect?!

The placebo effect can be blocked by naloxone, which is a competitive antagonist of opiate receptors (i.e. it binds to opiate receptors, thus preventing binding of opiates themselves to the receptors).

Describe the Placebo Effect in Pain

The placebo effect is defined as a physiological response following the administration of a pharmacologically inert remedy. E.g. in a study conducted on medical students given either a "sedative" or a "stimulant" (in fact they are both just sugar pills),more than half the number receiving the "sedative" reported feeling drowsy and a high percentage of students receiving the stimulant reported feeling less tired. Or, in another study, 75% of patients suffering from post-operative wound pain report feeling relief after injection of sterile saline. The placebo effect can be blocked by naloxone, which is a competitive antagonist of opiate receptors (i.e. it binds to opiate receptors, thus preventing binding of opiates themselves to the receptors). The placebo effect underlies the importance of regarding pain as a perception.

Describe the prevertebral ganglia

The prevertebral ganglia are all located adjacent to the aorta. They are innervated by unpaired preganglionic fibers that run within the sympathetic chain then exit to synapse in one of four prevertebral ganglia: the celiac (CG), the aorticorenal (ARG), the superior mesenteric (SMG), or the inferior mesenteric ganglia (IMG). Neurons in these ganglia send their axons to the stomach, superior duodenum, liver, gall bladder, pancreas (CG); the kidney (ARG); the duodenum, jejunum, ilium, ascending and transverse colon (SMG); the transverse colon, descending colon, and pelvic viscera [rectum, bladder, reproductive organs] (IMG).

What constitutes the pyramidal tract?

The pyramidal (corticospinal + corticobulbar) tract actually originates in several areas of cortex: about 1/3 from M1, 1/3 from premotor areas, 1/3 from primary somatosensory cortex. This tract is not exclusively motor in function. Projections from somatosensory cortex terminate in the dorsal horn and in sensory nuclei of the brainstem to modify processing in the somatosensory pathways. The remainder of the pathway includes the lateral and medial corticospinal tracts and the motor component of the corticobulbar tract (which end in motor nuclei of the cranial nerves). These projections terminate on alpha motor neurons, gamma motor neurons and interneurons.

What is the difference between "reception and analysis" and "perception'?

The reception and analysis of a physical stimulus is different from the perception (conscious awareness) of that stimulus. For example, a noxious stimulus may activate nociceptive receptors and sensory fibers, but not result in the perception of pain. This may occur because: -A stimulus may only activate reflex pathways, or -Sensory receptors or neurons may be suppressed by descending modulatory circuits

Describe deep brain stimulation treatment for Parkinson's Disease

There has also been success with a technique called "deep brain stimulation". Interestingly, the stimulation is applied to the subthalamic nucleus or GP (i.e., the same targets of surgical lesioning!). Stimulation parameters can be varied and, by mechanisms not understood, can be varied to activate or inactivate cells in the region of the electrode. One advantage of this technique over surgical lesions is that it is reversible and the level of stimulation can be easily controlled, allowing fine-tuning for beneficial motor effects while limiting side effects.

Describe the Receptive fields of primary somatosensory afferents

The receptive field is the area of the skin surface over which stimulation results in a significant change in the rate of action potentials. The size of the receptive field is a function of the branching characteristics of the afferent within the skin: smaller arborizations result in smaller receptive fields. There are systematic regional variations in the size of afferent receptive fields that reflect the density of afferent fibers supplying the area. The receptive fields in regions of dense innervation (fingers, lips, toes) are small compared to those in the forearm or back that are innervated by a smaller number of afferent fibers.

Generalize the Contributions of the cerebellum and basal ganglia to eye movements.

The roles of the cerebellum and basal ganglia in somatic movement have been described in previous lectures. Each of these systems has regions devoted to eye movements. Not surprisingly, damage to these regions leads to deficits in eye movements that, in many ways, are analogous to deficits seen in the somatic motor system.

Describe the rostral spinocerebellar tract

The rostral spinocerebellar tract carries information from the upper trunk/arm. Second order cells are in the cord scattered rostral to Clarke's nucleus (i.e., at the same relative location in the cord as the dorsal spinocerebellar neurons in Clarke's nucleus which is more caudal); the second order axons ascend ipsilaterally to enter the cerebellum via the inferior cerebellar peduncle.

Describe the anatomy of the sacral parasympathetic preganglionic cells

The sacral parasympathetic preganglionic cells are in the sacral parasympathetic nucleus (S2 to S4) and their axons exit the ventral roots then travel with the pelvic splanchnic nerve to synapse in ganglia near the rectum, the bladder and the reproductive organs. Parasympathetic activity directed toward to target organs leads to defecation, urination and sexual arousal, respectively.

What determines the spatial accuracy with which a tactile stimulus can be sensed?

The size of the receptive field and the innervation density determine the spatial accuracy with which tactile stimuli can be sensed. A higher density of nerve endings gives a better spatial resolution of stimuli.

Describe the Sub-systems of the somatosensory system

The somatosensory system can be divided into functionally distinct sub-systems with distinct sets of peripheral receptors and central pathways. The characteristics of the sensory afferents and peripheral somatosensory receptors affect the way somatosensory information is processed. 1. Discriminative touch. Cutaneous mechanoreceptors skin 2. Sensing the position of limbs and other body parts in space Proprioceptors muscles, tendons, joints. 3. Nociceptors (pain). Temperature receptors.

What subdivisions can the somatosensory system be divided into?

The somatosensory system can be divided into functionally distinct sub-systems with distinct sets of peripheral receptors and central pathways: (1) Cutaneous mechanoreceptors mediate sensation of fine touch, vibration, pressure (2) Proprioceptors are associated with muscles, tendons, joints, and sense position of limbs and other body parts in space (3) Nociceptors are involved in pain sensation and thermoreceptors sense temperature.

What is the Homunculus?

The somatotopic organization continues into the cerebral cortex. This organization gives rise to a map of the body surface called a homunculus (technically, then, a homunculus could be drawn for each of the parts of the pathway; it it is often drawn only for cortex). The figure shows the homunculus in the postcentral gyrus. Note the location of body parts and think about the distribution of cerebral arteries.

Generalize the function of spinocerebellar pathways

The spinocerebellar pathways carry sensory (proprioceptive) information to the cerebellum, which requires this information for production of coordinated movements. Damage to the pathways causes ataxia, or uncoordinated movements. Damage to these pathways does NOT eliminate conscious proprioception (remember, that was handled by the dorsal column system), so we say that the spinocerebellar pathways are carrying information for non-conscious proprioception. Non-conscious proprioception is associated with pathways to the cerebellum; conscious proprioception is associated with pathways to the cerebral cortex.

Describe the ventral trigeminothalamic tract

The ventral trigeminothalamic tract is not well circumscribed, but travels adjacent to the fibers of the medial lemniscus. Of course, as these tracts reach the thalamus, fibers carrying information from the trigeminal system terminate medially (in VPM) and fibers carrying information from the body terminate more laterally (in VPL). Projections from VPM travel in the internal capsule (in the genu or in the adjacent part of the posterior limb) and then in the corona radiata to reach the lateral part (face area) of the postcentral gyrus. (Of course, some pain fibers from the trigeminal system also terminate in intralaminar thalamic nuclei, which project to insula and anterior cingulate cortex).

Describe vertical gaze centers

The vertical gaze center is the rostral interstitial nucleus of the MLF (again, one on each side), which is located in the rostral midbrain. Horizontal or vertical eye movements are accomplished by activation of the appropriate gaze center.

Describe the monocular and binocular zones of the visual fields

The visual field has both binocular and monocular zones. Light from the binocular zone strikes the retinae in both eyes, but light from the monocular zone strikes the retina only in the eye on the same side. For example, light from a left monocular zone (temporal crescent) falls on only the ipsilateral nasal hemiretina and does not project upon the contralateral retina because the nose obstructs the path of light. The temporal and nasal hemiretinas are defined with respect to the fovea, the region in the center of the retina with highest acuity. The optic disc, the region where the ganglion cell axons leave the retina, is free of photoreceptors and therefore creates a gap, or blind spot, in the field for each eye. Each optic nerve carries all the visual information from one eye.

Describe some of the features of basal ganglia circuitry

There are a large number of connections between components of the basal ganglia. A major feature of the circuitry is the role of inhibitory, GABAergic projections from one nucleus to another. Moreover, these inhibitory projections can occur sequentially, whereby one GABAergic cell can inhibit a target GABAergic cell. This sets up a disinhibitory circuit, whereby inhibition of an inhibitory cell can lead to excitation of the ultimate target.

What are sensory modalities?

There are different forms of stimulus energy. These evoke different types of sensations, or sensory modalities (e.g., touch, vision, hearing, smell). Each sensory modality is based on the selective sensitivity of its sensory receptors.

Describe referred pain

There are few dorsal horn neurons in the spinal cord that are specialized solely for visceral pain. Visceral pain is conveyed centrally via dorsal horn neurons that are also concerned with cutaneous pain. As a result, pain in an internal organ is sometimes perceived as cutaneous pain. This is called referred pain. E.g. angina pain (heart wall) referred to the upper chest wall and radiating to the left arm.

What are the basic divisions of taste that we can detect?

There are four (and sometimes five) basic tastes: 1. sweet 2. sour 3. salty 4. bitter 5. umami (the taste associated with monosodium glutamate). -There are specific receptor mechanisms to deal with each type.

Where on the retina is there a lack of photoreceptors?

There are no photoreceptors at "blind spot", where the optic nerve exits and the central retinal artery enters the eye. Know location of blind spots relative to fovea in each eye.

List the chemical senses associated with the nose and mouth

There are three chemical sensory systems associated with the nose and mouth: (1) the olfactory (smell) (2) gustatory (taste) (3) trigeminal (chemosensory irritant) systems. Each of these systems detects chemicals in the environment.

What are the main groups of ascending somatosensory pathways?

There are three main groups of ascending somatosensory pathways: 1. The dorsal (posterior) column/medial lemniscus pathway carries information for discriminative touch, pressure, vibration and conscious proprioception. 2. Anterolateral pathways (spinothalamic pathways) carry information for pain & temperature as well as a "crude" sensation of touch. 3. Spinocerebellar pathways carry proprioceptive information to the cerebellum for motor coordination. This information does NOT reach consciousness via these pathways.

Describe the animal model of Parkinson's Disease

There is now an animal model of PD. A number of drug addicts in California developed PD in their 20's (PD usually appears after age 55). It was discovered that the drugs they were taking contained a contaminant known as MPTP (1-methyl-4-phenyl-1,2,3,6- tetrahydropyridine - you only need to know it as MPTP). MPTP causes degeneration of dopaminergic cells and the appearance of PD. MPTP is now administered to monkeys for study of the physiology, neurochemistry and treatment of PD.

What is the general function of the vestibulospinal tracts?

They are important for maintaining balance; this is reflected in their medial termination in the cord, where they influence axial and proximal limb muscles. They primarily excite the extensors. Note that this applies to both the medial and the lateral vestibulospinal tracts, which can be viewed functionally as part of the medial motor pathways.

Describe the interaction of excitation and inhibition in the cerebellum

Thus cerebellar output is driven by the main excitatory loop, and limited by the inhibitory cortical loop.

What is the impact of different genes in olfactory receptors?

Thus there are 1000 types of receptor cells each identified by the particular receptor gene that it expresses.

In what order to chemical stimuli act on papillae?

Thus, chemical stimuli on the tongue first stimulate receptors in the fungiform papillae and then in the foliate and circumvallate papillae. Tastants subsequently stimulate scattered taste buds in the pharynx, larynx, and upper esophagus.

Give a summary of nystagmus

To review, nystagmus can occur under a variety of normal and pathological conditions; the one that occurs in response to movement of a large part of the visual scene is called optokinetic nystagmus and is a normal reflex. This reflex can be tested by seating a subject in front of a screen on which a series of vertical bars is moved across the screen continuously. A normal reflex requires that both the smooth pursuit and the saccade circuits are intact (and, of course, that the visual sensory and motor cranial nerves and eye muscles are working normally!). Horizontal nystagmus is normally described as "left" or "right", being named according to the direction of the fast component of the movement. If a patient is observing a series of vertical bars moving to the right, she will (hopefully) have a slow smooth pursuit movement to the right followed by a fast saccade to the left. This normal movement would be called a left nystagmus. The vestibulo-ocular and optokinetic reflexes work together to allow stabilization of the visual image; disturbance of either mechanism will cause a patient to report blurriness or a "jumping" or "bouncy" image.

Describe the corneal reflex

Touching one cornea (say, with a bit of cotton) normally causes both eyes to close. Closing of the stimulated eye is called the direct response; closing of the opposite eye is called the consensual response. Sensory path: pain fibers in V ophthalmic division projecting to spinal trigeminal nucleus pars caudalis. Spinal V nucleus pars caudalis projects to the reticular formation, which projects bilaterally to the facial nucleus. Motor path: facial nucleus to orbicularis oculi to close lids. The motor and sensory limbs are linked by projections from spinal V nucleus to reticular formation to both facial nuclei. This reflex tests components of both V and VII.

Describe the role of the middle ear muscles. What are they?

Transmission is controlled by the middle ear muscles. Contractions increase stiffness of the ossicular chain, mainly attenuating low frequencies. i. Stapedius muscle innervated by facial nerve. ii. Tensor tympani muscle innervated by trigeminal nerve.

Describe the treatment direct for Parkinson's Disease

Treatment is directed at replacing the dopamine. Dopamine itself does not cross the blood-brain barrier, but a precursor, L-DOPA (dihydroxyphenyalanine), does. Combinations of drugs are used to increase the efficacy of L-DOPA and minimize side effects. Drug treatment often largely eliminates symptoms for a time. However, the degeneration of dopamine neurons continues, and treatment becomes less effective. Other approaches have been tried, with limited success. One treatment involves transplantation of dopamine-producing cells into the lateral ventricles. Success was limited and this approach has been abandoned. Surgical lesioning of the globus pallidus ("pallidotomy") or the subthalamic nucleus reduces inhibition of VA/VL and can relieve some PD symptoms.

Describe trigeminal neuralgia

Trigeminal neuralgia (tic douloureux) is a condition characterized by repetitive, transient intense stabbing pain in a region innervated by a branch of the trigeminal nerve. The pain is frequently elicited by stimuli that are normally non-painful (e.g., light touch of the cheek, chewing).

What makes the olfactory system unique?

Two features of olfactory receptors make the olfactory system unique among the sensory systems. -First, the olfactory system is the only one in which the first synapse occurs in the telencephalon (this is because the olfactory bulb is part of the telencephalon). -Second, each of the approximately 100 million olfactory receptor cells is replaced about every two months. The olfactory receptors, which are neurons, are constantly dying and being replaced by new cells which are derived from the basal cells). (Recall that taste receptors also turn over, but they are not neurons).

57 year old woman, "had a stroke" a couple years ago; "partly paralyzed" stiff right arm and leg; but can walk on the stiff leg.

UMN, with spasticity.

8. Describe the major symptoms of UMN syndrome. How does this differ from LMN (lower motor neuron) syndrome?

UMN: -Paralysis or paresis -Spasticity - hypertonia - hyperreflexia and maybe: -Babinski sign -Clonus -Decreased superficial reflexes LMN -Flaccid paralysis or paresis (weakness) -Hypo- or areflexia -Decreased muscle tone -Atrophy

Compare and contrast upper motor neuron and lower motor neuron syndrome.

UMN: • weakness, or paralysis • spasticity • increased tone • increased deep reflexes -perhaps: • clonus • Babinski'ssign • loss of fine voluntary movements • decreased superficial reflexes LMN: • weakness, or paralysis • decreased tone • decreased deep reflexes -perhaps: • decreased superficial reflexes

What are some of the main brainstem motor control centers?

Upper Motor Neurons • in vestibular nuclei and reticular formation for balance and posture • in superior colliculus for gaze control (not shown)

What are vestibule-ocular and optokinetic movements?

Vestibulo-ocular movements and optokinetic movements serve to stabilize the image of the world on the retina. Head movements cause a relative motion between the retina and the visual world. However, rotations of the head are normally accompanied by equal and opposite rotations of the eyes. As we saw in a previous lecture, the vestibular system detects head movements and produces compensatory vestibulo-ocular movements.

Describe the vestibulocerebellum

Vestibulocerebellum-flocculonodular lobe and the fastigial nucleus. The main function is balance and eye movements.

How does viagra work?

Viagra works by inhibiting PDE-5, the phosphodiesterase produced in erectile tissue that normally leads to a decrease in cGMP and lower NOS activity. By inhibiting PDE-5, high cGMP allows NOS to make nitric oxide.

What is conductive loss of hearing?

Vibrations associated with sound are not properly conducted (mechanically) into the inner ear. Results from disorders in the outer or middle ear (e.g. otitis media, otosclerosis).

What is sensorineural hearing loss?

Vibrations in the inner ear are not properly converted into electrical potentials or are not transmitted into the central nervous system. Results from disorders of hair cells or auditory nerve

Describe the importance of vision

Vision is fundamental to human behavior. We use it to orient within the world, to communicate, and even to entrain our daily activity patterns. Pathways underlying visual perception occupy a significant part of our brains. Moreover, vision depends on precise control of the eyes, requiring additional neural machinery. A clinical implication is that lesions occurring in many parts of the brain can impair aspects of vision.

Where do upper motor neurons rank in the motor neuron hierarchy?

With lower motor neurons representing the first level of the motor hierarchy, upper motor neurons (UMNs) represent the second level of the hierarchy. UMNs are organized into a variety of pathways that allow for voluntary control of movement. Some of these movements are results of conscious decisions to make a movement, while others are more reflexive and rely on a variety of sensory inputs.

What are the outputs of the hypothalamus?

a. Brainstem and other regions for autonomic control b. Neuroendocrine outputs c. Somatic motor control d. Limbic system and cerebral corte

What eye muscle nerves are crossed?

You may recall that the trochlear nerve is crossed - the axons from each trochlear nucleus innervate the superior oblique muscle on the opposite side. The oculomotor "nucleus" (actually a complex of nuclei) has separate cell groups innervating each of the muscles served by the third nerve. The axons that innervate the superior rectus are crossed (i.e., their cell bodies are on the side opposite to the muscle they innervate); axons to the other muscles are not crossed. The fact that the superior rectus and superior oblique muscles get crossed innervation will not complicate our discussion of control of eye movements because the actions of the eye muscles are coordinated by separate gaze centers.

What are the "shapes" of many limbic structures?

a. Cingulate and parahippocampal gyri, cingulum b. Hippocampus and fornix c. Amygdala/stria terminalis d. Caudate nucleus and lateral ventricles also follow this organization

What is reciprocal inhibition?

a. A general principle: excitation of motorneurons to skeletal muscles having one mechanical action also results in inhibition of motorneurons to skeletal muscles having the antagonistic mechanical action. b. Mediated by inhibitory interneurons activated by afferents from the opposing muscle c. Reciprocal innervation results in rapid contraction of the stretched muscle and simultaneous relaxation of the antagonist muscle. c. Occurs in a wide range of reflexes and in voluntary movement.

Describe bladder control after a spinal cord injury

a. Acute Period: No descending input. No signal to reduce sympathetic tone. No bladder emptying. b. Chronic Period: Filling, holding and voiding can occur smoothly after weeks to months of reorganization to achieve cord-level reflex function, though no conscious control. Stronger interneuron contribution.

How does the hypothalamus regulate endocrine systems?

a. Anterior pituitary and control of multiple glands b. Posterior pituitary

Does the optic tract only contain crossed components?

a. Axons from ipsilateral temporal hemiretina and contralateral nasal hemiretina b. Ganglion cells with axons in one optic tract all view the contralateral visual field c. The majority of fibers in the optic tract terminate in the lateral geniculate nucleus

Describe the optic chiasm

a. Axons of ganglion cells in lateral hemiretina proceed ipsilaterally into optic tract b. Axons of ganglion cells in nasal hemiretina cross through chiasm to contralateral optic tract

What are some general pathologies of the auditory conductive system? What are their effects?

a. Blockage of external ear canal. b. Perforation or rupture of tympanic membrane. c. Disruption of conductive system reduces sensitivity by as much as 30 decibels for lower frequencies, and is more severe at higher frequencies. d. Paralysis of middle ear muscles can distort sound conduction through middle ear (sometimes occurs in Bell 's palsy).

Describe the anatomy of sound transduction

a. Cochlea is a long, coiled tube (~35 mm, ~2.5 turns) extending from the base to the apex b. There are three fluid compartments in the cochlea (Scala Vestibuli, Scala Media, Scala Tympani) c. Basilar membrane d. Organ of Corti e. Inner hair cells f. Outer hair cells

How does the hypothalamus control visceral motor function?

a. Direct projections to autonomic preganglionic neurons in brainstem and spinal cord b. Projections to autonomic centers in brainstem

What are the diencephalic portions of the limbic system?

a. Dorsal thalamus: anterior nucleus and medial dorsal nucleus (=dorsomedial nucleus) b. Habenula c. Mammillary bodies

Describe the structure of a hair cell

a. Each has 40-70 stereocilia and one larger kinocilium. b. Stereocilia differ in length; there is a gradation from short to long in the direction of the kinocilium. c. Adjacent hair cells are bound by tight junctions. d. Hair cells have synaptic vesicles.

Describe the Common Principles of Organization of central sensory neurons

a. Elements and organization of ascending pathways. Each nucleus of an ascending sensory pathway may perform a unique analysis of sensory information, then project the analyzed information to the next level. b. Parallel organization of ascending pathways. Different features of a sensory stimulus may be separately analyzed and represented throughout ascending pathways (e.g., somatic sensory pathways (Fig.8), visual pathways that separately process the form and motion of a visual stimulus). c. Topographic sensory representations (= sensory maps). In most systems, there is an orderly representation of the sensory surface, present at both sub-cortical and cortical levels (e.g. somatotopic organization in both dorsal column nuclei and somatosensory cortex) (Fig. 9). However, different parts of sensory surface are not represented equally.

How does the hypothalamus regulate Central control of homeostatic mechanisms?

a. Energy balance and food intake b. Thermoregulation c. Blood volume and osmolality d. Circadian rhythms

What act as satiety signals?

a. Insulin acts directly on neurons of the arcuate nucleus and ventromedial hypothalamus. b. Gastric distention (input via vagus to n. solitarius) c. Cholecystokinin (CCK) d. Blood glucose may be sensed directly by hypothalamic neurons in the ventromedial hypothalamus

What are the primary features of reflexes?

a. Involuntary, but many can be modulated by higher centers and other inputs b. At minimum, consist of sensory neuron, motor neuron, and a single synapse between the two. c. Reflexes maintain consistent internal environment and a nearly automatic reaction to certain sensory stimuli.

Describe the functional properties of the lateral geniculate nucleus

a. LGN layers maintain separate M and P channels, and ocular dominance patterns b. Receptive fields are similar to retinal ganglion cells • center-surround • ON-center and OFF-center c. LGN has systematic representation of the two hemi-retinas, forming a systematic representation of the contralateral visual field d. Response is modulated by descending cortico-geniculate projections

Describe the superior colliculus pathway of the brachium of the superior colliculus

a. Located dorsal and slightly caudal to pretectum b. Superior Colliculus on one side receives direct input from parts of both retinas viewing contralateral visual field (input from contralateral eye crosses in optic chiasm). c. Functions in orienting eyes, head, and neck to novel visual (and other sensory) stimuli.

Describe the anatomy of the lateral geniculate nucleus

a. Located in caudal, lateral thalamus b. Six layers receive different inputs i. Ipsilateral vs contralateral input ii. M-type vs P-type retinal ganglion cells c. Highly organized inputs from the two retinae preserve topographic visual field representation.

Describe the Brain regions associated with reward are targets of addictive drugs and behaviors

a. Many addictive substances potentiate the effect of dopamine on neurons of the nucleus accumbens -Some substances act on dopaminergic ventral tegmental neurons to increase release onto nucleus accumbens neurons. (e.g., alcohol, opiates, nicotine) -Some substances act in nucleus accumbens to enhance dopamine release. (e.g., opiates, cannabinoids, nicotine, cocaine, amphetamines) b. Dopaminergic neurons of the ventral tegmental area project to other centers of the limbic system ("mesolimbic pathway") or to cortical areas ("mesocorticolimbic pathways") to influence motivated behaviors.

Describe Functional properties of visual neurons

a. Receptive field properties change from retina to cortex b. Parallel pathways analyze information about visual form/color and motion/location

What are the inputs of the hypothalamus?

a. Sensory information b. Higher centers such as limbic system

Describe sensory stimulus coding

a. Sensory stimuli vary along several dimensions, e.g., intensity, timing, location, modality. b. While receptors produce graded electrical potentials, central sensory neurons discharge all-or-none action potentials. The information within a stimulus must be represented by sensory neurons using one or more neural codes:

What complex systems is the hypothalamus involved in?

a. Sexual function, involving endocrine and visceral motor function as well as behavior b. Emotional responses, under command of limbic system c. Memory systems involve hypothalamus

What are some of the primary signs of vestibular dysfunction?

a. Spontaneous or inappropriate nystagmus b. Diminished antigravity reflexes c. Motion sickness (mismatch between vestibular and visual cues) d. Vertigo (sensation of spinning: perceived bodily motion)

3. What part(s) of the basal ganglia are associated with the following diseases (be as specific as you can, including nuclei and neurotransmitters, if known): a. Huntington's disease b. Parkinson's disease c. Hemiballism

a. Striatum b. SN pars compacta, nigrostriatal pathway, dopamine c. subthalamic nucleus

Describe the pupillary light reflex

a. The reflex: Light shone in one eye causes constriction of same pupil (direct light reflex) and constriction of pupil in contralateral eye (consensual light reflex) b. Pathway (constriction portion of path is under parasympathetic control): 1. Afferent path: Retinal ganglion cells project to pretectum 2. Efferent path: Pretectum projects bilaterally to-->Edinger--Westphal Nucleus-->Ciliary ganglion--> • Pupillary sphincters in ipsilateral eye (direct light reflex) • Pupillary sphincters in contralateral eye (consensual light reflex)

What is the auditory function of the middle ear?

a. Transmission of sound energy to inner ear. b. Transmission is controlled by the middle ear muscles. -Contractions increase stiffness of the ossicular chain, mainly attenuating low frequencies.

What can cause hearing disorders?

a. age- (presbycusis) b. use (e.g., exposure to noise or loud sounds) c. pathological processes (e.g. tumors, vascular accidents, genetic defects) d. toxic substances: aminoglycoside antibiotics

What centers can override the vestibular reflex?

a. cerebellum b. cerebral cortex

What are the neocortical portions of the limbic system?

a. cingulate gyrus, b. orbitofrontal and medial prefrontal cortex c. temporal pole d. subcallosal area

Describe the inhibitory interactions of sensory neurons

a. control overall level of excitability of sensory neurons (Fig. 7A). b. modify the temporal pattern of action potential discharges by a neuron (Fig. 6A) c. compare information from receptors located on different parts of a sensory surface

Describe the pathway that light takes through the eye

a. cornea (focusing)-->pupil (control light entering)-->anterior chamber (nutrients to cornea and lens)-->lens (focusing/accommodation)-->vitreous humor-->retina (photoreceptors) b. choroid contains capillary bed(for nourishing photoreceptors within retina) and melanin (light-absorbing pigment)

For the dorsal column/medial lemniscus system: a. What receptors are involved? b. What peripheral nerve fiber groups are involved? c. Where are the 1st order cells? 2nd order? 3rd order? d. Where (at what level of the neuraxis) does each pathway cross the midline? The axon of which order cell does the crossing?

a. receptors - encapsulated receptors, including Meissner, Merkel, Ruffini and Pacini; Golgi tendon organs, muscle spindles; free endings around hairs b. fibers - A-alpha, A-beta c. 1st order - dorsal root ganglion 2nd order - dorsal column nuclei 3rd order - thalamus (VPL) d. where cross? - lower medulla, internal arcuate fibers - 2nd order cell

For the anterolateral system: a. What receptors are involved? b. What peripheral nerve fiber groups are involved? c. Where are the 1st order cells? 2nd order? 3rd order? d. Where (at what level of the neuraxis) does each pathway cross the midline? The axon of which order cell does the crossing?

a. receptors - free nerve endings b. fibers - A-delta, C c. 1st order - dorsal root ganglion 2nd order - dorsal horn 3rd order - thalamus (VPL, intralaminar) d. where cross? - spinal cord, ventral white commissure - 2nd order cell

4. List the structures traversed by axons of the lateral corticospinal tract,starting from their cell bodies and continuing to their termination.

corona radiata, internal capsule posterior limb, cerebral peduncle, fiber bundles in base of pons, pyramid, pyramidal decussation and lateral corticospinal tract.

Describe the mechanisms of the center and surround stimulation of retinal ganglion cells

i. Center response is based on one or a few photoreceptors that connect directly to bipolar cell, which in turn connects to the ganglion cell (Fig. 13A,B). ii. In an ON-center ganglion cell, a light stimulus that shines on the center of its receptor field (on the center cones) causes increased numbers of action potentials. (Fig. 13B). iii. Surround response is based on many photoreceptors that surround the center photoreceptor (Fig. 13A). These connect to the center cone/bipolar cell via horizontal cells iv. When a light stimulus shines on the OFF-surround part of the ganglion cell's receptive field (i.e., on the photoreceptors that contribute to the surround), the net result is a decrease in the numbers of action potentials evoked by that ganglion cell (Fig. 12C). v. Since center and the surround have different effects on the ganglion cell's discharge, light shining over the entire receptive field has relatively little effect.

Describe how Rewarding behaviors are reinforced and regulated by a "basal ganglia" type circuit involving the ventral striatum, ventral pallidum, and dopaminergic neurons of the ventral tegmental area.

i. The nucleus accumbens is the major component of the ventral striatum ii. Together with the ventral pallidum, it has a reinforcing role on activity in neocortical areas associated with reward, via the mediodorsal nucleus of the thalamus iii. The activity of the nucleus accumbens is modulated by dopaminergic input from the ventral tegmental area adjacent to the substantia nigra (Fig. 6B).

What is the reticulospinal tract?

• originates from RF in pons and medulla • terminates medially in ventral horn of sp. cord, i.e., part of the "medial system" of descending motor control ***The reticulospinal tracts tend to enhance tone in antigravity muscles (axial and proximal limb extensors).***

Describe the main projections of the trigeminal nuclei

• the cerebellum (from spinal V, equal to spinocerebellar paths) • reticular formation and then to thalamus (equal to spino-reticulo-thalamic path)

What is Upper Motor Neuron Syndrome?

• weakness (paresis), or paralysis • spasticity • hypertonia • hyperreflexia -UMN lesion causes "spastic paralysis". Contrast with flaccid paralysis (LMN lesion).

What are some additional signs of upper motor neuron damage?

• weakness, or paralysis • spasticity • hypertonia • hyperreflexia Or: • Babinski's sign • clonus • loss of fine voluntary movements • reduced superficial reflexes


Related study sets

Wk 2 - Practice: Ch. 5, Competitive Advantage, Firm Performance

View Set

U.S. History Ch. 16-17 Practice Test

View Set

Risk Assessment and Response to Assessed Risks

View Set

Real Estate, Chapter 8, Lesson Cumulative Quiz

View Set

Chapter 7: Segmentation, Target Marketing and Positioning

View Set

договори та терміни

View Set